/
ESAP-ITE 2019 Slide Deck ESAP-ITE 2019 Slide Deck

ESAP-ITE 2019 Slide Deck - PowerPoint Presentation

myesha-ticknor
myesha-ticknor . @myesha-ticknor
Follow
416 views
Uploaded On 2019-11-25

ESAP-ITE 2019 Slide Deck - PPT Presentation

ESAPITE 2019 Slide Deck ADRENAL ITE 2019 Question 3 A 54yearold man has had difficulttocontrol hypertension for 3 years He has gained 33 lb 15 kg in the last 3 years and diabetes mellitus was diagnosed 1 year ago His medications include metformin amlodipine valsartan and hydrochloroth ID: 767934

patient answer patients diabetes answer patient diabetes patients mmol pmid insulin therapy nmol thyroid rationale normal year miu levels

Share:

Link:

Embed:

Download Presentation from below link

Download Presentation The PPT/PDF document "ESAP-ITE 2019 Slide Deck" is the property of its rightful owner. Permission is granted to download and print the materials on this web site for personal, non-commercial use only, and to display it on your personal computer provided you do not modify the materials and that you retain all copyright notices contained in the materials. By downloading content from our website, you accept the terms of this agreement.


Presentation Transcript

ESAP-ITE 2019 Slide Deck

ADRENAL

ITE 2019 Question 3 A 54-year-old man has had difficult-to-control hypertension for 3 years. He has gained 33 lb (15 kg) in the last 3 years and diabetes mellitus was diagnosed 1 year ago. His medications include metformin, amlodipine, valsartan, and hydrochlorothiazide. On physical examination, his BMI is 35 kg/m2 and blood pressure is 148/92 mm Hg. He has scattered 1- to 2-cm bruises on his extremities. He has no dorsocervical fat pad, striae, or proximal muscle strength weakness. Laboratory test results: Sodium = 139 mEq /L (136-142 mEq /L) (SI: 139 mmol/L [136-142 mmol/L]) Potassium = 3.5 mEq /L (3.5-5.0 mEq /L) (SI: 3.5 mmol/L [3.5-5.0 mmol/L]) Serum aldosterone = 14 ng/dL (4-21 ng/dL) (SI: 388.4 pmol /L [111.0-582.5 pmol /L]) Plasma renin activity = <0.6 ng/mL per h (0.6-4.3 ng/mL per h) Fasting glucose = 165 mg/dL (70-99 mg/dL) (SI: 9.2 mmol/L [3.9-5.5 mmol/L])

Abdominal CT with contrast demonstrates a 3.8-cm left adrenal mass and an atrophic right adrenal gland (see image, arrows).

Which of the following is the best next step in this patient’s management? Correct Answer: B Learning objective : Exclude hypercortisolism and pheochromocytoma when evaluating a large adrenal tumor suspected of causing primary aldosteronism. Prescribe spironolactone, 50 mg daily Measure plasma metanephrines and perform a 1-mg overnight dexamethasone suppression test Schedule adrenal venous sampling to measure aldosterone and cortisol Perform left adrenalectomy Measure serum aldosterone after administration of 2 L normal saline

Rationale: This patient was evaluated for primary aldosteronism because of resistant hypertension and low-normal potassium. The screening aldosterone-to-renin ratio is marginally positive, disproportionate to his hypertension. The weight gain, hyperglycemia, central obesity, and bruising are suggestive of hypercortisolism. His adrenal tumor is markedly larger than those that usually cause primary aldosteronism, and the contralateral adrenal gland is somewhat atrophic, suggesting hypercortisolism from the adrenal tumor. When the diameter of adrenal cortical tumors is greater than 2.4 cm, the risk of hypercortisolism rises, and if the tumor is removed without testing cortisol dynamics, adrenal crisis might occur postoperatively. The CT was performed with contrast, so density cannot be used to determine whether the tumor is lipid-rich and thus unlikely to be pheochromocytoma. Hypercortisolism and pheochromocytoma must be excluded before performing left adrenalectomy (Answer D) to avoid perioperative complications. With a large adrenal tumor, measuring plasma metanephrines and performing a 1-mg overnight dexamethasone suppression test (Answer B) is the correct next step.Spironolactone now (Answer A) would treat the mineralocorticoid excess but not glucocorticoid manifestations, and this patient needs further evaluation before treatment is determined. Confirmatory testing with saline infusion (Answer E) and adrenal venous sampling (Answer C) are customary steps for the evaluation of primary aldosteronism;

Rationale Continued: however, this patient’s large adrenal tumor first warrants complete hormonal evaluation. After further laboratory evaluation, a diagnosis of ACTH-independent hypercortisolism was established in this patient. Cortisol excess takes priority over aldosterone and directs treatment with left adrenalectomy and perioperative glucocorticoid coverage. Coproduction of aldosterone and cortisol from adrenal cortical adenomas, particularly larger tumors, is well described. It is possible, although unlikely, that the primary aldosteronism is bilateral and unrelated to the adrenal tumor, so the patient should be rescreened for primary aldosteronism after adrenalectomy. If it persists, spironolactone would be an appropriate treatment. Educational Objective Exclude hypercortisolism and pheochromocytoma when evaluating a large adrenal tumor suspected of causing primary aldosteronism.

Reference(s): Spath M, Korovkin S, Antke C, Anlauf M, Willenberg HS. Aldosterone- and cortisol-co-secreting adrenal tumors: the lost subtype of primary aldosteronism. Eur J Endocrinol. 2011;164(4):447-455. PMID: 21270113Morelli V, Reimondo G, Giordano R, et al. Long-term follow-up in adrenal incidentalomas: an Italian multicenter study. J Clin Endocrinol Metab. 2014;99(3):827-834. PMID: 24423350Fallo F, Bertello C, Tizzani D, et al. Concurrent primary aldosteronism and subclinical cortisol hypersecretion: a prospective study. J Hypertens. 2011;29(9):1773-1777. PMID: 21720261

ITE 2019 Question 12 A 31-year-old man presents with abdominal pain near his umbilicus. He describes symptoms of bloating and central abdominal pain for several months. He has no notable medical history or past surgeries and takes no medications. He does not smoke cigarettes. On physical examination, he is afebrile, blood pressure is 122/80 mm Hg, and pulse rate is 70 beats/min. His height is 69 in (175 cm), and weight is 148 lb (67.3 kg) (BMI = 21.9 kg/m2). He has no signs of Cushing syndrome and there is no palpable abdominal pain or mass. Abdominal CT with intravenous contrast documents a heterogeneously enhancing right adrenal mass measuring 3.4 x 3.2 cm (see images, arrows). The mass is relatively round and confined to the right adrenal gland. The left adrenal gland appears normal.

Axial View Coronal View

He has no history of episodic spells of adrenergic symptoms, including no sensations of palpitations, sweating, anxiety, tremors, pallor, flushing, headaches, or vision changes. He has had no known episodes of hypertension or orthostasis and no history of weight gain, weight loss, excessive virilization, or feminization. Laboratory test results: Serum cortisol following 1-mg dexamethasone suppression test = 0.7 µg/dL (SI: 19.3 nmol/L) Plasma ACTH following 1-mg dexamethasone suppression test = <5.0 pg /mL (SI: <1.1 pmol /L)Plasma renin activity = 1.5 ng/mL per h (0.6-4.3 ng/mL per h)Serum aldosterone = 5.0 ng/dL (4-21 ng/dL) (SI: 138.7 pmol/L [111.0-582.5 pmol/L])Plasma metanephrine = <39 pg/mL (<99 pg/mL) (SI: <0.20 nmol/L [<0.50 nmol/L])Plasma normetanephrine = 1245 pg/mL (<165 pg/mL) (SI: 6.8 nmol/L [<0.90 nmol/L])Urinary free cortisol = 37 µg/24 h (4-50 µg/24 h) (SI: 102 nmol/d [11-138 nmol/d])

Which of the following is the most likely diagnosis? Correct Answer: E Learning objective : Diagnose pheochromocytoma on the basis of radiographic and biochemical characteristics, even if a patient does not have classic adrenergic symptoms or spells. Adrenocortical carcinoma Hyperfunctioning adrenocortical adenoma Nonfunctional lipid-poor adrenocortical adenomaNonfunctional lipid-rich adrenocortical adenomaPheochromocytoma

Rationale: Pheochromocytomas (Answer E) are chromaffin-cell tumors that originate in the adrenal medulla. The diagnosis of pheochromocytoma should involve evaluation of clinical symptoms, evidence of hormonal activity, and radiographic appearance.Pheochromocytomas may secrete catecholamines (such as norepinephrine, epinephrine, and, less frequently, dopamine) that induce adrenergic symptoms and signs, such as palpitations, anxiety, sweating, pallor, and elevations in blood pressure and heart rate. However, some pheochromocytomas do not secrete high concentrations of catecholamines, and in some instances, even when high catecholamine concentrations are detected some patients do not exhibit the classic symptoms. Typically, pheochromocytomas that induce clinical symptoms are associated with metanephrine and/or normetanephrine levels that are substantially higher than the upper limit of the reference range—usually 4 times or more (less commonly 2 or 3 times higher). Importantly, mild elevations above the upper limit of the metanephrines reference range (<2 times) are common and are usually attributed to enhanced sympathoadrenergic tone (eg, in a state of anxiety or stress) and/or the use of norepinephrine reuptake inhibitors (eg, some antidepressants medications and cocaine). In this regard, these milder elevations are frequent causes of false-positive values. Metanephrines are inactive metabolites of catecholamines and elevations either suggest the secretion of high circulating concentrations of catecholamines, or that catecholamines are being metabolized in a tumor to inactive metanephrines before secretion.

Rationale Continued: The absence of classic adrenergic symptoms in this patient does not exclude a pheochromocytoma; however, the marked elevation in normetanephrine levels (approximately 7.5 times the upper limit of the reference range) strongly suggests that this adrenal tumor is a pheochromocytoma.Once the clinical and/or biochemical characteristics either confirm or strongly suggest a pheochromocytoma, attention should be paid to the radiographic features. Pheochromocytomas are typically 2 cm or larger when a clinical syndrome of adrenergic excess is detected. However, the incidental detection of a pheochromocytoma can occur at any size. Pheochromocytomas tend to be dense and vascular. Therefore, they often have high attenuation on unenhanced CT imaging (>10 Hounsfield units) or high contrast avidity (often with heterogeneous enhancement) on CT imaging done with intravenous contrast, as in this patient’s case, or they have poor delayed contrast washout when CT imaging is performed with an adrenal washout protocol. On MRI, pheochromocytomas tend to display hyperintensity on T2-weighted imaging and have features suggestive of low lipid content (no loss of signal on out-of-phase sequences).The constellation of biochemical and radiographic findings in this vignette argues against a benign or malignant adrenocortical neoplasm (thus, Answers A, C, and D are incorrect). There is no biochemical evidence of aldosterone or cortisol excess (thus, Answer B is incorrect).This patient underwent a laparoscopic right adrenalectomy after preoperative α-adrenergic blockade, and pathologic examination revealed a 3.5-cm pheochromocytoma. Postoperatively, he did report a marked improvement in his overall anxiety level, which he had previously considered to be normal.

Reference(s): Lenders JW, Duh QY, Eisenhofer G, et al; Endocrine Society. Pheochromocytoma and paraganglioma: an Endocrine Society clinical practice guideline. J Clin Endocrinol Metab . 2014;99(6):1915-1942. PMID: 24893135 Fassnacht M, Arlt W, Bancos I, et al. Management of adrenal incidentalomas: European Society of Endocrinology clinical practice guideline in collaboration with the European Network for the Study of Adrenal Tumors. Eur J Endocrinol. 2016;175(2):G1-G34. PMID: 27390021

ITE 2019 Question 31 A 46-year-old man is referred for evaluation of resistant hypertension. Chlorthalidone was discontinued 1 month ago due to hypokalemia. He is currently treated with amlodipine, benazepril, carvedilol, and hydralazine. On this regimen, he was screened for primary aldosteronism 1 week ago. Laboratory test results: Sodium = 139 mEq /L (136-142 mEq /L) (SI: 139 mmol/L [136-142 mmol/L])Potassium = 2.9 mEq/L (3.5-5.0 mEq/L) (SI: 2.9 mmol/L [3.5-5.0 mmol/L])Serum aldosterone = 8 ng/dL (4-21 ng/dL) (SI: 221.9 pmol/L [111.0-582.5 pmol/L])Plasma renin activity = 0.8 ng/mL per h (0.6-4.3 ng/mL per h)

Which of the following should be your next step on the basis of these test results? Correct Answer: B Learning objective : IDENTIFY CAUSES OF FALSE-NEGATIVE SCREENING FOR PRIMARY ALDOSTERONISM. No further testing; primary aldosteronism has been excluded Rescreen after correcting hypokalemia Rescreen after stopping hydralazineRescreen after substituting doxazosin for amlodipineProceed to adrenal venous sampling

Rationale: Although many medications interact with the renin-angiotensin-aldosterone axis, most antihypertensive agents act by vasodilation or volume depletion, which tend to raise plasma renin activity. Thus, when the renin is low (<1 ng/mL per h), the aldosterone-to-renin ratio is generally valid. β-Adrenergic blockers can lower renin, but aldosterone falls in parallel. The aldosterone-to-renin ratio in this case is 10, which is intermediate between a clearly positive screen (>20) and a normal screen (<4). During proper preparation for screening, however, it is important to first correct hypokalemia, as low potassium impairs aldosterone production. Thus, in a setting of high clinical suspicion and an unusual result with a very low potassium level, the screen cannot be dismissed as normal and should be repeated after correcting the hypokalemia (thus, Answer B is correct and Answer A is incorrect). Hydralazine and amlodipine do not significantly interfere with screening because the plasma renin activity is less than 1 ng/mL per hour. Thus, rescreening after stopping hydralazine (Answer C) or amlodipine (Answer D) is incorrect). Adrenal venous sampling (Answer E) is a localizing study that is conducted after confirming the diagnosis and performing cross-sectional imaging.

Reference(s): Raizman JE, Diamandis EP, Holmes D, Stowasser M, Auchus R, Cavalier E. A renin- ssance in primary aldosteronism testing: obstacles and opportunities for screening, diagnosis, and management. Clin Chem. 2015;61(8):1022-1027. PMID: 26106077Funder JW, Carey RM, Mantero F, et al. The management of primary aldosteronism: case detection, diagnosis, and treatment: an Endocrine Society Clinical Practice Guideline. J Clin Endocrinol Metab. 2016;101(5):1889-1916. PMID: 26934393

ITE 2019 Question 37 A 59-year-old woman is referred for evaluation of possible Cushing syndrome after bilateral adrenal nodules were found incidentally 6 months earlier. She has gained 150 lb (68.2 kg) over the past 15 years and has developed hypertension, type 2 diabetes mellitus, hyperlipidemia, sleep apnea, and depression. She has progressive weakness and relies on a wheelchair. On physical examination, she is a morbidly obese woman. Her blood pressure is 135/82 mm Hg. Her height is 67 in (170.2 cm), and weight is 359 lb (163.2 kg) (BMI = 56.2 kg/m2). She has facial rounding, plethora, acanthosis nigricans on her neck, multiple skin tags, and a marked increase in supraclavicular fullness and dorsocervical fat accumulation. Cutaneous wasting is present with multiple ecchymoses on her arms and hands. She has 2+ pretibial edema and proximal muscle weakness.

Laboratory test results: Electrolytes, normal Complete blood cell count, normal Hemoglobin A1c = 8.3% (4.0%-5.6%) (67 mmol/mol [20-38 mmol/mol]) Aldosterone = 4.4 ng/dL (4-21 ng/dL) (SI: 122.1 pmol /L [111.0-582.5 pmol/L])Plasma renin activity = 9.1 ng/mL per h (0.6-4.3 ng/mL per h)Urinary free cortisol (2 measurements) = 31 µg/24 h (4-50 µg/24 h) (SI: 85.6 nmol/d [11-138 nmol/d]) (second measurement: 41 µg/24 h [SI: 113.2 nmol/d]). Creatinine measurements on each sample confirm these were both full 24 hour collections.Late-night salivary cortisol (4 measurements): 0.43 µg/dL (<0.13 µg/dL) (SI: 11.9 nmol/L [<3.6 nmol/L]) (second, third, and fourth measurements: 0.17 µg/dL [SI: 4.69 nmol/L], 0.30 µg/dL [SI: 8.28 nmol/L], and 0.24 µg/dL [SI: 6.62 nmol/L])Cortisol after overnight 1-mg dexamethasone suppression test = 8.3 µg/dL (SI: 229.0 nmol/L)Basal plasma ACTH = 53 pg/mL (10-60 pg/mL) (SI: 11.7 pmol/L [2.2-13.2 pmol/L])

Pituitary MRI findings are normal. Abdominal CT performed 6 months earlier is reviewed. Bilateral adrenal nodules (8 Hounsfield units on the right side and 4 Hounsfield units on the left side) are identified (see image, arrows).

Which of the following diagnostic studies should you recommend next? Bilateral adrenal venous sampling for cortisol and aldosterone Measurement of plasma free fractionated metanephrines Bilateral inferior petrosal sinus sampling for ACTH Dexamethasone–corticotropin-releasing hormone test CT-guided percutaneous biopsy of the right adrenal nodule Correct Answer: CLearning objective: Recommend bilateral inferior petrosal sinus sampling for ACTH as the diagnostic test of choice in patients with ACTH-dependent hypercortisolism and a normal pituitary imaging study.

Rationale: The diagnosis and differential diagnosis of endogenous Cushing syndrome is the one of the most challenging problems in clinical endocrinology. The clinical and biochemical findings in this patient certainly support the diagnosis of true Cushing syndrome. Several late-night salivary cortisol measurements are well above the normal reference range and results from an overnight 1-mg dexamethasone suppression test are abnormal. In light of the clinical findings in this patient, the diagnosis of endogenous hypercortisolism is secure. Although the 24-hour urinary free cortisol excretion is in the normal range, it has become increasingly appreciated that urinary free cortisol is an insensitive diagnostic test in the evaluation of patients with suspected hypercortisolism. The sensitivity of urinary cortisol is only 70% for the diagnosis of Cushing syndrome. Longitudinal studies in patients after pituitary surgery for Cushing disease have shown that an increase in late-night cortisol secretion is the earliest biochemically detectable abnormality during recurrence and that urinary free cortisol elevation is a much later event. The sensitivity of late-night salivary cortisol and the overnight 1-mg dexamethasone suppression test for Cushing disease is 94% to 98%. Because the clinical and biochemical data in this patient demonstrate unequivocal Cushing syndrome, no other studies to confirm the diagnosis, including a dexamethasone–corticotropin-releasing hormone test (Answer D), are required.

Rationale Continued: After the clinical and biochemical diagnosis of Cushing syndrome has been established, the initial differential diagnostic test should be measurement of basal plasma ACTH. This patient’s plasma ACTH elevation clearly suggests an ACTH-secreting neoplasm as the cause of her cortisol excess. The imaging studies in this woman could certainly cause diagnostic confusion. Because ACTH-dependent Cushing syndrome is expected, a pituitary MRI is the initial imaging study of choice. In light of her ACTH elevation, the low-attenuation adrenal nodules most likely represent ACTH-dependent nodular adrenal hyperplasia. Chronic ACTH stimulation causes nodular adrenocortical hyperplasia in at least 15% to 20% of patients. Since pheochromocytoma has been associated with ectopic ACTH production, it may seem reasonable to measure plasma free and fractionated metanephrine levels (Answer B) to exclude this possibility. However, this patient has very low-attenuation nodules (<10 Hounsfield units), which would be incompatible with pheochromocytoma. Adrenal nodules are usually associated with ACTH-independent hypercortisolism. Bilateral inferior petrosal sinus sampling for ACTH (Answer C) is always the diagnostic test of choice in patients with ACTH-dependent hypercortisolism and a normal pituitary imaging study. Corticotropin-releasing hormone is administered during this study, and a basal central-to-peripheral ACTH gradient greater than 2 or a post–corticotropin-releasing hormone central-to-peripheral ACTH gradient greater than 3 is consistent with an ACTH-secreting pituitary tumor. Because corticotroph adenomas usually express vasopressin receptors, desmopressin acetate (DDAVP) may be substituted if corticotropin-releasing hormone is not available.

Rationale Continued: Bilateral adrenal venous sampling for cortisol and aldosterone (Answer A) is not needed in patients with ACTH-dependent Cushing syndrome. CT-guided percutaneous biopsy (Answer E) of adrenal nodules is not indicated in patients with low-attenuation nodules. The only indications for CT-guided percutaneous biopsy of an adrenal nodule would be the presence of bilateral adrenal enlargement and primary adrenal insufficiency or in the setting of a known primary malignancy with potential metastasis to the adrenal gland where a clinical diagnosis may alter the treatment protocol or the prognosis. 

Reference(s): Aron DC, Findling JW, Fitzgerald PA, et al. Pituitary ACTH dependency of nodular adrenal hyperplasia in Cushing’s syndrome: report of two cases and review of the literature. Am J Med. 1981;71(2):302-306. PMID: 6266251 Alexandraki KI, Grossman AB. Is urinary cortisol of value in the diagnosis of Cushing’s syndrome? Curr Opin Endocrinol Diabetes Obes. 2011;18(4):259-263. PMID: 21681089

ITE 2019 Question 44 A 61-year-old man undergoes right adrenalectomy, right nephrectomy, and partial hepatectomy for adrenocortical carcinoma. Eight months earlier, he was discovered to have a 16-cm right adrenal mass with evidence of liver and pulmonary metastases. He had no clinical evidence of Cushing syndrome, although no steroid profile was obtained. He was referred to an oncologist. The patient received 6 cycles of adriamycin , cisplatin, and etoposide. Mitotane was also prescribed, and he has taken 3 to 6 g daily since his diagnosis and this has resulted in therapeutic mitotane levels. The adrenal mass regressed to 12 cm, his pulmonary metastases were no longer visible, and his liver metastases were stable. Eight hours after surgery, he develops shock with a blood pressure of 60/30 mm Hg, a pulse rate of 118 beats/min, and a temperature of 104oF (40oC). He is intubated after a bolus of etomidate, 10 mg intravenously, and mechanical ventilation is started. Urine output diminishes to less than 20 mL/h. Aggressive fluid resuscitation with isotonic fluids is initiated along with vasopressors (dopamine and norepinephrine infusion). A rapid ACTH-stimulation test is performed and then hydrocortisone, 100 mg intravenously every 8 hours, is immediately initiated.

Which of the following patterns of laboratory test results would you most likely expect from the rapid ACTH-stimulation test in this patient? Answer Basal ACTH Basal Cortisol +30 min Cortisol A. <5 pg/mL (<1.1 pmol/L)4.3 µg/dL(188.6 nmol/L) 15.0 µg/dL (413.8 nmol/L) B. 765 pg/mL (168.3 pmol/L) 4.2 µg/dL (115.9 nmol/L) 4.3 µg/dL (118.6 nmol/L) C. 365 pg/mL (80.3 pmol/L) 4.2 µg/dL (115.9 nmol/L) 15.0 µg/dL (413.8 nmol/L) D. <5 pg/mL (<1.1 pmol/L) 42.0 µg/dL (1158.7 nmol/L) 45.0 µg/dL (1241.5 nmol/L) E. 23 pg/mL (5.1 pmol/L) 18.0 µg/dL (496.6 nmol/L) 45.0 µg/dL (1241.5 nmol/L) Correct Answer: B. B Learning objective : Predict the pattern of laboratory test results in a patient with mitotane-induced primary adrenal insufficiency.

Rationale: Adrenocortical carcinoma (ACC) is a rare malignancy with incompletely understood pathogenesis and a very poor prognosis. Approximately 60% of patients with ACC present with steroid hormone excess (Cushing syndrome is the most common) or local mass effect. Even in the absence of clinical Cushing syndrome, all patients with ACC should have careful assessment of adrenal function before and during therapy. Cytotoxic chemotherapy in ACC with a combination of etoposide, doxorubicin, cisplatin, and mitotane provides the best outcomes. The response rate is as high as 50%; however, the median progression-free survival is short (only 5 months). Mitotane is the only adrenal-specific agent available for treatment of ACC, and its specific cytotoxic effects produce focal degeneration of the fasciculata and reticularis zones of the adrenal cortex. The efficacy of mitotane in patients with ACC is a matter of controversy with some studies showing clinical benefit and others not demonstrating any positive impact. Adverse effects are common.  Because of its adrenolytic activity, mitotane inevitably induces primary adrenal insufficiency. Mitotane also increases the metabolic clearance of glucocorticoids (by its induction of CYP3A4), increases corticosteroid-binding globulin, and may further reduce the free fraction of cortisol and cause confusion when measuring total cortisol concentrations. All patients receiving mitotane should be expected to develop primary adrenal insufficiency and should receive glucocorticoid support. Most patients on therapeutic mitotane require 40 to 60 mg of hydrocortisone daily in divided doses. Monitoring of ACTH and cortisol levels is important. 

Rationale Continued: Further confounding the adrenal function in this patient was the use of etomidate before mechanical ventilation. Like ketoconazole, etomidate is an imidazole derivative and abruptly inhibits adrenal steroid biosynthesis by inhibiting both 11β-hydroxylase and side-chain cleavage enzymes. Accordingly, this patient had mitotane-induced primary adrenal insufficiency further aggravated by the use of etomidate. Therefore, elevated basal ACTH and absent cortisol response to ACTH (Answer B) is the pattern of laboratory test results would you most likely expect. If the patient had a cortisol-producing adrenocortical tumor (previously untreated with adrenolytic or adrenostatic therapy), its removal may be associated with secondary adrenal insufficiency due to its autonomous cortisol production with suppression of the hypothalamic-pituitary secretion of corticotropin-releasing hormone and ACTH. However, this patient did not have any clinical evidence of preexisting Cushing syndrome and had been treated for many months with an adrenolytic agent (therefore, Answer A is incorrect). Answer C is incorrect because a plasma ACTH concentration of 365 pg /mL (80.3 pmol /L) would provoke a maximum cortisol response and further administration of ACTH would not cause any additional increase in cortisol concentration. Furthermore, the adrenolytic effect of mitotane would limit the adrenocortical secretion of cortisol regardless of the magnitude of ACTH secretion. Answers D and E are incorrect because they represent a normal cortisol secretory response and are not consistent with the clinical picture of adrenal insufficiency. 

Reference(s): Else T, Kim AC, Sabolch A, Raymond VM, Kandthil A, Caoili EM, et al. Adrenocortical carcinoma. Endocr Rev. 2014;35(2):282-326. PMID: 24423978Terzolo M, Fassnacht M, Ciccone G, Allolio B, Berruti A. Adjuvant mitotane for adrenocortical cancer--working through uncertainty. J Clin Endocrinol Metab. 2009;94(6):1879-1880. PMID: 19494162Terzolo M, Baudin AE, Ardito A, Kroiss M, Leboulleux S, Daffara F, et al. Mitotane levels predict the outcome of patients with adrenocortical carcinoma treated adjunctively following radical resection. Eur J Endocrinol. 2013;169(3):263-270. PMID: 23704714

ITE 2019 Question 64 A 68-year-old man is referred to you after a retroperitoneal mass was incidentally identified on abdominal CT performed to investigate intermittent abdominal pain (see image, arrow). He has a 6-year history of poorly controlled hypertension, but no other notable medical history. His current medication regimen consists of hydrochlorothiazide and lisinopril. His brother was recently treated for a kidney tumor.

On physical examination, his height is 72.5 in (184.2 cm) and weight is 202 lb (91.8 kg) (BMI = 27 kg/m2). Fundoscopic examination reveals grade II hypertensive changes only (silver wiring, arteriovenous nicking). The rest of the examination findings are normal. Urinary catecholamine fractionation: Dopamine = 480 µg/24 h (<700 µg/24 h) (SI: 3132 nmol/d [<4567 nmol/d]) Epinephrine = 25 µg/24 h (<35 µg/24 h) (SI: 136 nmol/d [<191 nmol/d]) Norepinephrine = 568 µg/24 h (<170 µg/24 h) (SI: 3359 nmol/d [<1005 nmol/d]) Laboratory test results:

Which of the following genes should be tested first to search for a pathogenic variant? VHL (von Hippel–Lindau tumor suppressor) SDHB (succinate dehydrogenase complex, subunit B) SDHC (succinate dehydrogenase complex, subunit C) SDHD (succinate dehydrogenase complex, subunit D) Genetic testing is not indicated Correct Answer: B Learning objective:Determine the optimal sequence of genetic testing in paraganglioma/pheochromocytoma on the basis of clinical, imaging, and biochemical findings at presentation.

Rationale: This vignette describes an abdominal paraganglioma that is secreting excess norepinephrine (as is the case with approximately 50% of abdominal paragangliomas). At least one-third of all patients with paraganglioma have a disease-causing pathogenic variant. Thus, while it was previously common practice to restrict genetic testing to patients presenting with pheochromocytoma or paraganglioma at a young age ( ie , younger than 50 years), the Endocrine Society now recommends that genetic testing be offered to all patients with paraganglioma (therefore, Answer E is incorrect). Since 1990, 14 different pheochromocytoma and paraganglioma susceptibility genes have been identified. Most cases of familial paraganglioma are caused by mutations in the succinate dehydrogenase (SDH; succinate:ubiquinone oxidoreductase) subunit genes (SDHB, SDHC, SDHD, SDHAF2, SDHA), which compose portions of mitochondrial complex II. However, paraganglioma has also been reported in the setting of von Hippel–Lindau syndrome, and pathogenic variants in the SDHAF2 gene, TMEM127 gene (transmembrane protein 127), and MAX gene ( Myc-associated factor X) have recently been described in familial paraganglioma. Such pathogenic variants are rare and the mechanism behind tumor formation in these circumstances remains unclear.

Rationale Continued: Genotyping of the main pheochromocytoma/paraganglioma susceptibility genes has been reported in a series of studies that include 3000 patients. Germline pathogenic variants were documented in 33.8% and were most common in SDHB (10.30%), SDHD (8.09%), VHL (6.80%), RET (5.00%), and NF1 (2.40%). Pathogenic variants in SDHC, SDHA, MAX, and TMEM127 collectively occur in less than 2% of such cases. With this in mind, and given the costs of genetic testing, it is recommended that a stepwise approach be applied to genetic testing on the basis of clinical and biochemical features at presentation. The current algorithm recommended by the Endocrine Society is outlined in the Figure. The gene most likely to harbor a pathogenic variant in this man with an extra-adrenal, norepinephrine-producing tumor is SDHB (Answer B). In addition, the family history of possible renal cancer further supports this answer because SDHB pathogenic variants are associated with renal carcinoma. Although SDHD testing (Answer D) should be performed if no germline pathogenic variants are found in the SDHB gene, this should not be the initial gene of choice. Pathogenic variants in SDHD are most commonly associated with head and neck paragangliomas, which are usually (in 95% of cases) nonfunctional. Pathogenic variants in SDHC (Answer C) are very rare in paraganglioma and this gene should only be screened if no pathogenic variants are found in SDHB or SDHD. The absence of any other clinical features of von Hippel–Lindau syndrome, as well as the lower frequency of VHL pathogenic variants (Answer A) in hereditary paraganglioma, make this a less likely genetic cause of paraganglioma.

Rationale Continued: Figure. Algorithm outlining optimal sequence of genetic testing in patients with pheochromocytoma or paraganglioma. Adapted from Lenders JW, Duh QY, Eisenhofer G, et al; Endocrine Society. 2014 Pheochromocytoma and paraganglioma: an Endocrine Society clinical practice guideline. J Clin Endocrinol Metab . 2014;99(6):1915-1942.

Reference(s): Galan SR, Kann PH. Genetics and molecular pathogenesis of pheochromocytoma and paraganglioma. Clin Endocrinol ( Oxf ) . 2013;78(2):165-175. PMID: 23061808Lenders JW, Duh QY, Eisenhofer G, et al; Endocrine Society. 2014 Pheochromocytoma and paraganglioma: an Endocrine Society clinical practice guideline. J Clin Endocrinol Metab. 2014;99(6):1915-1942. PMID: 24893135

ITE 2019 Question 69 A 22-year-old woman is sent for evaluation of irregular menses and hirsutism. She reports that she developed pubic hair and body odor at age 4 years and was diagnosed with “premature adrenarche,” but no testing was performed. She was tall relative to her peers until sixth grade, when she stopped growing. She had menarche at age 10.5 years, and she developed acne, facial hair, and menstrual irregularities starting at age 14 years. Her last menstrual period was 3 months ago. She has a brother who was also a tall child but stopped growing in the sixth grade and a sister who had normal growth and development and now has regular monthly menses. On physical examination, her blood pressure is 115/70 mm Hg and BMI is 23 kg/m2. She has coarse terminal hair stubble on her chin, upper lip, and sides of her face and acne on her cheeks and forehead. She has no moon facies, dermal atrophy, myopathy, striae, or acanthosis nigricans. Pelvic examination findings are normal, including typical external genitalia. Laboratory test results: Serum testosterone = 60 ng/dL (8-60 ng/dL) (SI: 2.1 nmol/L [0.3-2.1 nmol/L]) Serum DHEA-S = 545 µg/dL (44-332 µg/dL) (SI: 14.8 µmol/L [1.19-9.00 µmol/L]) Serum 17-hydroxyprogesterone = 300 ng/dL (<80 ng/dL [follicular]; <285 ng/dL [luteal]; <51 ng/dL [postmenopausal]) (SI: 9.1 nmol/L [<2.42 nmol/L (follicular)]; [<8.64 nmol/L (luteal)]; [<1.55 nmol/L (postmenopausal)])

Which of the following is the most appropriate next step in this patient’s evaluation? ACTH-stimulation test measuring 17-hydroxyprogesterone and cortisol ACTH-stimulation test measuring 17-hydroxypregnenolone and DHEA Adrenal-directed CT Plasma ACTH measurement No further testing Correct Answer: A Learning objective:Guide the biochemical evaluation of adrenal androgen excess.

Rationale: If cortisol production is normal and the history indicates androgen excess in childhood, then nonclassic 21-hydroxylase deficiency (21-OHD) should be considered. (Note that “premature adrenarche” is a descriptive term that requires the exclusion of 21-OHD, which was not previously done.) A morning 17-hydroxyprogesterone concentration less than 200 ng/dL (<6.1 nmol/L) excludes nonclassic 21-OHD, and a value greater than 1000 ng/dL (>30.3 nmol/L) establishes the diagnosis. Serum 17-hydroxyprogesterone varies with time of day and across the menstrual cycle. Given the suspicious history and an equivocal random value of 300 ng/dL (9.1 nmol/L), a formal ACTH-stimulation test for 17-hydroxyprogesterone and cortisol (Answer A) is warranted. Adrenal-directed CT (Answer C) is recommended if the testosterone concentration is markedly elevated (>150 ng/dL [>5.2 nmol/L]). Nonclassic 3β-hydroxysteroid dehydrogenase/isomerase deficiency is exceedingly rare and is only considered in unusual cases after nonclassic 21-OHD has been excluded. Furthermore, the best parameter for this diagnosis is the 17-hydroxypregnenolone-to-cortisol ratio, which must be measured together (thus, Answer B is incorrect). Plasma ACTH measurement (Answer D) will not aid in this patient’s diagnosis. In a patient with childhood-onset androgen excess sufficient to advance bone age, a diagnosis should be pursued. Thus, no further testing (Answer E) is incorrect.

Reference(s): Auchus RJ. The classic and nonclassic congenital adrenal hyperplasias . Endocr Pract. 2015;21(4):383-389. PMID: 25536973Witchel SF. Nonclassic congenital adrenal hyperplasia. Curr Opin Endocrinol Diabetes Obes. 2012;19(3):151-158. PMID: 22499220Carbunaru G, Prasad P, Scoccia B, et al. The hormonal phenotype of nonclassic 3beta-hydroxysteroid dehydrogenase (HSD3B) deficiency in hyperandrogenic females is associated with insulin-resistant polycystic ovary syndrome and is not a variant of inherited HSD3B2 deficiency. J Clin Endocrinol Metab. 2004;89(2):783-794. PMID: 14764797

ITE 2019 Question 82 A 54-year-old woman with a history of HIV infection and asthma presents with polyuria, polydipsia, and a blood glucose value of 317 mg/dL (17.6 mmol/L). She has no history of hyperglycemia or diabetes mellitus. HIV was diagnosed 20 years ago and is well controlled, with a recent CD4 cell count greater than 500 and an undetectable viral load. Her medication regimen includes darunavir (protease inhibitor), ritonavir (protease inhibitor), etravirine (nonnucleoside reverse transcriptase inhibitor), emtricitabine (nucleoside reverse transcriptase inhibitor), inhaled fluticasone (250 mcg twice daily), salmeterol (50 mcg twice daily), atorvastatin, and lisinopril. On physical examination, she has several features of Cushing syndrome, including moon facies, dorsocervical fat pad, central adiposity, lipoatrophy of the arms and legs, and violaceous striae.

Her primary care physician had measured morning cortisol, which returned surprisingly low at 1.7 µg/dL (46.9 nmol/L) and prompted further testing (ACTH-stimulation test): Measurement 8 AM →→→→→ 9 AM Cortisol 1.3 µg/dL (SI: 35.7 nmol/L)Administration of intravenous cosyntropin , 250 mcg                   8.4 µg/dL (SI: 231.7 nmol/L)     ACTH <5 pg/mL (SI: 1.1 pmol/L) … Aldosterone 4.3 ng/dL (SI: 119.3 pmol/L)     8.9 ng/dL (SI: 246.9 pmol/L) Plasma renin activity    1.1 ng/mL per h …

Which of the following is the most likely diagnosis? Primary adrenal insufficiency Fluticasone-induced secondary adrenal insufficiency Cortisol-producing adrenal adenoma Ectopic ACTH secretion and Cushing syndrome ACTH-secreting pituitary adenoma Correct Answer: B Learning objective:Diagnose secondary adrenal insufficiency in a patient taking ritonavir (a protease inhibitor that inhibits CYP3A4 activity) and glucocorticoids.

Rationale: This patient presents with the clinical features of Cushing syndrome; however, her biochemical evaluation reveals a low morning cortisol level with an inappropriately low ACTH, as well as a suboptimal cortisol response to cosyntropin . Collectively, this patient has clinical Cushing syndrome combined with chronic secondary adrenal insufficiency (secondary adrenal insufficiency that has resulted in prolonged ACTH suppression, consequent atrophy of the zona fasciculata, and suboptimal cortisol stimulation to cosyntropin ). The main culprit is the combination of ritonavir and fluticasone. Ritonavir is a unique protease inhibitor because it is a potent inhibitor of P450 CYP3A4. CYP3A4 is responsible for the hepatic metabolism of many medications, including glucocorticoids; therefore, ritonavir inhibits the metabolism of glucocorticoids and results in a potentiation of exogenous glucocorticoids. In fact, ritonavir is most commonly used to “boost” the effect of other protease inhibitors in HIV therapy. Since some protease inhibitors cause gastrointestinal adverse effects that limit their tolerability, administering them in smaller and less toxic doses in combination with ritonavir permits tolerability while maintaining adequate blood levels due to CYP3A4 inhibition. However, an undesired consequence of ritonavir use is that exogenous glucocorticoids, including inhaled, topical, and intraarticular glucocorticoids that have small systemic absorption, are potentiated. This patient’s combination of fluticasone (Answer B) and ritonavir most likely resulted in high levels of systemic fluticasone circulation, resultant Cushing syndrome, and hypothalamic-pituitary-adrenal axis suppression. Fluticasone is a synthetic glucocorticoid and is not measured by the cortisol assay.

Rationale Continued: This patient’s management is complex. While the fluticasone is most likely being potentiated at supratherapeutic levels and inducing Cushing syndrome and adrenal insufficiency, it is also treating adrenal insufficiency. Cessation of the medication could result in an adrenal crisis. Replacing fluticasone with an oral glucocorticoid taper is also challenging because the concomitant use of ritonavir will decrease glucocorticoid metabolism and result in unpredictable pharmacokinetics. A multidisciplinary discussion is recommended for these challenging cases. This patient’s antiretroviral regimen was changed to avoid the use of ritonavir, her fluticasone was stopped, and she was treated with a gradual low-dosage prednisone taper until her hypothalamic-pituitary-adrenal axis normalized. There is no evidence of primary adrenal insufficiency (Answer A), which would be characterized by very high ACTH and renin activity levels. There is no evidence of ectopic ACTH syndrome (Answer D), an ACTH-secreting pituitary adenoma (Answer E), both of which would be characterized by a high cortisol level combined with an inappropriately high ACTH level. A cortisol-producing adrenal adenoma (Answer C) would be associated with a low ACTH, but there is no evidence of endogenous hypercortisolism here.

Reference(s): Bornstein SR, Allolio B, Arlt W, et al. Diagnosis and treatment of primary adrenal insufficiency: an Endocrine Society clinical practice guideline. J Clin Endocrinol Metab. 2016;101(2):364-389. PMID: 26760044

ITE 2019 Question 87 A 23-year-old woman is referred for evaluation of weight gain and difficulty sleeping. She has just graduated from college and is having trouble finding a job. She reports a 15.5-lb (7-kg) weight gain over the past 3 months even though she thinks her caloric intake has been low and physical activity has been high. Her current weight is 145 lb (66 kg). At night, she has had difficulty falling asleep and staying asleep. She takes norgestimate -ethinyl estradiol daily for contraception. On physical examination, her blood pressure is 131/92 mm Hg. She has a small dorsocervical fat pad, but no obvious moon facies, supraclavicular fat pads, striae, lipoatrophy, or lipodystrophy. Her primary care physician had performed an evaluation for Cushing syndrome (testing performed at 8 AM following 1-mg of dexamethasone taken at 11 PM the night before):Cortisol = 30 µg/dL (SI: 827.6 nmol/L)ACTH = 45 pg/mL (10-65 pg/mL) (SI: 9.9 pmol/L [2.2-14.3 pmol/L])On the basis of these lab values, her primary care physician performed both pituitary and adrenal imaging. Pituitary MRI showed a possible 5-mm pituitary adenoma; however, the finding was small enough that confidence for an abnormality was low. Abdominal CT was also performed and demonstrated a 1.1-cm left adrenal adenoma.

Which of the following is the most appropriate next step? Inferior petrosal sinus sampling Transsphenoidal pituitary adenoma resection Laparoscopic left-sided adrenalectomy Late-night salivary cortisol testing CT imaging of the chest Correct Answer: D Learning objective:Evaluate for Cushing syndrome with late-night salivary cortisol measurement and recognize when dexamethasone suppression testing may not be accurate or reliable.

Rationale: This patient presents with weight gain and difficulty sleeping, but she does not have overt signs of Cushing syndrome on physical examination. The first step in evaluating Cushing syndrome is to convincingly confirm hypercortisolism. Once hypercortisolism is documented, subsequent steps involve determining whether the hypercortisolism is ACTH-dependent or ACTH-independent and localizing the source of the problem. The most commonly recommended methods to confirm hypercortisolism include late-night salivary cortisol testing, 24-hour urinary free cortisol testing, and 1-mg dexamethasone suppression testing. This patient’s 1-mg dexamethasone suppression test shows a relatively high serum cortisol level with unsuppressed ACTH. One interpretation of these results could be that this is ACTH-dependent hypercortisolism. However, caution should be taken in interpreting dexamethasone suppression tests in patients using estrogen-containing oral contraceptives. Estrogens can increase hepatic globulin synthesis, including that of cortisol-binding globulin. Higher cortisol-binding globulin levels result in higher total serum cortisol levels, and, therefore, a substantial proportion of these patients have a false-positive result on dexamethasone suppression testing. Further, without a dexamethasone level at the time of the morning blood testing, it may not be safe to assume that the testing conditions were appropriate.

Rationale Continued: The best options to test for hypercortisolism in this patient would be (1) late-night salivary cortisol testing (Answer D) (preferably on 2 separate occasions); (2) 24-hour urinary free cortisol testing (preferably on 2 separate occasions); or (3) withdrawal of oral contraception for 6 to 8 weeks followed by a 1-mg dexamethasone suppression test. All the other choices involve a localizing method (Answers A and E) or treatment intervention (Answers B and C) that is not indicated since evidence for hypercortisolism has not yet been convincingly established. This patient elected to stop her oral contraception, and 8 weeks later results from a 1-mg dexamethasone suppression test were normal. Her symptoms were ascribed to other factors and gradually resolved without intervention.

Reference(s): Nieman LK, Biller BM, Findling JW, et al. The diagnosis of Cushing's syndrome: an Endocrine Society clinical practice guideline. J Clin Endocrinol Metab . 2008;93(5):1526-1540. PMID: 18334580Loriaux DL. Diagnosis and differential diagnosis of Cushing’s syndrome. N Engl J Med. 2017;377(2):e3. PMID: 28700850

BONE / CALCIUM

ITE 2019 Question 2 A 78-year-old homeless woman is admitted to the hospital with diffuse bony pain and a pathologic, atraumatic fracture of her left humerus . Radiographs reveal widespread lytic and mixed (lytic/sclerotic) lesions throughout the appendicular and axial skeleton.On physical examination, a large breast mass is palpated, and needle biopsy confirms adenocarcinoma of the breast. Laboratory test results: Serum calcium = 12.3 mg/dL (8.2-10.2 mg/dL) (SI: 3.08 mmol/L [2.1-2.6 mmol/L])Albumin = 2.8 g/dL (3.5-5.0 μg/dL) (SI: 28 μg/L [35-50 μg/L])Creatinine = 6.2 mg/dL (0.6-1.1 mg/dL) (SI: 548 µmol/L [53.0-97.2 μmol/L])Intact PTH = <10 pg/mL (10-65 pg/mL) (SI: <10 ng/L [10-65 ng/L])Alkaline phosphatase = 320 U/L (50-120 U/L) (SI: 5.3 µkat/L [0.84-2.00 µkat/L])After 24 hours of vigorous intravenous saline hydration, the following laboratory results are documented:Serum calcium = 11.0 mg/dL (SI: 2.75 mmol/L)Creatinine = 4.2 mg/dL (SI: 371.3 µmol/L) The oncology team decides to treat her with a subcutaneous injection of denosumab, 120 mg.

In this patient, which of the following is the most likely adverse effect of this therapy? Worsening renal function Osteonecrosis of the jaw A severe flulike syndrome Impaired fracture healing Severe hypocalcemia Correct Answer: E Learning objective:Anticipate the risk of symptomatic hypocalcemia following denosumab therapy in the setting of renal failure.

Rationale: Denosumab, a human monoclonal antibody against RANK ligand, is approved for treatment of osteoporosis, as well as metastatic solid tumors such as breast cancer. It is a very potent inhibitor of osteoclastic bone resorption. Unlike bisphosphonates, denosumab is not cleared via the kidneys and has no effect on renal function (Answer A). Although cases of osteonecrosis of the jaw (Answer B) have been reported with both bisphosphonates and denosumab, this is very unlikely after only 1 dose of the drug. Severe flu-like syndromes (Answer C) (also known as “acute phase reactions”) are much more common with intravenous bisphosphonates than with denosumab. Potent antiresorptive agents such as denosumab have not been shown to impair fracture healing (Answer D). The major adverse effect to be expected in this setting is profound, symptomatic hypocalcemia (Answer E) within 7 to 10 days following denosumab administration. Patients at greatest risk for this complication are those with significantly impaired renal function and high bone turnover with elevated alkaline phosphatase, as demonstrated in this patient. The potent inhibition of osteoclastic bone resorption by denosumab stops the efflux of calcium from bone and can lower serum calcium dramatically. The usual physiologic responses to hypocalcemia, such as a rise in PTH and calcitriol production, that would normally lead to increased renal tubular reabsorption of calcium and increased gut absorption of calcium, will not work in the setting of renal failure.

Reference(s): Dave V, Chiang CY, Booth J, Mount PF. Hypocalcemia post denosumab in patients with chronic kidney disease stage 4-5. Am J Nephrol . 2015;41(2):129-137. PMID: 25790847 Body JJ, Bone HG, de Boer RH, et al. Hypocalcaemia in patients with metastatic bone disease treated with denosumab. Eur J Cancer. 2015;51(13):1812-1821. PMID: 26093811Kinoshita Y, Arai M, Ito N, et al. High serum ALP level is associated with increased risk of denosumab-related hypocalcemia in patients with bone metastases from solid tumors. Endocr J. 2016;63(5):479-484. PMID: 26860123Stopeck AT, Lipton A, Body JJ, et al. Denosumab compared with zoledronic acid for the treatment of bone metastases in patients with advanced breast cancer: a randomized, double-blind study. J Clin Oncol. 2010;28(35):5132-5139. PMID: 21060033

ITE 2019 Question 16 A 52-year-old woman is referred for evaluation after a DXA scan shows osteoporosis at the hip, with right femoral neck and total hip T-scores of –3.5 and –3.0, respectively. The lumbar spine T-score is –1.6. She has a history of recurrent metatarsal stress fractures without significant trauma. She underwent natural menopause 1 year ago and has not taken hormone therapy. She takes calcium, 600 mg twice daily, and cholecalciferol, 2000 IU daily, but no multivitamin or additional supplements. Review of systems is notable for multiple previous tooth extractions due to poor dentition and ongoing mild but vague right upper thigh pain. She reports her mother had a femoral fracture in her 50s. On physical examination, she has no significant height loss. She has multiple missing teeth. She has mild tenderness to palpation over the right anterolateral upper femur. There is no evident thoracic kyphosis.

Laboratory test results: Serum calcium = 9.5 mg/dL (8.2-10.2 mg/dL) (SI: 2.4 mmol/L [2.1-2.6 mmol/L]) Serum phosphate = 4.0 mg/dL (2.3-4.7 mg/dL) (SI: 1.3 mmol/L [0.7-1.5 mmol/L]) Serum albumin = 4.0 mg/dL (3.5-5.0 g/dL) (SI: 40 g/L [35-50 g/L]) Serum creatinine = 1.0 mg/dL (0.6-1.1 mg/dL) (SI: 88.4 µmol/L [53.0-97.2 µmol/L]) Serum intact PTH = 45 pg /mL (10-65 pg/mL) (SI: 45 ng/L [10-65 ng/L])Serum 25-hydroxyvitamin D = 48 ng/mL (25-80 ng/mL) (SI: 119.8 nmol/L [62.4-199.7 nmol/L])Serum alkaline phosphatase = 30 U/L (50-120 U/L) (SI: 0.5 µkat/L [0.84-2.00 µkat/L])Serum liver transaminases normalUrinary calcium = 200 mg/24 h (100-300 mg/24 h) (SI: 5.0 mmol/d [2.5-7.5 mmol/d])Urinary creatinine = 1.2 g/24 h (1.0-2.0 g/24 h) (SI: 10.6 mmol/d [8.8-17.7 mmol/d])

Which of the following is this patient’s most likely diagnosis? Osteogenesis imperfecta type 1 Paget disease of bone Hypophosphatasia X-linked hypophosphatemic rickets OsteoporosisCorrect Answer: CLearning objective: Identify the clinical signs and laboratory findings that are characteristic of hypophosphatasia.

Rationale: While it is tempting for the busy clinician to approach all patients who present with low bone mineral density with or without fragility fractures as representing some form of osteoporosis, it is critical to recognize that there are a number of metabolic bone disorders in which skeletal fragility is due to undermineralization of bone ( eg , osteomalacia ). In order to make a diagnosis of osteoporosis (Answer E), these alternate disorders must first be excluded.  This patient presents with many of the hallmark signs of hypophosphatasia (Answer C), an osteomalacic disorder that is due to pathogenic variants in the tissue nonspecific alkaline phosphatase gene (ALPL). ALPL pathogenic variants result in defective catabolism of pyrophosphate molecules, which are natural inhibitors of mineralization within the bone matrix. Abnormal dental mineralization with excessive primary tooth loss is also common. The disease has a variable presentation that confers disproportionate morbidity and mortality in neonates and infants due to respiratory failure (rib hypomineralization), whereas older children, and particularly adults, have a milder skeletal phenotype. In fact, many adults with pathogenic variants in ALPL are asymptomatic. Affected adults with hypophosphatasia most commonly present with stress fractures of the lower extremities (metatarsal, proximal femur). This patient’s history of right proximal femur pain and multiple previous metatarsal fractures, family history of femoral fracture in her mother, and frankly low alkaline phosphatase level are highly suggestive of hypophosphatasia.

Rationale Continued: The low bone density that is disparately lower in the lower extremity most likely represents hypomineralization instead of low bone density, particularly given her relatively young age and expected regional differences in bone mineral density decline at specific skeletal sites with aging (typically later declines in bone mineral density in the proximal femur compared with the spine). At present, there is no specific US FDA-approved therapy for adults with hypophosphatasia, although a recombinant form of alkaline phosphatase ( asfotase alfa) is approved for treatment of hypophosphatasia of infantile and childhood onset. At least intuitively, bisphosphonates should be avoided because they may inhibit mineralization and exacerbate an underlying osteomalacic disorder. Indeed, previous reports have identified patients with hypophosphatasia who present with a femoral stress fracture that mimics atypical femoral fractures seen rarely in patients on long-term bisphosphonate therapy.

Rationale Continued: Although patients with type 1 osteogenesis imperfecta (Answer A) have increased skeletal fragility, the more diffuse pattern of early-life fractures, normal or higher alkaline phosphatase levels (reflective of higher skeletal turnover), and absence of reported physical examination characteristics (blue sclerae) make this diagnosis unlikely. Paget disease of bone (Answer B) could be associated with bone pain, although it is characteristically associated with elevated alkaline phosphatase levels. Patients with X-linked hypophosphatemic rickets (Answer D), which is due to pathogenic variants in the PHEX gene, have disordered metabolism of phosphate (low) and 1,25-dihydroxyvitamin D (low) due to excessive effect of the circulating phosphaturic factor FGF-23. While patients with this osteomalacia disorder may develop stress fractures, as has this patient, the frankly normal phosphate level rules out this condition.

Reference(s): Mornet E. Hypophosphatasia. Metabolism . 2017;82:142-155. PMID: 28939177 Sutton RA, Mumm S, Coburn SP, Ericson KL, Whyte MP. "Atypical femoral fractures" during bisphosphonate exposure in adult hypophosphatasia. J Bone Miner Res. 2012;27(5):987-994. PMID: 22322541

ITE 2019 Question 21 A 55-year-old woman is referred for evaluation of multiple fractures. She entered menopause 3 years ago and has not taken hormone therapy. During childhood and adolescence, she sustained several long-bone fractures that were attributed to her active lifestyle and participation in sports, but she had no additional fractures until recently. Since menopause, she has sustained fractures at the wrist, humerus , and femur in low-trauma falls. Recent DXA reveals T-scores of –3.0 at the spine, –2.8 at the femoral neck, and –2.7 at the total hip. Her mother was diagnosed with osteoporosis at age 65 years. On physical examination, she is a well-appearing woman with no dysmorphic features. Her height is 65 in (165.1 cm). Sclerae appear slightly bluish. She has no joint deformities or laxity. Her dentition appears normal. She wears bilateral hearing aids. Laboratory test results are normal for the following: complete blood cell count, comprehensive chemistry panel (including liver function and alkaline phosphatase), thyroid function, 25-hydroxyvitamin D, 1,25-dihydroxyvitamin D, and intact PTH.

Which of the following tests will establish the diagnosis? Bone-specific alkaline phosphatase measurement Sequencing of type 1 collagen α 1 and 2 genes (COL1A1/COL1A2) Sequencing of the chloride channel 7 gene (CLCN7) Sequencing of the vitamin D receptor gene (VDR) Tetracycline-labeled bone biopsy of the iliac crest Correct Answer: B Learning objective:Diagnose osteogenesis imperfecta type 1 (the mildest form).

Rationale: This patient has the mildest form of osteogenesis imperfecta, known as type 1. Inheritance is autosomal dominant, but many pathogenic variants can occur de novo, so the family history may be negative. Patients with osteogenesis imperfecta type 1 have normal stature and little or no skeletal deformity. Fractures occur in childhood or adolescence and decrease markedly after puberty. As is the case in this vignette, affected patients may then present in middle age with “osteoporosis.” In 50% of affected patients, there is early-onset hearing loss before age 40 years. On physical examination, there may be blue sclerae and easy bruising. Joint laxity can be present, but dentinogenesis imperfecta is usually absent. Diagnosis is made by sequencing the genes that encode type 1 collagen (α1 and α2) (COL1A1/COL1A2) (Answer B). The main therapy for osteogenesis imperfecta remains bisphosphonates (intravenous pamidronate and zoledronic acid and oral bisphosphonates). Denosumab has been used in rare case reports. Teriparatide does not dramatically change clinical outcomes.

Rationale Continued: Low bone-specific alkaline phosphatase (Answer A) is characteristic of hypophosphatasia, a heritable form of osteomalacia that is characterized by subnormal activity of the tissue-nonspecific (bone/liver/kidney) isoenzyme of alkaline phosphatase. Given the normal alkaline phosphatase level in this patient and her other features, this diagnosis is unlikely. Pathogenic variants in the chloride channel 7 gene (Answer C) are found in osteopetrosis, a rare sclerosing bone disorder associated with high bone mass, hearing loss, and fractures due to diminished osteoclast-mediated skeletal resorption. Pathogenic variants in the vitamin D receptor gene (Answer D) can be found in patients with vitamin D–resistant rickets, but this would be accompanied by a high 1,25-dihydroxyvitamin D level, as well as hypophosphatemia, hypocalcemia, and osteomalacia . A tetracycline-labeled bone biopsy (Answer E) would not be useful in diagnosing this patient with osteogenesis imperfecta, but skin biopsies to examine elastin and collagen can be diagnostic.

Reference(s): Van Dijk FS, Sillence DO. Osteogenesis imperfecta: clinical diagnosis, nomenclature and severity assessment [published correction appears in Am J Med Genet A . 2015;167A(5):1178]. Am J Med Genet A . 2014;164A(6):1470-1481. PMID: 24715559

ITE 2019 Question 27 A 53-year-old man presents for evaluation after having a kidney stone. Stone analysis reveals calcium oxalate. He has normal serum calcium and PTH levels, with a 24-hour urinary calcium excretion of 335 mg/24 h (100-300 mg/24 h) (SI: 8.4 mmol/d [2.5-7.5 mmol/d]), but normal 24-hour urinary oxalate, uric acid, sodium, and citrate levels. His urine volume is 1650 mL/24 h.

Which of the following recommendations would provide the greatest reduction in his risk of future calcium oxalate stone disease? Increased fluid intake Reduced dietary sodium Reduced dietary oxalate Potassium citrate Hydrochlorothiazide Correct Answer: A Learning objective : Recommend increased fluid intake as a means to reduce the risk of a second kidney stone.

Rationale: According to a comprehensive meta-analysis in patients who had a single kidney stone, increased fluid intake (Answer A) was the one intervention that was clearly shown to reduce recurrent stone disease. Of interest, reducing phosphate-containing soft drink consumption (not an answer option above) was moderately helpful. In patients with multiple stone episodes—most of whom had already increased their fluid intake—thiazide diuretics (Answer E) and citrate supplements (Answer D) were similarly effective in patients with hypercalciuria, as well as in unselected patients. Hydrochlorothiazide acts to enhance renal calcium reabsorption to reduce urinary calcium excretion and might be an additional measure to consider if he has more stones. Hypercalciuria may be caused by increased sodium intake, which leads to increased sodium excretion and an obligatory loss of calcium in the urine; however, this patient’s normal urinary sodium excretion indicates that is not the case here, so recommending reduced dietary sodium (Answer B) is incorrect. His urinary oxalate level is not elevated, so there would be no benefit in reducing his dietary intake of oxalate (Answer C).

Reference(s): Fink HA, Wilt TJ, Eldman KE, et al. Medical management to prevent recurrent nephrolithiasis in adults: a systematic review for an American College of Physicians Clinical Guideline [published correction appears in Ann Intern Med. 2013;159(3):230-232]. Ann Intern Med . 2013;158(7):535-543. PMID: 23546565Vigen R, Weideman RA, Reilly RF. Thiazides diuretics in the treatment of nephrolithiasis: are we using them in an evidence-based fashion? Int Urol Nephrol. 2011;43(3):813-819. PMID: 20737209Borghi L, Schianchi T, Meschi T, et al. Comparison of two diets for the prevention of recurrent stones in idiopathic hypercalciuria. N Engl J Med. 2002;346(2):77-84. PMID: 11784873

ITE 2019 Question 33 A 62-year-old woman with osteoporosis seeks evaluation for left leg pain. She has taken alendronate for the past 7 years; in addition, she took hormone therapy from age 52 to 60 years for menopausal symptoms. She has been adherent to her calcium and vitamin D regimen. Until 3 months ago when she developed left leg pain, she had been walking 20 minutes 3 times weekly. There had been no change in her exercise pattern or frequency before the pain started. At that time, her primary care physician gave her crutches and prescribed no weight-bearing for 3 months. Despite this rest period, her left leg continues to hurt. On physical examination, her BMI is 27 kg/m2. When she walks, she has an antalgic gait. She has no pain in the contralateral thigh.

DXA documents T-scores of –2.0 at the lumbar spine and –1.7 at the right femoral neck. Bone mineral density at the spine and mean total hip sites are both statistically unchanged from a DXA performed 2 years earlier. Plain film of the left femur is shown (see image).

Laboratory test results: Urinary N-telopeptide = 31 nmol BCE/mmol creat (26-124 nmol BCE/mmol creat [postmenopausal]) Serum creatinine = 0.73 mg/dL (0.6-1.1 mg/dL) (SI: 64.5 µmol/L [53.0-97.2 μmol/L])Calcium = 10 mg/dL (8.2-10.2 mg/dL) (SI: 2.5 mmol/L [2.1-2.6 mmol/L])Alkaline phosphatase = 105 U/L (50-120 U/L) (SI: 1.75 µkat/L [0.84-2.00 µkat/L])Magnesium = 2.0 mg/dL (1.5-2.3 mg/dL) (SI: 0.8 mmol/L [0.6-0.9 mmol/L])Phosphate = 4.3 mg/dL (2.3-4.7 mg/dL) (SI: 1.4 mmol/L [0.7-1.5 mmol/L])25-Hydroxyvitamin D = 25 ng/mL (30-80 ng/mL [optimal]) (62.4 nmol/L [74.9-199.7 nmol/L])PTH = 12 pg/mL (10-65 pg/mL) (SI: 12 ng/L [10-65 ng/L])

Which of the following is the best treatment recommendation for this patient? Discontinue alendronate and refer to orthopedic surgery Discontinue alendronate and start teriparatide Discontinue alendronate and change to an intravenous bisphosphonate Continue alendronate Continue alendronate and restart hormone therapy Correct Answer: A Learning objective:Manage an atypical femur fracture associated with prolonged bisphosphonate use.

Rationale: Atypical femur fractures are an increasingly recognized, albeit rare, adverse event associated with both oral and intravenous forms of bisphosphonates (as well as with denosumab). These fractures are defined as low-energy fractures located in the subtrochanteric region or femoral shaft, which are noncomminuted and present with characteristic prodromal hip or thigh pain. At this time, a direct causal association between these drugs and the development of atypical femur fractures is not clear. Proposed mechanisms of pathogenesis include microcrack accumulation due to suppressed osteoclastic resorption of damaged bone and variation in lower limb geometry. The image in the stem shows a long segment of cortical thickening along the medial and lateral aspect of the proximal subtrochanteric femur. A beaklike configuration on the lateral femur is present with a small linear lucency coursing perpendicular to the cortex. This occurs 4 cm below the lesser trochanter.

Rationale Continued: The lesion seen in this question could progress to a complete fracture. A visual representation of typical and atypical subtrochanteric fractures is shown (see image).

Rationale Continued: To manage these fractures, the American Society for Bone and Mineral Research Task Force recommends stopping antiresorptive medication (thus, Answers C, D, and E are incorrect). For incomplete fractures that present with pain, the task force recommends referring these patients for prophylactic nail reconstruction (thus, Answer A is correct). Conservative therapy is recommended only if there is minimal or no pain. Such therapy includes non–weight-bearing ambulation with a walker or crutches, discontinuation of antiresorptive agents, and reevaluation of radiographic improvement in 2 to 3 months. Numerous case reports describe the use of teriparatide (Answer B) in the setting of atypical femur fractures and report fracture healing without surgical intervention. However, the response appears to be variable as other reports indicate that not all patients with atypical femur fractures show improvement with the use of this drug. Current guidelines set forth by the American Society for Bone and Mineral Research Task Force state that the use of teriparatide to treat such fractures is not conclusively recommended because there are no data from randomized controlled trials.

Reference(s): Shane E, Burr D, Abrahamsen B, et al. Atypical subtrochanteric and diaphyseal femoral fractures: second report of a task force of the American Society for Bone and Mineral Research. J Bone Miner Res. 2014;29(1):1-23. PMID: 23712442 Girgis CM, Sher D, Seibel MJ. Atypical femoral fractures and bisphosphonate use. N Engl J Med. 2010;362(19):1848-1849. PMID: 20463351

ITE 2019 Question 41 A 70-year-old woman is referred for management of osteoporosis. She sustained a vertebral fracture 5 years ago, which prompted DXA imaging that showed osteoporosis in the lumbar spine (T-score –3.0). After an appropriate workup for secondary causes of osteoporosis, which was normal, she was prescribed risedronate, 150 mg monthly, which she did not tolerate because of severe gastroesophageal reflux. She was then switched to denosumab, 60 mg twice yearly. Follow-up DXA studies show improvement in lumbar spine bone mineral density, such that her lumbar spine T-score is now in the osteopenic range (T-score, –1.9). However, she has developed significant erythema with pruritis at the injection site following the last 2 injections. The most recent reaction was worse than the prior one and she returns to discuss the best approach to her osteoporosis management. She has rate-controlled atrial fibrillation but is otherwise healthy. She has no active dental concerns. Denosumab and a β-adrenergic blocker are her only current medications.

Current laboratory test results: Creatinine = 0.8 mg/dL (0.6-1.1 mg/dL) (SI: 70.7 µmol/L [53.0-97.2 µmol/L]) Calcium adjusted for albumin = 9.5 mg/dL (8.2-10.2 mg/dL) (SI: 2.4 mmol/L [2.1-2.6 mmol/L]) 25-Hydroxyvitamin D = 45 ng/mL (30-80 ng/mL [optimal]) (SI: 112.3 nmol/L [74.9-199.7 nmol/L]) The rest of her relevant chemistry and hematology profiles are unremarkable.

Which of the following is the most appropriate recommendation regarding management of her osteoporosis now? Continue denosumab twice yearly Continue denosumab, but decrease the dosing to once yearly Discontinue denosumab, but continue supplemental calcium and vitamin D Discontinue denosumab and start raloxifene Discontinue denosumab and start zoledronic acid once yearly Correct Answer: E Learning objective: In a postmenopausal woman with osteoporosis, determine the need to start zoledronic acid based on the risk-benefit assessment upon discontinuation of denosumab.

Rationale: Denosumab is a human monoclonal antibody to RANK ligand that has been approved by the US FDA since 2009 for the treatment of postmenopausal osteoporosis. It has also received US FDA approval for the prevention of bone loss in women and men who take gonadal deprivation therapies. Randomized controlled trials have confirmed a vertebral and nonvertebral benefit of the drug when given as a 60-mg dose twice yearly for postmenopausal osteoporosis. It is generally well tolerated, and it may be given to patients with impaired renal function (due to nonrenal clearance of the drug). However, in clinical trials, it has been associated with a higher rate of serious infections. In addition, studies have shown that the drug is entirely reversible, in that there is rapid bone mineral density decline within the first year of drug discontinuation that has been associated with multiple vertebral fractures in some patients. Given these observations, discontinuation of denosumab without initiation of pharmacologic therapy (Answer C) is not recommended. This patient cannot tolerate risedronate, which, based on its chemical structure (pyridinyl bisphosphonate vs amino bisphosphonate [ ie , alendronate and ibandronate]), is typically the most well-tolerated oral bisphosphonate from an upper gastrointestinal standpoint. Therefore, an appropriate option for her would be intravenous zoledronic acid, 5 mg annually (Answer E). She has no active dental concerns ( ie , risk for osteonecrosis of the jaw) or renal concerns that would preclude its use. A published small case series of patients from the FREEDOM study of denosumab found that a single dose of intravenous zoledronic acid was not adequate to fully prevent bone loss in the setting of denosumab discontinuation.

Rationale Continued: This patient has experienced a repeated cutaneous reaction to denosumab, which was observed more frequently in the pivotal randomized clinical trial compared with the occurrence in the placebo group. Therefore, the drug should not be continued (Answer A) due to safety concerns. Denosumab must be given every 6 months for therapeutic effect based on the biologic and pharmacokinetic profile of the drug, so decreasing administration to once yearly (Answer B) would also be incorrect. While raloxifene (Answer D)—the alternative but known to be weaker antiresorptive agent—could be considered, the patient’s age and underlying atrial fibrillation substantially increase her risk of stroke. Pertinent to this concern, the RUTH trial (Raloxifene Use in The Heart) demonstrated a higher risk for fatal stroke in women at higher baseline risk ( ie , those with hypertension, diabetes mellitus, atrial fibrillation). Thus, risk-benefit assessment would not favor the use of raloxifene for this patient.

Reference(s): Tsourdi E, Langdahl B, Cohen-Solal M, et al. Discontinuation of denosumab therapy for osteoporosis: a systematic review and position statement by ECTS. Bone . 2017105:11-17. PMID: 28789921Bone HG, Bolognese MA, Yuen CK, et al. Effects of denosumab treatment and discontinuation on bone mineral density and bone turnover markers in postmenopausal women with low bone mass. J Clin Endocrinol Metab. 2011;96(4):972-980. PMID: 21289258Reid IR, Horne AM, Mihov B et al.  Bone Loss After Denosumab:  Only Partial Protection with Zoledronate. Calcif Tissue Int. 2017; 101(4):371-374. PMID: 28500448

ITE 2019 Question 43 A 36-year-old man is referred for evaluation of an abnormal left humerus x-ray. He describes a 2- to 3-year history of constant aching in his left mid-upper arm that has gradually worsened over time. He recalls no antecedent trauma. He has no history of fractures. He describes frequent headaches involving both parietal regions, which are poorly responsive to ibuprofen. He takes no medications or supplements. The rest of his medical history and accompanying family history are unremarkable. Review of systems is notable for an unintentional 10-lb (4.5-kg) weight loss. On physical examination, he has mild, diffuse tenderness over the left mid- humerus without an apparent mass or deformity. There is a hyperpigmented, macular lesion with irregular borders over his upper back and neck that extends from the midline laterally. The thyroid is palpably enlarged and nodular without a dominant lesion. Deep tendon reflexes are brisk with a shortened recovery phase. The rest of his examination findings, including exam of the cranium, are normal. Results from complete blood cell count, creatinine, AST, ALT, total bilirubin, calcium, phosphate, intact PTH, and 25-hydroxyvitamin D are normal. Abnormalities identified on laboratory workup:TSH = 0.03 mIU/L (0.5-5.0 mIU/L)Free T4 = 2.5 ng/dL (0.8-1.8 ng/dL) (SI: 32.2 ng/dL [10.30-23.17 pmol/L])Alkaline phosphatase = 310 U/L (50-120 U/L) (SI: 5.18 µkat/L [0.84-2.00 µkat/L])

Anteroposterior radiograph of the left humerus is shown (see image).

Which of the following is the most likely diagnosis in this patient? Celiac disease Fibrous dysplasia Graves disease Paget disease of bone Tumor-induced osteomalacia Correct Answer: BLearning objective:Diagnose McCune-Albright syndrome and fibrous dysplasia.

Rationale: This patient presents with a painful left humeral lesion of uncertain etiology. The markedly elevated alkaline phosphatase in the presence of otherwise normal liver function studies is consistent with a bone-specific source. While an increase in bone alkaline phosphatase is observed in a number of diseases that affect the skeleton, specific clinical factors in this patient confirm a diagnosis of fibrous dysplasia (Answer B), in this case related to McCune-Albright syndrome. This condition is due to mosaic, somatic activating pathogenic variants in the GNAS gene that encodes the cAMP pathway-associated G-protein, Gsα. Affected patients present with skeletal lesions ( monostotic or polyostotic), pigmented skin macules, and endocrinopathies. The displayed humeral shaft lesion has a “ground-glass” appearance that is characteristic of fibrous dysplasia/McCune-Albright syndrome. The presence of headaches suggests that he may well have skull involvement, which is common in this syndrome. Additionally, his unilateral skin finding is consistent with a café-au-lait lesion that is pathognomonic for the disease. The irregular border of the lesions is commonly referred to as mimicking the coast of Maine, as opposed to smooth-bordered café-au-lait lesions (coast of California) that are seen in neurofibromatosis type 1. Finally, the patient’s weight loss, thyromegaly, and thyroid biochemical findings are consistent with fibrous dysplasia/McCune-Albright syndrome, in which hyperthyroidism is present in approximately one-third of patients. Other common endocrinopathies include precocious puberty in females and macro- orchidism in males, although less common endocrine disorders also occur ( eg , hypercortisolism and GH excess). While there is no specific therapy for this syndrome, parenteral bisphosphonates may decrease bone pain at sites of skeletal involvement. Additionally, regular surveillance for skeletal and endocrine manifestations and targeted management of such disorders is recommended.

Rationale Continued: While Paget disease of bone (Answer D) is a diagnostic consideration for this patient, the specific radiographic appearance of the lesion, young age of onset, and absence of associated endocrinopathies in patients with Paget disease make this very unlikely. Tumor-induced osteomalacia (Answer E), which is most commonly due to benign mesenchymal tumors that overproduce the phosphaturic factor fibroblast growth factor 23, can also present with bone pain and an elevated alkaline phosphatase level. However, this patient’s normal serum phosphate level essentially rules out this disorder. Graves disease (Answer C) can sometimes be associated with an elevated alkaline phosphatase level due to an increase in bone remodeling that is induced by elevated levels of thyroid hormone. However, autoimmune thyroid disease is not associated with localized skeletal lesions.Finally, although celiac disease (Answer A) can present with signs and symptoms of osteomalacia that may mimic those seen in this patient (bone pain and high alkaline phosphatase), it does not cause focal lesions.

Reference(s): Wong SC, Zacharin M. Long-term health outcomes of adults with McCune-Albright syndrome. Clin Endocrinol ( Oxf ). 2017;87(5):627-634. PMID: 28699175Robinson C, Collins MT, Boyce AM. Fibrous dysplasia/McCune-Albright syndrome: clinical and translational perspectives. Curr Osteoporos Rep. 2016;14(5):178-186. PMID: 27492469

ITE 2019 Question 48 A 55-year-old man with progressive multiple myeloma undergoes laboratory testing before initiation of therapy with a novel experimental agent. Other than mild fatigue and achiness, he has no new or worrisome symptoms. Specifically, he has not experienced anorexia, nausea, vomiting, polyuria, polydipsia, or abdominal pain. On neurologic examination, he is alert and oriented without focal findings. Laboratory test results (serum): Calcium = 18.4 mg/dL (8.2-10.2 mg/dL) (SI: 4.6 mmol/L [2.1-2.6 mmol/L]) Albumin = 3.9 g/dL (3.5-5.0 g/dL) (SI: 39 g/L [35-50 g/L]) Phosphate = 4.0 mg/dL (2.3-4.7 mg/dL) (SI: 1.3 mmol/L [0.7-1.5 mmol/L]) Creatinine = 1.0 mg/dL (0.7-1.3 mg/dL) (SI: 88.4 µmol/L [61.9-114.9 µmol/L]) Intact PTH = 34 pg/mL (10-65 pg/mL) (SI: 34 ng/L [10-65 ng/L])

Which of the following should you do now? Refer for emergent parathyroid exploration Measure serum ionized calcium Begin cinacalcet, 30 mg orally 3 times daily Refer for emergent plasmapheresis Administer zoledronic acid, 5 mg intravenously Correct Answer: B Learning objective:Diagnose “pseudohypercalcemia” and recommend measurement of ionized calcium in certain cases of multiple myeloma.

Rationale: Fifty percent of circulating calcium is bound to serum proteins, primarily to albumin. In this patient with multiple myeloma, the serum albumin is normal. However, in some patients with multiple myeloma, high circulating levels of serum immunoglobulins can also bind calcium, leading to “ pseudohypercalcemia .” The key to recognizing this condition is to note the marked discordance between the serum calcium (very high) and the patient’s symptoms (almost none). The only test needed to confirm the diagnosis is measurement of serum ionized calcium (Answer B), which in this case was normal. Emergent parathyroid exploration (Answer A), cinacalcet therapy (Answer C), emergent plasmapheresis (Answer D), and zoledronic acid therapy (Answer E) are reasonable steps in the treatment of true, severe hypercalcemia, but they are not needed in this case.

Reference(s): Jacobs TP, Bilezikian JP. Clinical review: rare causes of hypercalcemia. J Clin Endocrinol Metab . 2005;90(11):6316-6322. PMID: 16131579Schwab JD, Strack MA, Hughes LD, Shaker JL. Pseudohypercalcemia in an elderly patient with multiple myeloma: report of a case and review of literature. Endocr Pract. 1995;1(6):390-392. PMID: 15251564

ITE 2019 Question 61 An 18-year-old man is seen for follow-up of hypoparathyroidism. He initially presented at age 3 years with a seizure and severe hypocalcemia. Since then, he has been maintained on calcium and calcitriol. He has also been treated for intermittent oral candidiasis and fungal infections of his fingernails and toenails since childhood. Otherwise, he feels well. On physical examination, he has some tinea of the nails and negative Chvostek and Trousseau signs. Laboratory test results are normal for the following: complete blood cell count, serum calcium, phosphate, creatinine, liver function, and 25-hydroxyvitamin D.

Which of the following additional tests is indicated now? TPO antibody assessment Antinuclear antibody assessment Glutamic acid decarboxylase (GAD65) antibody assessment 21-Hydroxylase antibody assessment No further testing is indicated Correct Answer: D Learning objective:Diagnose autoimmune polyendocrine syndrome type 1 as a cause of hypoparathyroidism and recommend appropriate screening

Rationale: This patient has autoimmune polyendocrine syndrome type 1 (APS type 1) due to a pathogenic variant in the autoimmune regulator gene (AIRE). APS type 1 is also known by the acronym APECED (autoimmune polyendocrinopathy -candidiasis-ectodermal dystrophy). The classic presentation includes at least 2 of the following 3 major clinical components: chronic mucocutaneous candidiasis, primary hypoparathyroidism, and autoimmune adrenal insufficiency.  Measurement of adrenal autoantibodies such as 21-hydroxylase antibodies (Answer D) can help predict risk of developing adrenal insufficiency in these patients. In APECED, the presence of adrenal autoantibodies has a greater than 90% positive predictive value for development of adrenal insufficiency. Assessment for TPO antibodies (Answer A) is used to diagnose Hashimoto thyroiditis, the most common cause of hypothyroidism, which is not a typical feature of APS type 1. Antinuclear antibody assessment (Answer B) is not necessary, as systemic lupus erythematosus is not typically part of APS type 1. Glutamic acid decarboxylase (GAD65) antibodies (Answer C) are often present in patients with APS type 1, but they have not been shown to predict the development of type 1 diabetes mellitus, which occurs in about 18% of those with APS type 1. Islet-cell antibodies are a better predictor. Because Addison disease can be life threatening, it is important to screen for this even in the absence of symptoms. Thus, pursuing no further testing (Answer E) is incorrect.

Reference(s): Weiler FG, Dias-da-Silva MR, Lazaretti -Castro M. Autoimmune polyendocrine syndrome type 1: case report and review of literature. Arq Bras Endocrinol Metabol. 2012;56(1):54-66. PMID: 22460196Akirav EM, Ruddle NH, Herold KC. The role of AIRE in human autoimmune disease. Nat Rev Endocrinol. 2011;7(1):25-33. PMID: 21102544Eisenbarth GS, Gottlieb PA. Autoimmune polyendocrine syndromes. N Engl J Med. 2004;350(20):2068-2079. PMID: 15141045

ITE 2019 Question 67 A 63-year-old woman returns for management of osteoporosis. She has no fracture history, although her mother had sustained a hip fracture from a ground-level fall at age 80 years. On the patient’s baseline DXA scan performed 2 years ago, she was documented to have low bone mineral density (see table). No secondary causes of low bone mineral density were identified and the patient was prescribed daily teriparatide, which she has taken consistently for the past 2 years. She also takes supplemental calcium, 1000 mg daily, and over-the-counter vitamin D, 1000 IU daily. She exercises regularly and does not smoke cigarettes or drink alcohol. Her most recent DXA scan this year documented a significant improvement in bone mass. Region      Age at the Time of DXA      61 Years 63 Years Lumbar spine T-score    –3.1 –2.1 Femoral neck T-score    –2.4 –2.3

Laboratory test results: Complete blood count, normal Comprehensive chemistry panel, normal Serum osteocalcin  Pretreatment = 31 ng/mL (SI: 31 µg/L)  Posttreatment = 66 ng/mL (SI: 66 µg/L)

In discussing her future management, which of the following sets of results would be most likely 1 year from now if you were to elect no further therapy? Correct Answer: B Learning objective : In a patient with osteoporosis who discontinues an anabolic agent, predict the effects of therapy cessation on bone mineral density and markers of bone turnover.   Answer     Lumbar Spine Bone Mineral Density     Serum Osteocalcin   A. Unchanged Lower B. Lower Lower C. Higher Lower D. Higher Higher E. Lower Unchanged

Rationale: Recombinant human PTH 1-34 (teriparatide) is an anabolic agent used in the management of osteoporosis. It is given by daily subcutaneous injection with resulting increases in bone mineral density and markers of bone turnover, including serum osteocalcin. Interestingly, although substantial increases in both of these parameters are noted in the early phases of treatment, the rise in bone turnover tends to wane over time (although it remains markedly above baseline), whereas bone mineral density continues to rise. This patient’s response, in terms of both bone density and bone remodeling, is typical for patients when assessed after 2 years of treatment. Because of concerns about the development of osteosarcoma (a complication described in rodent studies with the medication but not definitively linked to its use in humans) the US FDA has limited therapy to a period of 24 months. Thus, as with this patient, long-term management requires consideration of alternative strategies.

Rationale Continued: The effects of teriparatide do not persist after therapy. Both bone remodeling and bone density decline and return toward baseline values after treatment cessation (thus, Answer B is correct). Whether fracture protection is retained despite the decline in these parameters of bone health is uncertain. Strategies that have been used to retain the favorable actions of teriparatide include either subsequent (sequential) therapy with an antiresorptive agent such as a bisphosphonate, inhibitor of RANK ligand, estrogen or selective estrogen-receptor modulator or concurrent therapy with teriparatide and an antiresorptive agent. Many of these strategies appear successful in terms of bone mineral density, although no large-scale prospective fracture prevention trials have been completed. Answers A and C (both incorrect) reflect the subsequent use of an antiresorptive agent after teriparatide in which a patient maintained bone density or had further gains, which is commonly observed. Bone remodeling declines rapidly after cessation of therapy and hence Answers D and E are also incorrect.

Reference(s): Leder BZ, Neer RM, Wyland JJ, Lee HW, Burnett-Bowie SM, Finkelstein JS. Effects of teriparatide treatment and discontinuation in postmenopausal women and eugonadal men with osteoporosis. J Clin Endocrinol Metab. 2009;94(8):2915-2921. PMID: 19435827

ITE 2019 Question 73 A 50-year-old woman is referred for recommendations on treatment of osteoporosis that was recently identified by DXA scan. She has no history of low-trauma fractures as an adult. Her medical history is notable only for hypertension treated with lisinopril. She underwent natural menopause at age 46 years and did not take hormone therapy. She does not take calcium or vitamin D supplements, but she does consume 4 servings of dairy products per day. Her family history is notable for osteoporosis in her father who suffered a hip fracture in his 70s. Review of systems is notable for vague bilateral lower leg pain that is longstanding and worse with ambulation. She has no significant height loss. On physical examination, she has no evident thoracic kyphosis. There is moderate anterior tibial tenderness to direct palpation. The rest of her examination findings are noncontributory.

Laboratory test results: Serum calcium = 8.3 mg/dL (8.2-10.2 mg/dL) (SI: 2.1 mmol/L [2.1-2.6 mmol/L]) Serum phosphate = 2.3 mg/dL (2.3-4.7 mg/dL) (SI: 0.7 mmol/L [0.7-1.5 mmol/L]) Serum creatinine = 0.9 mg/dL (0.6-1.1 mg/dL) (SI: 79.6 µmol/L [53.0-97.2 µmol/L]) Serum albumin = 4.4 mg/dL (3.5-5.0 g/dL) (SI: 44 g/L [35-50 g/L]) Serum intact PTH = 90 pg /mL (10-65 pg/mL) (SI: 90 ng/L [10-65 ng/L])Serum 25-hydroxyvitamin D = 9 ng/dL (25-80 ng/mL) (SI: 22.5 nmol/L [62.4-199.7 nmol/L])Alkaline phosphatase = 145 U/L (50-120 U/L) (SI: 2.42 µkat/L [0.84-2.00 µkat/L])AST = 25 U/L (20-48 U/L) (SI: 0.42 µkat/L [0.33-0.80 µkat/L])ALT = 30 U/L (10-40 U/L) (SI: 0.50 µkat/L [0.17-0.67 µkat/L])

DXA results are shown (see images and tables):

Region BMD, g/cm 2 T-Score L1 0.642 –4.3 L20.703 –4.5 L3 0.698 –4.5 L4 0.618 –5.2 Total 0.662 –4.6

Region BMD, g/cm 2 T-Score Neck 0.682 –3.0 Total0.788 –2.2

Which of the following is the most appropriate treatment in the management of this patient’s bone disorder? Ergocalciferol Alendronate Zoledronic acid Calcium citrate Elemental phosphorus and calcitriol Correct Answer: A Learning objective:Identify subclinical osteomalacia due to vitamin D deficiency in a patient with osteoporosis and explain the importance of vitamin D repletion before bisphosphonate treatment.

Rationale: On initial review of this patient’s clinical data, she appears to have severe osteoporosis based on her bone mineral density assessment and would seem to be a good candidate for pharmacologic antifracture therapy. However, closer review of her clinical and biochemical data reveals information that should cause one to hesitate to use this approach. Her lower-extremity pain, while nonspecific, could suggest an alternative metabolic bone disorder ( ie , osteomalacia ). This suspicion is supported by the presence of tibial tenderness on examination, which in osteomalacia is thought to be due to expansion of the highly innervated periosteal lining by excess unmineralized osteoid. A diagnosis of osteomalacia is further supported biochemically by low-normal serum calcium and phosphate and elevated alkaline phosphatase. Most importantly and most likely the underlying etiology, this patient is frankly deficient in vitamin D with evidence of secondary hyperparathyroidism. Given that DXA cannot distinguish reduced bone mineral content from unmineralized osteoid, treatment with ergocalciferol (generally 50,000 IU weekly for 8 weeks) should be the initial treatment offered to this patient (Answer A). Additionally, existing evidence supports achievement of an adequate 25-hydroxyvitamin D status for optimal bone mineral density response and antifracture benefit.

Rationale Continued: For the reasons described above, bisphosphonate therapy (Answers B and C) would not be indicated until this patient’s vitamin D deficiency and associated osteomalacia are addressed. Moreover, bisphosphonate therapy could potentially precipitate severe hypocalcemia in this setting. Calcium supplementation independent of vitamin D repletion (Answer D) would not address underlying osteomalacia if present, and it has also not been shown to independently reduce the risk of osteoporotic fracture. Finally, calcitriol and phosphorus treatment (Answer E), which is prescribed to patients with hypophosphatemic osteomalacia due to disorders of fibroblast growth factor 23 excess, would not adequately address osteomalacia due to 25-hydroxyvitamin D deficiency.

Reference(s): Deane A, Constancio L, Fogelman I, Hampson G. The impact of vitamin D status on changes in bone mineral density during treatment with bisphosphonates and after discontinuation following long-term use in post-menopausal osteoporosis. BMC Musculoskelet Disord. 2007;8:3. PMID: 17214897Prieto-Alhambra D, Pagès-Castellà A, Wallace G, et al. Predictors of fracture while on treatment with oral bisphosphonates: a population-based cohort study. J Bone Miner Res. 2014;29(1):268-274. PMID: 23761350

ITE 2019 Question 76 A 42-year-old woman who is currently 10 weeks pregnant is referred to you for assistance in the management of hypercalcemia. Six months ago, a screening chemistry panel revealed a calcium concentration of 11.1 mg/dL (SI: 2.78 mmol/L) and a normal creatinine concentration of 0.7 mg/dL (SI: 61.9 µmol/L). Several calcium measurements in her 20s and 30s were normal. Six weeks ago, the patient returned for further laboratory tests when she had a positive home pregnancy test: Calcium = 10.8 mg/dL (8.2-10.2 mg/dL) (SI: 2.7 mmol/L [2.1-2.6 mmol/L]) Albumin = 4.0 g/dL (3.5-5.0 g/dL) (SI: 40 g/L [35-50 g/L]) Alkaline phosphatase = 42 U/L (50-120 U/L) (SI: 0.70 µ kat /L [0.84-2.00 µkat/L])PTH = 95.7 pg/mL (10-65 pg/mL) (SI: 95.7 ng/L [10-65 ng/L])25-Hydroxyvitamin D = 28 ng/mL (30-80 ng/mL [optimal]) (SI: 69.9 nmol/L [74.9-199.7 nmol/L])1,25-Dihydroxyvitamin D = 84 pg/mL (16-65 pg/mL) (SI: 218.4 pmol/L [41.6-169.0 pmol/L])Urinary calcium = 281 mg/24 h (100-300 mg/24 h) (SI: 7.0 mmol/d [2.5-7.5 mmol/d])Physical examination findings are normal, including those from head and neck exam. She is reluctant to do anything regarding her hypercalcemia out of concern for the fetus. The pregnancy progresses normally and the patient’s calcium concentration remains between 10.7 mg/dL and 11.3 mg/dL (2.7-2.8 mmol/L). The baby is delivered vaginally at 40 weeks’ gestation.

Which of the following represents the most likely immediate postpartum complication for the baby? Hyperparathyroidism Hypocalcemia Fracture Nephrocalcinosis Nephrolithiasis Correct Answer: B Learning objective:Counsel patients on the risk of maternal primary hyperparathyroidism to the developing fetus and newborn and review the physiology of calcium homeostasis in pregnancy.

Rationale: This patient presents with asymptomatic hypercalcemia, an elevated PTH level, and a urinary calcium excretion in the upper reference range. This pattern is diagnostic of primary hyperparathyroidism. Primary hyperparathyroidism is more common in women, and the incidence increases with age. Additional risk factors include a history of head and neck irradiation, as well as a family history of hyperparathyroidism. A number of familial hyperparathyroidism syndromes exist. Presentation with hyperparathyroidism at the relatively young age of 42 years is less common, but cases of primary hyperparathyroidism, as seen in multiple endocrine neoplasia syndromes, have been reported in adolescents and young adults. The coincidence of primary hyperparathyroidism and pregnancy is relatively rare and represents a significant medical management issue. First-trimester miscarriage, nephrolithiasis (due in part to the increased glomerular filtration of pregnancy combined with hypercalcemia), pancreatitis, and preeclampsia have been reported. Maternal life-threatening hypercalcemic crisis during the third trimester has been documented as well. To recognize the potential effect of maternal hyperparathyroidism on the developing fetus, one must understand the physiology of calcium regulation during pregnancy. Calcium is actively transported across the placenta from mother to fetus to ensure adequate substrate for skeletal and cellular development.

Rationale Continued: While maternal PTH levels remain normal, placental production of PTHrP increases such that maternal and fetal levels of PTHrP are significantly elevated. Fetal PTH levels are low because of active accumulation of calcium from the maternal circulation and the actions of PTHrP . Fetal calcium depends on maternal sources via placental transfer and regulation by placental PTHrP. Regarding vitamin D, 25-hydroxyvitamin D can diffuse across the placenta to provide substantial fetal levels. In contrast, 1,25-dihydroxyvitamin D does not cross the placenta and fetal levels depend on fetal 1α-hydroxylation, a process that is driven by PTHrP at the fetal kidney. Thus, maternal hyperparathyroidism results in mild fetal hypercalcemia in utero, further contributing to PTH suppression. At birth, fetal PTHrP and calcium levels fall rapidly as the placental transport of PTHrP and calcium cease. With little or no endogenous PTH (thus, Answer A is incorrect), newborns of mothers with primary hyperparathyroidism may have up to a 50% risk of developing neonatal hypocalcemia and, depending on the severity, overt tetany (thus, Answer B is correct). Fetal hypocalcemia can last for days to weeks. Fetal bone structure and strength are normal (thus, Answer C is incorrect). Nephrocalcinosis (Answer D) and nephrolithiasis (Answer E) are unlikely.

Reference(s): Diaz-Soto G, Linglart A, Sénat MV, Kamenicky P, Chanson P. Primary hyperparathyroidism in pregnancy. Endocrine. 2013;44(3):591-597. PMID: 23670708Harsoulis F, Karayiannis B, Karvounaris D, Mamopoulos M. Primary hyperparathyroidism in pregnancy. J Obstet Gynaecol. 2000;20(2):188-189. PMID: 15512516Malekar-Raikar S, Sinnott BP. Primary hyperparathyroidism in pregnancy-a rare cause of life-threatening hypercalcemia: case report and literature review. Case Rep Endocrinol. 2011;2011:520516. PMID: 22937284

ITE 2019 Question 80 A 42-year-old man presents with bilateral hip pain and the radiographic findings shown (see image). He underwent Roux-en-Y gastric bypass surgery for morbid obesity 3 years ago and has lost more than 100 lb (45.5 kg).

Laboratory test results: Serum calcium = 8.2 mg/dL (8.2-10.2 mg/dL) (SI: 2.1 mmol/L [2.1-2.6 mmol/L]) Phosphate = 2.2 mg/dL (2.3-4.7 mg/dL) (SI: 0.7 mmol/L [0.7-1.5 mmol/L]) Creatinine = 0.9 mg/dL (0.7-1.3 mg/dL) (SI: 79.6 µmol/L [61.9-114.9 µmol/L]) Serum alkaline phosphatase = 346 U/L (50-120 U/L) (SI: 5.78 µ kat /L [0.84-2.00 µ kat/L])

Measurement of which of the following is most likely to provide this patient’s diagnosis? 25-Hydroxyvitamin D 1,25-Dihydroxyvitamin D FGF-23 Intact PTH C-telopeptide Correct Answer: A Learning objective:Identify clinical and radiographic findings in osteomalacia after gastric bypass (severe vitamin D deficiency).

Rationale: Shown on the radiograph is a Looser zone, characteristic of osteomalacia . Mechanical stress of blood vessels overlying the uncalcified cortical bone affected by osteomalacia is thought to cause “ pseudofractures ” that appear as transverse zones of rarefaction, sometimes as wide as 1 cm, often multiple, and generally symmetric. Typical locations are the ischium, ilium, pubis, femur, tibia, radius, fibula, lower ribs, and scapula.This patient had malabsorption of both vitamin D and calcium after bariatric surgery, and he did not adhere to calcium and vitamin D supplementation. His serum 25-hydroxyvitamin D level (Answer A) was undetectable (<7 ng/mL [<17.5 nmol/L]). High-dosage vitamin D3, 100,000 IU daily, did not correct the vitamin D deficiency, but ultraviolet light (tanning salon) was successful. Chemical clues to osteomalacia include hypocalcemia, hypophosphatemia, and elevated alkaline phosphatase. Measuring FGF-23 (Answer C), 1,25-dihydroxyvitamin D (Answer B), intact PTH (Answer D), and C-telopeptide (Answer E) would not clarify his diagnosis.

Reference(s): Karefylakis C, Näslund I, Edholm D, Sundbom M, Karlsson FA, Rask E. Vitamin D status 10 years after primary gastric bypass: gravely high prevalence of hypovitaminosis D and raised PTH levels. Obes Surg. 2014;24(3):343-348. PMID: 24163201Bal BS, Finelli FC, Shope TR, Koch TR. Nutritional deficiencies after bariatric surgery. Nat Rev Endocrinol. 2012;8(9):544-556. PMID: 22525731Reginato AJ, Falasca GF, Pappu R, McKnight B, Agha A. Musculoskeletal manifestations of osteomalacia: report of 26 cases and literature review. Semin Arthritis Rheum. 1999;28(5):287-304. PMID: 10342386Thacher TD, Clarke BL. Vitamin D insufficiency. Mayo Clin Proc. 2011;86(1):50-60. PMID: 21193656Bhan A, Rao AD, Rao DS. Osteomalacia as a result of vitamin D deficiency. Endocrinol Metab Clin North Am. 2010;39(2):321-331. PMID: 20511054

ITE 2019 Question 86 An 82-year-old man presents for evaluation of a 6-month history of hip pain. The pain is worse at rest and wakes him up at night. Monostotic Paget disease of the right pelvis was diagnosed approximately 10 years ago and was treated with daily alendronate (40 mg) for 6 months. His pain resolved, and he did not feel the need to follow-up until now. He has no family history of Paget disease and no personal history of radiation exposure to the bone. His last PSA measurement and digital rectal examination findings were normal. On physical examination, he is an elderly man with an antalgic gait and he is in mild to moderate distress. His height is 65 in (165 cm), and weight is 178 lb (80.9 kg) (BMI = 29.6 kg/m2). There is warmth over the region of the right hip. Range of motion is limited by pain. No other joints appear to be involved. Laboratory test results:Creatinine = 0.83 mg/dL (0.7-1.3 mg/dL) (SI: 73.4 µmol/L [61.9-114.9 µmol/L])Calcium = 9.4 mg/dL (8.2-10.2 mg/dL) (SI: 2.4 mmol/L [2.1-2.6 mmol/L])25-Hydroxyvitamin D = 28 ng/mL (30-80 ng/mL [optimal]) (SI: 69.9 nmol/L [74.9-199.7 nmol/L])Alkaline phosphatase = 355 U/L (50-120 U/L) (SI: 5.93 µkat/L [0.84-2.00 µkat/L])PTH = 45 pg/mL (10-65 pg/mL) (SI: 45 ng/L [10-65 ng/L])PSA = 0.5 ng/mL (<7.2 ng/mL) (SI: 0.5 µg/L [<7.2 µg/L])Hematocrit, normalUrinary calcium, normal

The following shows the trend of his alkaline phosphatase concentrations over the past 6 years (see table). Measurement 6 Years Ago 5 Years Ago 4 Years Ago3Years Ago2 Years Ago 1 Year Ago 6 Months Ago 3 Months Ago Alkaline Phosphatase 47 U/L (SI: 0.78 µkat/L) 48 U/L (SI: 0.80 µkat/L) 55 U/L (SI: 0.92 µkat/L) 91 U/L (SI: 1.52 µkat/L) 83 U/L (SI: 1.39 µkat/L) 185 U/L (SI: 3.09 µkat/L) 255 U/L (SI: 4.26 µkat/L) 270 U/L (SI: 4.51 µ kat /L)

Plain film of the pelvis (see image) reveals the finding marked by the arrow.

Which of the following is the most likely cause of this patient’s clinical scenario? Osteoarthritis Pathologic fracture from metastatic prostate cancer Subtrochanteric stress fracture Development of osteosarcoma Development of an additional pagetoid lesion Correct Answer: D Learning objective:Identify transformation of Paget disease to osteosarcoma.

Rationale: Paget disease of bone is a disorder of bone remodeling caused by increased osteoclastogenesis and bone resorption resulting in focal areas of impaired bone quality with increased vascularity, deformity, and an increased propensity for fracture. The serum alkaline phosphatase concentration is usually elevated in patients with untreated Paget disease and it correlates with the extent of the disease. A rare complication of Paget disease is the transformation of the pagetoid bone into malignant osteosarcoma (Answer D), which is demonstrated in this patient. This is estimated to occur in less than 1% of cases and is described in patients with extensive longstanding skeletal disease. It has been hypothesized that this transformation results from a somatic mutation in the abnormal bone. Affected patients typically report a worsening of bone pain, which is unresponsive to medical therapy. Sarcoma should be suspected when new pain and swelling develop in a previously affected bone.

Rationale Continued: Because pagetic bone is larger and weaker than normal bone, it is associated with osteoarthritis from altered joint dynamics (Answer A); however, the joint space here looks relatively well preserved and this diagnosis would not explain the pattern of his alkaline phosphatase. Bone metastasis can cause the increase in alkaline phosphatase and pain, but the normal digital rectal examination and PSA make metastatic prostate cancer (Answer B) very unlikely here. Subtrochanteric stress fractures (Answer C) occur after treatment with bisphosphonates, even after the medications have been discontinued. While the mechanism remains unknown, thigh pain followed by acute exacerbation and a transverse fracture is the typical presentation. However, there is no “ beaking ” or other sign of subtrochanteric changes in this patient’s radiograph. Strikingly, the appearance of new pagetoid lesions (Answer E) is very uncommon. Most patients present with stable disease and most patients who are treated with bisphosphonates remain in remission. Disease recurrence is usually in the same location as the initial presentation. Thus, a new lesion is unlikely.

Reference(s): Bolland MJ, Cundy T. Paget’s disease of bone: clinical review and update. J Clin Pathol . 2013:66(11):924-927. PMID: 24043712Siris ES, Lyles KW, Singer FR, Meunier PJ. Medical management of Paget’s disease of bone: indications for treatment and review of current therapies. J Bone Miner Res. 2006;21(Supp 2):P94-P98. PMID: 17229018

DIABETES

ITE 2019 Question 1 A 61-year-old woman presents to her primary care clinic with polyuria, polydipsia, and lightheadedness and is subsequently admitted to the hospital for symptomatic hyperglycemia. Type 2 diabetes mellitus was recently diagnosed and metformin was initiated. However, after 5 months, her regimen was transitioned to NPH insulin and regular insulin due to poor glycemic control. The current total daily insulin dose is 170 units. Her history is notable for a 55.5-lb (27-kg) weight loss since the diagnosis of diabetes. On physical examination, she has temporal wasting and extensive acanthosis nigricans, including on the palms and tongue. Her height is 64 in (162.5 cm), and weight is 116 lb (52.7 kg) (BMI = 19.9 kg/m2). Laboratory test results: Anion gap, normalTriglycerides = 39 mg/dL (<150 mg/dL [optimal]) (SI: 0.44 mmol/L [<1.70 mmol/L])Her hospital course is characterized by the persistence of hyperglycemia without ketosis or fasting hypoglycemia. Intravenous insulin is initiated, peaking at 11,650 units daily.

Which of the following is the underlying cause of this patient’s presentation? Antiinsulin antibodies Antiinsulin receptor antibodies Occult infection Cushing syndrome Occult malignancyCorrect Answer: BLearning objective: Identify severe insulin resistance and the clinical and laboratory findings that suggest type B insulin resistance as a cause.

Rationale: This patient exhibits extreme insulin resistance, often described functionally as a requirement of greater than 200 units of insulin per day. Common causes of insulin resistance are obesity, pregnancy, exposure to excess glucocorticoids, acromegaly, and underlying metabolic stressors such as infection. In most circumstances, the etiology is unknown. This patient has several unusual features, namely substantially increased insulin requirements, extensive acanthosis, and a low serum triglyceride level. A rare cause of severe insulin resistance is an autoimmune condition in which antibodies to the insulin receptor develop (Answer B), known as type B insulin resistance. Affected persons are typically middle-aged, nonobese black women with acanthosis nigricans, and they often have other rheumatologic conditions. In typical cases of severe insulin resistance, one might expect to see elevated triglyceride levels. But in cases of type B insulin resistance, interestingly, serum triglyceride levels are low, as in the described patient. Similarly, circulating levels of the adipokine adiponectin, which often varies inversely with insulin sensitivity, are lower than anticipated for the degree of insulin resistance. While there are reports of antiinsulin antibodies (Answer A) being associated with severe insulin resistance, this is more rare than the classic type B insulin resistance. Interestingly, patients with antiinsulin antibodies may have worse outcomes than those with classic type B insulin resistance and often still require insulin after treatment with immunosuppressive regimens. This patient does not exhibit specific features to suggest Cushing syndrome (Answer D). While infection (Answer C) may exacerbate hyperglycemia in patients with diabetes mellitus, it would not explain the underlying cause of diabetes in this patient. Occult malignancy (Answer E) would not explain this clinical presentation.

Rationale Continued: Type B insulin resistance is important to recognize, as it is associated with high mortality—as high as 54% in one series. Furthermore, the National Institutes of Health has reported a protocol involving rituximab, cyclophosphamide, and dexamethasone followed by maintenance azathioprine that is associated with long-term remission of diabetes and freedom from insulin requirement.

Reference(s): Semple RK, Cochran EK, Soos MA, et al. Plasma adiponectin as a marker of insulin receptor dysfunction: clinical utility in severe insulin resistance. Diabetes Care . 2008;31(5):977-979. PMID: 18299442 Flier JS, Kahn CR, Roth J, Bar RS. Antibodies that impair insulin receptor binding in an unusual diabetic syndrome with severe insulin resistance. Science. 1975;190(4209):63-65. PMID: 170678Arioglu E, Andewelt A, Diabo C, Bell M, Taylor SI, Gorden P. Clinical course of the syndrome of autoantibodies to the insulin receptor (type B insulin resistance): a 28-year perspective. Medicine (Baltimore). 2002;81(2):87-100. PMID: 11889410Malek R, Chong AY, Lupsa BC, et al. Treatment of type B insulin resistance: a novel approach to reduce insulin receptor autoantibodies. J Clin Endocrinol Metab. 2010;95(8):3641-3647. PMID: 20484479

ITE 2019 Question 4 A 24-year-old woman with an 8-year history of type 1 diabetes mellitus is evaluated in the emergency department for management of hyperglycemia. Her most recent hemoglobin A1c value was 6.8% (51 mmol/mol) 8 weeks ago. The night before admission, her insulin pump malfunctioned and despite increased fluid intake and several subcutaneous insulin injections, she developed nausea, vomiting, and abdominal pain. Urine ketones were strongly positive. On physical examination, she is awake and able to answer questions, but she appears fatigued and pauses between sentences. She is afebrile, blood pressure is 96/68 mm Hg, pulse rate is 112 beats/min, and respiratory rate is 24 breaths/min. Her height is 64 in (162.5 cm), and weight is 121 lb (55 kg) (BMI = 20.8 kg/m2). Oral examination reveals dry mucous membranes. She is tachypneic, and Kussmaul respiration is noted. There is diffuse abdominal tenderness but no rebound, guarding, or focal pain. The rest of her physical examination findings are normal.

Initial laboratory test results reveal: pH = 7.0 (7.35-7.45) Anion gap = 17 Blood glucose = 268 mg/dL (70-99 mg/dL) (SI: 14.9 mmol/L [3.9-5.5 mmol/L]) Serum urea nitrogen = 56 mg/dL (8-23 mg/dL) (SI: 20.0 mmol/L [2.9-8.2 mmol/L]) Creatinine = 2.6 mg/dL (0.6-1.1 mg/dL) (SI: 229.8 µmol/L [53.0-97.2 µmol/L]) Sodium = 135 mEq/L (136-142 mEq/L) (SI: 135 mmol/L [136-142 mmol/L])Potassium = 2.5 mEq/L (3.5-5.0 mEq/L) (SI: 3.2 mmol/L [3.5-5.0 mmol/L])Magnesium = 1.7 mg/dL (1.5-2.3 mg/dL) (SI: 0.7 mmol/L [0.6-0.9 mmol/L])Phosphate = 3.6 mg/dL (2.3-4.7 mg/dL) (SI: 1.2 mmol/L [0.7-1.5 mmol/L])

Which of the following is the best initial prescription in this patient’s management? Give 6 units regular insulin intravenously and infuse 0.45% NaCl at 100 cc/h with 40 mEq KCl per L over 1 hour Give 6 units regular insulin intravenously and infuse 1000 mL 0.9% NaCl with 40 mEq KCl over 1 hourInfuse 1000 mL 0.45% NaCl with 40 mEq KCl over 1 hourInfuse 1000 mL 0.9% NaCl with 40 mEq of KCl over 1 hourInfuse 0.45% NaCl at 100 cc/h with 40 mEq of KCl per L Correct Answer: D Learning objective : Recommend appropriate fluid and electrolyte management in the setting of diabetic ketoacidosis.

Rationale: Appropriate fluid and electrolyte management is crucial in the treatment of diabetic ketoacidosis. This vignette focuses on fluid and potassium replacement in this setting. If there is no evidence of cardiac compromise, the initial fluid choice should be 0.9% NaCl to correct hypovolemia. An initial rate of fluid administration of 15 to 20 mL/kg is recommended for the first hour. Following this, if serum sodium is normal or high, fluid replacement can be switched to 0.45% NaCl. If serum sodium is low, 0.9% NaCl should be continued. This patient weighs 121 lb (55 kg), so 1000 mL replacement in first hour is a reasonable volume. The answer options that suggest initial fluid replacement should be 100 cc/h (Answers A and E) are incorrect because the replacement is not fast enough. 0.45% NaCl (Answer C) would not be the initial choice. The serum potassium level at admission determines the timing of insulin use and the extent of potassium replacement. If the potassium value is less than 3.3 mEq /L, 20 to 40 mEq is added to each liter of fluid and replaced in the first hour. In addition, insulin administration is delayed until the potassium rises above 3.3 mEq/L. Since the patient’s serum potassium is less than 3.3 mEq/L, insulin should not be administered now (Answers A and B).The best choice is Answer D because no insulin is started, 0.9% NaCl is used with adequate KCl (40 mEq), and it is infused at an aggressive rate of 1000 mL over the first hour.

Reference(s): Fayfman M, Pasquel FJ, Umpierrez GE. Management of hyperglycemic crises: diabetic ketoacidosis and hyperglycemic hyperosmolar state. Med Clin North Am. 2017;101(3):587-606. PMID: 28372715Umpierrez G, Korytkowski M. Diabetic emergencies – ketoacidosis, hyperglycaemic hyperosmolar state and hypoglycaemia. Nat Rev Endocrinol. 2016;12(4):222-232. PMID: 26893262Savage MW, Dhatariya KK, Kivert A, et al; Joint British Diabetes Societies. Joint British Diabetes Societies guideline for the management of diabetic ketoacidosis. Diabet Med. 2011;28(5)508-515. PMID: 21255074Kitabchi AE, Umpierrez GE, Miles JM, Fisher JN. Hyperglycemic crises in adult patients with diabetes. Diabetes Care. 2009;32(7):1335-1343. PMID: 19564476

ITE 2019 Question 6 A 66-year-old woman with type 2 diabetes mellitus has been taking metformin and liraglutide for the past 2 years. She has no personal history or family history of cardiovascular disease, but hypertension was recently diagnosed. Lisinopril was started and titrated to maximum dosing. On physical examination, her seated blood pressure is 150/90 mm Hg, and pulse rate is 76 beats/min. Her weight is 174 lb (79.1 kg); she has lost 5.5 lb (2.5 kg) in the last year. Her BMI is 28.0 kg/m2. Recent laboratory test results:Hemoglobin A1c = 6.4% (4.0%-5.6%) (46 mmol/mol [20-38 mmol/mol])Fasting blood glucose = 120 mg/dL (70-99 mg/dL) (SI: 6.7 mmol/L [3.9-5.5 mmol/L])Serum potassium = 3.8 mEq/L (3.5-5.0 mEq/L) (SI: 3.8 mmol/L [3.5-5.0 mmol/L])Serum creatinine = 1.0 mg/dL (0.6-1.1 mg/dL) (SI: 88.4 µmol/L [53.0-97.2 µmol/L])

In addition to maintaining lisinopril therapy, which of the following medications should be added to manage this patient’s blood pressure? Amlodipine Losartan Metoprolol Hydrochlorothiazide Correct Answer: A Learning objective :Manage hypertension in patients with type 2 diabetes mellitus.

Rationale: The use of antihypertensive agents in patients with type 2 diabetes mellitus is based on the need to prevent adverse cardiovascular events and limit the progression of renal disease. Several clinical trials have been conducted to clarify the choice of antihypertensive agent. The Avoiding Cardiovascular Events through Combination Therapy in Patients Living with Systolic Hypertension (ACCOMPLISH) trial was designed to compare cardiovascular outcomes after ACE inhibitor treatment in combination with amlodipine vs combination with a thiazide diuretic. The ACCOMPLISH trial demonstrated that the ACE inhibitor + amlodipine combination (Answer A) provided better protection against cardiovascular outcomes in patients with diabetes than the ACE inhibitor + low-dosage hydrochlorothiazide combination (Answer D). Although the Antihypertensive and Lipid-Lowering Treatment to Prevent Heart Attack Trial (ALLHAT) demonstrated that patients with diabetes had a significantly lower rate of new-onset heart failure with low-dosage thiazide compared with the rate in patients taking amlodipine and lisinopril, the ACCOMPLISH trial data did not support the use of a thiazide. In addition, this patient has a low-normal serum potassium level and thiazide diuretics have an adverse effect on glucose metabolism. Although both ACE inhibitors and angiotensin II receptor blockers (Answer B) protect against the development of diabetic kidney disease, they should not be used in combination. Losartan was more effective than atenolol in reducing cardiovascular morbidity and mortality, as well as mortality from all causes, in patients with hypertension, diabetes, and left ventricular hypertrophy. If a β-adrenergic blocker is given, carvedilol may be the drug of choice because of potential benefits on glycemic control and lower rate of developing moderately increased albuminuria compared with outcomes with metoprolol (Answer C).

Reference(s): American Diabetes Association. 9. Cardiovascular disease and risk management: standards of medical care in diabetes-2018. Diabetes Care . 2018;41(Suppl 1):S86-S104. PMID: 29222380 Jamerson KA, Bakris GL, Wun CC, et al. Rationale and design of the avoiding cardiovascular events through combination therapy in patients living with systolic hypertension (ACCOMPLISH) trial: the first randomized controlled trial to compare the clinical outcome effects of first-line combination therapies in hypertension. Am J Hypertens. 2004;17(9):793-801. PMID: 15363822Lindholm LH, Ibsen H, Dahlöf B, Devereux RB, Beevers G, de Faire U, Fyhrquist F, Julius S, Kjeldsen SE, Kristiansson K, Lederballe-Pedersen O, Nieminen MS, Omvik P, Oparil S, Wedel H, Aurup P, Edelman J, Snapinn S, LIFE Study Group. Cardiovascular morbidity and mortality in patients with diabetes in the Losartan Intervention For Endpoint reduction in hypertension study (LIFE): a randomised trial against atenolol. Lancet. 2002;359(9311):1004-1010. PMID: 11937179

ITE 2019 Question 9 A 45-year-old woman has a skin lesion on her left lower extremity (see image). A similar but smaller lesion is also present on her right lower extremity. Biopsy documents an inflammatory granulomatous dermatitis with collagen degeneration and fat deposition.

Which of the following diagnoses does this patient most likely have? Type 1 diabetes mellitus Graves disease Glucagonoma Pseudohypoparathyroidism Familial hypercholesterolemia Correct Answer: A Learning objective:Diagnose necrobiosis lipoidica and recognize its association with diabetes mellitus.

Rationale: The lesion in the photograph is necrobiosis lipoidica , which frequently occurs in association with diabetes mellitus (Answer A), thus accounting for the past use of the term necrobiosis lipoidica diabeticorum for this condition. Necrobiosis lipoidica usually begins as asymptomatic, well-circumscribed, yellow to pink-brown or red-brown, slightly elevated papules or plaques. Erythema or a violaceous skin color can be present at the periphery. Necrobiosis lipoidica is most often found on the pretibial area, but can involve the scalp, face, trunk, genitals, or upper extremities. Biopsy of the lesion shows collagen degeneration with a granulomatous and inflammatory response, thickening of blood vessel walls, and fat deposition in the dermis.The other listed conditions can have associated skin manifestations that look different from this patient’s lesion. Glucagonoma (Answer C) is associated with necrolytic migratory erythema (biopsy would show superficial necrolysis with separation of the outer layers of the epidermis and perivascular infiltration with lymphocytes and histiocytes). Graves disease (Answer B) is associated with pretibial myxedema (biopsy would show mucinous edema and the fragmentation of collagen fibers with deposition of acid mucopolysaccharides [hyaluronic acid] in the papillary and reticular dermis). Pseudohypoparathyroidism (Answer D) can be associated with subcutaneous calcifications. Familial hypercholesterolemia (Answer E) can be associated with xanthomas (biopsy would show deposition of lipid and associated inflammation in the skin).

Reference(s): Murphy- Chutorian B, Han G, Cohen SR. Dermatologic manifestations of diabetes mellitus: a review. Endocrinol Metab Clin North Am . 2013;42(4):869-898. PMID: 24286954Reid SD, Ladizinski B, Lee K, Baibergenova A, Alavi A. Update on necrobiosis lipoidica: a review of etiology, diagnosis, and treatment options. J Am Acad Dermatol. 2013;69(5):783-791. PMID: 23969033Jabbour SA. Skin manifestations of hormone-secreting tumors. Dermatol Ther. 2010;23(6):643-650. PMID: 21054708

ITE 2019 Question 11 A 56-year-old man with a 15-year history of type 2 diabetes mellitus complicated by peripheral neuropathy and microalbuminuria returns for follow-up. He required the addition of insulin to oral hypoglycemic agents soon after diagnosis. His insulin requirements have consistently increased since that time. His current medication regimen includes metformin, 1000 mg twice daily; insulin glargine, 65 units twice daily; and aspart insulin, 80 units 3 times daily with meals. He reports excellent adherence to his prescribed insulin regimen and monitors his fingerstick blood glucose levels 3 to 4 times daily. His carefully maintained blood glucose log reveals values ranging between 120 and 280 mg/dL (6.7-15.5 mmol/L). He has had no recent hypoglycemia and has never had problems with hypoglycemia unawareness. On physical examination, his blood pressure is 128/87 mm Hg, and pulse rate is 85 beats/min. His height is 69 in (175.3 cm), and weight is 230 lb (104.5 kg) (BMI = 34 kg/m 2). Laboratory test results:Hemoglobin A1c = 7.6% (4.0%-5.6%) (62 mmol/mol [20-38 mmol/mol])Creatinine = 1.0 mg/dL (0.7-1.3 mg/dL) (SI: 88.4 µmol/L [61.9-114.9 µmol/L])Estimated glomerular filtration rate = 82.2 mL/min per 1.73 m2 (>60 mL/min per 1.73 m2)

The best next step in this patient’s care is to change his regimen to which of the following? U500 regular insulin, 0.2 mL (100 units) 2 times daily before meals U500 regular insulin, 0.15 mL (75 units) 3 times daily before meals U500 regular insulin, 0.2 mL (100 units) 3 times daily before meals U500 regular insulin, 0.25 mL (125 units) 3 times daily before meals U500 regular insulin, 0.3 mL (150 units) 3 times daily before meals Correct Answer: C Learning objective:Guide conversion from a U100 to a U500 regular insulin regimen in a patient with severe insulin resistance.

Rationale: This patient has inadequate glycemic control despite use of metformin and 370 units of U100 insulin daily. He has severe insulin resistance, generally defined as an insulin requirement greater than 200 units of insulin daily. Use of such very high insulin doses is inconvenient because multiple injections per dose are required and high volumes may lead to injection site discomfort. In addition, the absorption of insulin from large-volume subcutaneous depots may be poor and/or unpredictable. In patients such as this, conversion from U100 to 5-fold concentrated U500 regular insulin (U500R) represents a very reasonable alternative glycemic management strategy. Studies of U500R insulin action suggest that this insulin formulation has an onset of action similar to that of U100 regular insulin and a duration of action similar to that of U100 NPH insulin. The per-unit cost of U500R is estimated to be less than that of U100 insulin, and the costs of needles and syringes may also be reduced because fewer injections are needed. Use of U500R in clinical practice has resulted in instances of clinically significant hypoglycemia because of dosing errors. U500R is available in a vial and, as of 2016, in pen form. For syringes, some prescribers prefer to have patients draw up U500R in a U100 insulin syringe with guidance on the conversion of doses. However, the Institute for Safe Medication Practices has recommended “consistent use of a tuberculin syringe with U500 insulin, with total doses expressed in terms of both units and volume, ie , 200 units (0.4 mL).” Irrespective of the type of syringe chosen for insulin administration, the patient and any caregivers must be fully educated regarding the correct drawing up and administration of prescribed doses. Very careful attention must be paid to insulin dosing when users of U500R are hospitalized, as many reported errors in insulin dosing have occurred in hospitalized patients.

Rationale Continued: Recommendations for the conversion from U100 insulin to U500R generally incorporate the baseline U100 insulin doses, as well as the hemoglobin A1c level, to determine appropriate doses of the concentrated insulin. In general, experts recommend that the total daily insulin dose be increased by 10% in patients with a hemoglobin A1c level above 10.0% (>86 mmol/mol); that the total daily insulin dose be maintained in patients with a hemoglobin A1c level between 8.0% and 10.0% (64-86 mmol/mol); and that the total daily dose be decreased by 10% to 20% in patients with a hemoglobin A1c level below 8% (<64 mmol/mol) at the time of conversion. It should be noted, however, that some experts recommend total daily insulin dose reductions of 10% to 20% for all patients when their regimen is converted to U500R. In patients using 150 to 300 units of insulin daily, the total amount may be administered in 2 or 3 divided doses, while users of 300 to 600 units daily require 3 to 4 injections daily or continuous subcutaneous insulin infusion. Users of more than 600 units of insulin daily should have their regimens converted to either 4 insulin injections daily or continuous subcutaneous insulin infusion.  This patient is currently taking a total of 370 units of U100 insulin daily and has a hemoglobin A1c level of 7.6% (62 mmol/mol). Conversion to U500R at 0.2 mL (100 units) 2 times daily (Answer A) or 0.15 mL (75 units) 3 times daily (Answer B) represents approximately 40% to 45% reductions in his total daily insulin dosing, and thus would provide less insulin than he would be expected to require. Use of U500R at 0.2 mL (100 units) 3 times daily (Answer C) provides an approximately 20% reduction in his total daily insulin dosing. As his hemoglobin A1c level is currently less than 8.0% (<64 mmol/mol), this option represents the best next step in this patient’s care. Conversion to 0.25 mL (125 units) 3 times daily (Answer D) is incorrect because it provides no reduction in his total daily insulin dosage. Conversion to 0.3 mL (150 units) 3 times daily (Answer E) is incorrect because this in fact represents an unwarranted increase in insulin administration.

Reference(s): Reutrakul S, Wroblewski K, Brown RL. Clinical use of U-500 regular insulin: review and meta-analysis. J Diabetes Sci Technol . 2012;6(2):412-420. PMID: 22538155Dailey AM, Tannock LR. Extreme insulin resistance: indications and approaches to the use of U-500 insulin in type 2 diabetes mellitus. Curr Diab Rep. 2011;11(2):77-82. PMID: 21161448Institute for Safe Medication Practices. ISMP Medication Safety Alert. As U-500 Insulin Safety Concerns Mount, It’s Time to Rethink Safe Use of Strengths Above U-100. Available at https://www.ismp.org/resources/u-500-insulin-safety-concerns-mount-its-time-rethink-safe-use-strengths-above-u-100. Accessed for verification October 2018.

ITE 2019 Question 17 A 42-year-old man seeks follow-up of diabetes mellitus that was diagnosed during a hospitalization 3 months ago when he presented with polyuria, shortness of breath, and abdominal pain. He reports that at the time of initial presentation, his blood glucose concentration was 700 mg/dL (38.9 mmol/L) and his hemoglobin A1c level was 12.0% (108 mmol/mol). Outside hospital records document that he had marked metabolic acidosis and ketonemia on admission. He was treated with intravenous insulin and was discharged 4 days later on insulin glargine, 10 units at bedtime daily. He has not been monitoring his blood glucose at home, but he reports feeling that his blood glucose is low most mornings. His medical history is unremarkable, and his family history is notable for type 2 diabetes diagnosed in his father at age 65 years. On physical examination, his blood pressure is 122/73 mm Hg and BMI is 32 kg/m2. The rest of his examination findings are unremarkable.

Which of the following sets of measurements would most accurately predict this patient’s need for long-term insulin management? BMI, glutamic acid decarboxylase antibodies, and islet antigen-2 antibodies Glutamic acid decarboxylase antibodies, hemoglobin A1c, and islet antigen-2 antibodies BMI, fasting C-peptide, and hemoglobin A1c Fasting C-peptide, glutamic acid decarboxylase antibodies, and islet antigen-2 antibodies Correct Answer: D Learning objective :Choose the appropriate classification system for predicting duration of insulin therapy in ketosis-prone diabetes.

Rationale: Ketosis-prone diabetes, previously referred to as Flatbush diabetes or type 1b diabetes, has been increasingly recognized since the mid-1990s. In the United States, 20% to 50% of newly diagnosed patients are black or Hispanic. Although these patients present with diabetic ketoacidosis similar to a typical patient with type 1 diabetes, they may not require long-term insulin therapy.    To facilitate the understanding of the diagnosis and predict ongoing insulin requirement in patients who present with an acute diabetic ketoacidosis episode, 4 classification systems have been developed. These systems categorize patients using criteria such as markers of β-cell autoimmunity, proxies for β-cell function, and/or BMI. The system of classification that most accurately (99% sensitivity and 96% specificity) predicts the need for insulin treatment 12 months after presentation with diabetic ketoacidosis is known as the Aß system, which uses evidence of autoimmunity (autoantibodies against glutamic acid decarboxylase or islet-cell autoantibodies) and markers of β-cell function (thus, Answer D is correct). Other classification systems measure BMI as a surrogate for β-cell function, but do not include β-cell autoimmunity (thus, Answer A is incorrect). The American Diabetes Association classification measures only β-cell autoimmunity and does not include hemoglobin A1c (thus, Answer B is incorrect). None of the 4 classification systems measures β-cell function with hemoglobin A1c (thus, Answer C is incorrect).

Reference(s): Banerji MA, Dham S. A comparison of classification schemes for ketosis-prone diabetes. Nat Clin Pract Endocrinol Metab . 2007;3(7):506-507. PMID: 17489086Balasubramanyam A, Garza G, Rodriguez L, et al. Accuracy and predictive value of classification schemes for ketosis-prone diabetes. Diabetes Care. 2006;29(12):2575-2579. PMID: 17130187Mauvais-Jarvis F, Sobngwi E, Porcher R, et al. Ketosis-prone type 2 diabetes in patients of sub-Saharan African origin: clinical pathophysiology and natural history of beta-cell dysfunction and insulin resistance. Diabetes. 2004;53(3):645-653. PMID: 14988248

ITE 2019 Question 23 A 19-year-old man presents for continued management of type 1 diabetes mellitus, having “aged-out” of pediatric endocrine care. Diabetes was diagnosed at age 16 years when glycosuria and moderate hyperglycemia were found on a yearly checkup. Insulin therapy was started immediately. His current insulin dose is approximately 0.4 units/kg per day, administered as multiple daily injections, and his current hemoglobin A1c level is 6.4% (46 mmol/mol) with occasional hypoglycemia. His family history is positive for diabetes in his mother, maternal grandfather, and an older sibling. His BMI is 23 kg/m2. Tests for glutamic acid decarboxylase antibodies, islet-cell antibodies, and insulinoma-associated protein 2 (IA-2) antibodies are negative, although he did not have antibody testing at diagnosis.

Optimal management of this patient’s diabetes will most likely include which of the following? Insulin administration via a continuous subcutaneous insulin infusion pump Addition of pramlintide Discontinuation of insulin and initiation of metformin Discontinuation of insulin and initiation of glimepiride Correct Answer: D Learning objective :Diagnose monogenic diabetes mellitus that was initially misdiagnosed as type 1 diabetes and assess the treatment implications.

Rationale: Monogenic forms of diabetes comprise a heterogeneous group of disorders. They are caused by single gene pathogenic variants and are characterized by impaired insulin secretion. It is estimated that up to 1% to 5% of all diabetes cases are monogenic and affected patients are often undiagnosed or are misclassified as having type 1 or type 2 diabetes. Correctly diagnosing monogenic forms of diabetes is important because an incorrect diagnosis of type 1 or type 2 diabetes can result in suboptimal treatment and delays in diagnosing other family members. Monogenic diabetes includes maturity-onset diabetes of the young (MODY), mitochondrial diabetes, and neonatal diabetes. Many pathogenic variants have been identified that cause diabetes by disturbing the coupling of blood glucose concentration and insulin secretion. MODY 3, associated with pathogenic variants in the HNF1A gene (hepatocyte nuclear factor-1-alpha gene on chromosome 12), is the most prevalent MODY form. The second most common form, MODY 2, is associated with pathogenic variants in the GCK gene (glucokinase).  The patient in this vignette has MODY, characterized by (1) young age at diagnosis, often under 25 years, (2) a marked family history of diabetes in every generation due to autosomal dominant inheritance, (3) absence of obesity and signs of insulin resistance, and (4) commonly mild hyperglycemia without the need for insulin therapy and negative results for β-cell antibodies. MODY 3 presents with early glycosuria and hyperglycemia, which is often postprandial. Patients with MODY 3 are at risk for microvascular and macrovascular complications of type 1 and type 2 diabetes. In addition, patients with diabetes caused by a pathogenic variant in HNF1A appear to have an increased risk of cardiovascular mortality compared with risk of unaffected family members. The diagnosis can be confirmed by genetic testing.

Rationale Continued: Optimal treatment for MODY 3 is sulfonylureas (thus, Answer D is correct and Answer A is incorrect). One study documented a significantly greater drop in hemoglobin A 1c level with a sulfonylurea than with metformin (Answer C). Almost 70% of patients previously treated with insulin are successfully switched to sulfonylureas once a pathogenic variant in HNF1A is identified. Pramlintide (Answer B) is not indicated because his hemoglobin A 1c level is less than 7.0% (<53 mmol/mol) and he is experiencing hypoglycemic episodes.

Reference(s): Henzen C. Monogenic diabetes mellitus due to defects in insulin secretion. Swiss Med Wkly . 2012;142:w13690. PMID: 23037711 Thanabalasingham G, Owen KR. Diagnosis and management of maturity onset diabetes of the young (MODY). BMJ. 2011;343:d6044. PMID: 22012810Wherrett DK, Bundy B, Becker DJ, et al; Type 1 Diabetes TrialNet GAD Study Group. Antigen-based therapy with glutamic acid decarboxylase (GAD) vaccine in patients with recent-onset type 1 diabetes: a randomised double-blind trial. Lancet. 2011;378(9788):319-327. PMID: 21714999Shepherd M, Shields B, Ellard S, Rubio-Cabezas O, Hattersley AT. A genetic diagnosis of HNF1A diabetes alters treatment and improves glycaemic control in the majority of insulin-treated patients. Diabet Med. 2009;26(4):437-441. PMID: 19388975

ITE 2019 Question 24 A 56-year-old man is referred for evaluation of obesity. His medical history is notable for HIV, treated with antiretroviral therapy. His viral load is undetectable, and he has no history of opportunistic infections. He also has hypertension and dyslipidemia. On review of systems, he had steady weight gain of 50 lb (22.7 kg) over the past 10 years. His father died of heart disease at age 45 years, and his mother has hypertriglyceridemia. Current medications include a once-daily combination pill for antiretroviral therapy and atorvastatin, 10 mg daily. On physical examination, his blood pressure is 130/80 mm Hg and pulse rate is 80 beats/min. His height is 66.5 in (169 cm), and weight is 220 lb (100 kg) (BMI = 35 kg/m 2). He has moderate truncal obesity and no lipoatrophy.Laboratory test results:LDL cholesterol = 180 mg/dL (<100 mg/dL [optimal]) (SI: 4.66 mmol/L [<2.59 mmol/L])HDL cholesterol = 25 mg/dL (>60 mg/dL [optimal]) (SI: 0.65 mmol/L [>1.55 mmol/L])Triglycerides = 450 mg/dL (<150 mg/dL [optimal]) (SI: 5.09 mmol/L [<1.70 mmol/L])

Measurement of which of the following is the best method to screen for the presence of diabetes mellitus in this patient? C-peptide Random glucose Hemoglobin A 1c Fructosamine Fasting glucoseCorrect Answer: ELearning objective:Recommend appropriate screening for diabetes mellitus in a patient with HIV infection.

Rationale: With improvement in antiretroviral therapies and excellent clinics and physicians available, patients with HIV can live a normal lifespan. However, the disease itself and various antiretroviral medications, in particular protease inhibitors, can predispose patients to diabetes. The cause of this increased risk appears to be via increased insulin resistance and possibly apoptosis of pancreatic β cells. Nucleoside reverse transcriptase inhibitors also may increase risk for diabetes by adversely affecting fat distribution. Because of this increased risk, the American Diabetes Association recommends that patients with HIV be screened for diabetes and prediabetes by measuring fasting glucose (Answer E) every 6 to 12 months before starting antiretroviral therapy and every 3 months after starting or changing antiretroviral therapy. If initial screening results are normal, then checking fasting glucose every year is advised. Screening this patient by measuring hemoglobin A1c (Answer C) is not recommended because it underestimates glycemia in the setting of HIV infection. A random glucose measurement (Answer B) is only useful if the value is less than 100 mg/dL (<5.6 mmol/L) or ≥200 mg/dL (≥11.1 mmol/L), so it is not the best test. C-peptide levels (Answer A) and fructosamine (Answer D) are not used in the diagnosis of diabetes.

Reference(s): American Diabetes Association. 3. Comprehensive medical evaluation and assessment of comorbidities: standards of medical care in diabetes-2018. Diabetes Care . 41(Suppl 1):S28-S37. PMID: 29222374 Kim PS, Woods C, Georgoff P, et al. A1C underestimates glycemia in HIV infection. Diabetes Care. 2009;32(9):1591-1593. PMID: 19502538

ITE 2019 Question 29 A 68-year-old man with a history of ulcerative colitis and benign prostatic hypertrophy presents with fatigue, a 22-lb (10-kg) weight loss, diarrhea, anorexia, and increasing nocturia. His fasting blood glucose level is 236 mg/dL (13.1 mmol/L). One year ago, a random blood glucose measurement was 115 mg/dL (6.4 mmol/L). On physical examination, he is a cachectic elderly man with a violaceous rash across his feet (see image) that has been unsuccessfully treated with topical clotrimazole. Laboratory analysis is notable for anemia and low levels of zinc and essential fatty acids.

Measurement of which of the following is most likely to reveal the etiology of his diabetes? Glucagon Somatostatin CA 19-9 24-Hour urinary free cortisol Correct Answer: A Learning objective :Diagnose glucagonoma as a secondary cause of diabetes mellitus.

Rationale: The development of type 2 diabetes mellitus after age 65 to 70 years, particularly in a relatively abrupt fashion, is unusual and should prompt consideration of a secondary cause. Categories of secondary diabetes include medication-induced ( eg , corticosteroids), other endocrinopathies ( eg , acromegaly), pancreatic diseases ( eg , pancreatitis), infections (eg, cytomegalovirus), and genetic conditions (eg, Rabson-Mendenhall syndrome). In this older patient with cachexia and rash on his feet, glucagonoma syndrome should be considered. Thus, measuring glucagon (Answer A) is correct. Glucagonomas are rare neuroendocrine tumors of the pancreas (others include insulinomas, somatostatinomas, carcinoid, and nonsecreting neuroendocrine tumors). In this setting, hyperglycemia, which is typically quite severe, results predominantly through the counterregulatory effects of glucagon. In larger tumors, destruction of nearby islet cells and pancreatic insulin secretion may also have an etiologic role. Glucagonomas are often malignant and frequently present already metastatic to the liver. If localized, surgical resection is necessary. Unless large portions of the pancreas are sacrificed, hyperglycemia typically resolves relatively rapidly postoperatively. If metastatic or residual tumor is demonstrated after surgery, somatostatin receptor agonist therapy (eg, octreotide, lanreotide) should be considered. Necrolytic migratory erythema is the typical rash associated with glucagonoma syndrome, as observed in this patient. It can be itchy and painful and it often affects the genital and anal region, groin, buttocks, and lower legs, but any site can be involved. The rash is nontender with irregular borders, sometimes associated with scaling or crusting, and progresses through an initial ring-shaped red area that blisters, erodes, then crusts over and leaves behind a brown mark.

Rationale Continued: Somatostatinomas are very rare and also not associated with rash, so somatostatin (Answer B) does not need to be measured in this patient. CA 19-9 (carbohydrate antigen) (Answer C) is elevated in adenocarcinoma of the pancreas, as well as in colorectal and hepatocellular carcinomas. While any tumor of the pancreas can lead to diabetes mellitus, pancreatic adenocarcinoma is not associated with necrolytic migratory erythema. No clinical features in this vignette suggest Cushing syndrome; therefore, measurement of urinary free cortisol (Answer D) is unlikely to be helpful.

Reference(s): Jabbour SA. Skin manifestations of hormone-secreting tumors. Dermatol Ther . 2010;23(6):643-650. PMID: 21054708  Warner RR. Enteroendocrine tumors other than carcinoid: a review of clinically significant advances. Gastroenterology. 2005;128(6):1668-1684. PMID: 15887158

ITE 2019 Question 35 A 21-year-old woman with a 3-year history of type 1 diabetes mellitus returns for follow-up during her summer break from college. She is using multiple-daily injections of insulin with mealtime dosing based on her carbohydrate intake, and she measures her blood glucose 3 to 4 times daily. A physician near her university monitors her closely during the school year, and the patient reports overall reasonable glycemic control without clinically significant hypoglycemia. She has had no recent changes in her weight or energy level and has regular menses. She does not smoke cigarettes. She has seen an ophthalmologist for comprehensive dilated eye examinations on 2 occasions, with no evidence of retinopathy found during her appointment 1 year ago. On physical examination, her blood pressure is 116/72 mm Hg and pulse rate is 68 beats/min. Her height is 67 in (170.2 cm), and weight is 136 lb (61.8 kg) (BMI = 21.3 kg/m 2 ). Laboratory data obtained at last year’s visit to your office include:Hemoglobin A1c = 6.8% (4.0%-5.6%) (51 mmol/mol [20-38 mmol/mol])Creatinine = 0.8 mg/dL (0.6-1.1 mg/dL) (SI: 70.7 µmol/L [53.0-97.2 µmol/L])Estimated glomerular filtration rate = 91 mL/min per 1.73 m2 (>60 mL/min per 1.73 m2)TSH = 1.4 mIU/L (0.5-5.0 mIU/L)You plan to perform a routine physical examination and to reassess her overall glycemic control with hemoglobin A 1c  testing.

Which of the following is also indicated now? Screening for distal polyneuropathy using a 10-g monofilament and 128-Hz tuning fork Measurement of the urinary albumin-to-creatinine ratio Repeated comprehensive dilated eye examination Performance of baseline resting electrocardiography Measurement of fasting lipid profile Correct Answer: E Learning objective:Use current recommendations to implement appropriate screening in patients with type 1 diabetes mellitus.

Rationale: This young woman with type 1 diabetes mellitus of short duration is currently at low risk for either microvascular or macrovascular complications of diabetes. The American Diabetes Association recommends that all individuals with diabetes undergo at least annual foot inspection; however, the patient is presently at very low risk for the presence of distal polyneuropathy. Screening for this complication using a 10-g monofilament and 128-Hz tuning fork (Answer A) is indicated 5 years after type 1 diabetes is diagnosed and is thus not required now. Similarly, the patient is presently at low risk for the development of diabetic nephropathy. Therefore, measurement of her urinary albumin-to-creatinine ratio (Answer B) is not yet indicated. As with screening for peripheral neuropathy, screening for nephropathy should begin 5 years after a diagnosis of type 1 diabetes. However, serum creatinine with estimated glomerular filtration rate calculation should be performed annually in all adults with diabetes. Nonpregnant adults with type 1 diabetes should undergo a dilated comprehensive eye examination within 5 years of diabetes diagnosis. If no retinopathy is noted on the first or second examination, repeated examinations at 2-year intervals would be reasonable. Patients with evidence of diabetic retinopathy should have a comprehensive eye examination at least annually. This woman with a 3-year history of type 1 diabetes has undergone 2 such evaluations, and no retinopathy has been detected. Thus, repeated comprehensive dilated eye examination (Answer C) does not represent the best next step in her care. It is important to note that screening recommendations for individuals with type 1 diabetes differ from those for individuals with type 2 diabetes. Persons with type 2 diabetes generally experience years of hyperglycemia before their diabetes diagnosis; therefore, most guidelines recommend that screening for microvascular complications begin when type 2 diabetes is first detected.

Rationale Continued: The likelihood that this patient will experience a cardiovascular event within the next 10 years is quite low because she is young; she has had diabetes for a short duration; and she has no history of cigarette smoking, hypertension, or dyslipidemia. In asymptomatic individuals with diabetes, routine screening for coronary artery disease is not recommended. Therefore, baseline resting electrocardiography (Answer D) is not indicated now. However, the American Diabetes Association does recommend that adults with diabetes undergo measurement of a fasting lipid profile (Answer E). Repeated measurement should take place annually. However, if the patient’s lipid values are found to be within optimal ranges (LDL cholesterol <100 mg/dL [<2.59 mmol/L], HDL cholesterol >50 mg/dL [>1.30 mmol/L], and triglycerides <150 mg/dL [<1.70 mmol/L]), the lipid panel may be performed at 2-year intervals.

Reference(s): American Diabetes Association. 10. Microvascular complications and foot care: standards of medical care in diabetes-2018. Diabetes Care . 2018;41(Suppl 1):S105-S118. PMID: 29222381 Hooper P, Boucher MC, Cruess A, et al. Canadian Ophthalmological Society evidence-based clinical practice guidelines for the management of diabetic retinopathy. Can J Ophthalmol. 2012;47(2 Suppl 1):S1-S30, S31-S54. PMID: 22632804

ITE 2019 Question 45 A 76-year-old woman returns for follow-up of diabetes mellitus. She describes persistent tingling in her feet and legs over the last 4 months. These sensations are distinct from symptoms of numbness in the feet that she has had for the last several years. She also notes unsteadiness, with several near falls when she gets up at night. She describes eating a well-balanced diet with all food groups represented. During her 13-year history of type 2 diabetes, she has been treated with metformin and glipizide. On physical examination, her blood pressure is 126/74 mm Hg and BMI is 28 kg/m2. Cranial nerves II through XII are intact. There is evidence of muscle atrophy of the distal lower extremities with loss of pinprick sensation to the mid shin, decreased vibratory sense in the great toes, and loss of patellar and Achilles reflexes. The Romberg test is positive and the Babinski sign is present. Laboratory test results: Hemoglobin A1c = 7.3% (4.0%-5.6%) (56 mmol/mol [20-38 mmol/mol]) Hemoglobin = 10.7 g/dL (12.1-15.1 g/dL) (SI: 107 g/L [121-151 g/L]) Hematocrit = 35% (35%-45%) (SI: 0.35 [0.35-0.45]) Creatinine = 1.5 mg/dL (0.6-1.1 mg/dL) (SI: 132.6 µmol/L [53.0-97.2 µmol/L])AST = 55 U/L (20-48 U/L) (SI: 0.92 µkat/L [0.33-0.80 µkat/L])ALT = 72 U/L (10-40 U/L) (SI: 1.20 µkat/L [0.17-0.67 µkat/L])Urinalysis, positive for leukocyte esterase and protein

Which of the following is the best next step in this patient’s management? Measurement of serum vitamin B 12 Measurement of serum vitamin B 1 MRI of the lumbar spineElectromyelography and nerve conduction studiesMyelography Correct Answer: A Learning objective : Diagnose and manage adverse effects of metformin.

Rationale: This patient has new-onset neuropathic symptoms and examination findings consistent with peripheral polyneuropathy and posterior column and upper motor neuron disease. Although this could represent progression of her diabetic neuropathy, these findings are not typical for diabetic neuropathy. She has anemia, and the combination of neurologic and hematologic disturbance is compatible with vitamin B12 deficiency. The use of metformin for periods of several years has been associated with lower plasma levels of vitamin B12 in up to 30% of patients and it doubles the risk of clinically significant deficiency. The 2018 American Diabetes Association standards recommend periodic measurement of B12 in metformin-treated patients. While the mechanism is not entirely clear, most evidence points to interference with food-derived B12 absorption, primarily in the ileum. Replacement with oral or parenteral vitamin B12 is usually successful and precludes discontinuation of metformin. Currently, there is no consensus on the best test for diagnosing B12 deficiency; however, numerous laboratory options are available to assess vitamin B12 levels. Some, such as serum vitamin B12 measurement (Answer A), are most commonly used for initial assessment, while others, such as the tests that access levels of holotranscobalamin , methylmalonic acid, or homocysteine, are reserved for confirmatory testing. Electromyelography and nerve conduction studies (Answer D) would most likely be abnormal in this woman, but they are not specific for subacute combined degeneration (neuropathy due to vitamin B12 deficiency). Although vitamin B1 (Answer B) is necessary for proper nerve function, it is present in plant sources unlike vitamin B12; therefore, B1 deficiency is less likely in this patient. Her presentation is not consistent with spinal or nerve root irritation, which can manifest as pain and numbness or weakness radiating down to the legs. Thus, MRI of the lumbar spine (Answer C) is incorrect. Similarly, myelography (Answer E), an older technique occasionally still used in certain clinical scenarios to detect spinal pathology, is also incorrect.

Reference(s): American Diabetes Association. 8. Pharmacologic approaches to glycemic treatment: standards of medical care in diabetes-2018. Diabetes Care . 2018;41(Suppl 1):S73-S85. PMID: 29222379 Pierce SA, Chung AH, Black KK. Evaluation of vitamin B12 monitoring in a veteran population on long-term, high-dose metformin therapy. Ann Pharmacother. 2012;46(11):1470-1476. PMID: 23115224Pflipsen MC, Oh RC, Saguil A, Seehusen DA, Seaquist D, Topolski R. The prevalence of vitamin B(12) deficiency in patients with type 2 diabetes: a cross-sectional study. J Am Board Fam Med. 2009;22(5):528-534. PMID: 19734399Ting RZ, Szeto CC, Chan MH, Ma KK, Chow KM. Risk factors of vitamin B(12) deficiency in patients receiving metformin. Arch Intern Med. 2006;166(18):1975-1979. PMID: 17030830Ward PC. Modern approaches to the investigation of vitamin B12 deficiency. Clin Lab Med. 2002;22(2):435-445. PMID: 12134470

ITE 2019 Question 49 A 52-year-old woman recently completed a successful course of pegylated-interferon-α and ribavirin therapy for chronic hepatitis C virus infection. Approximately 1 month after this treatment was completed, she presented to the emergency department with weight loss, fatigue, polydipsia, and polyuria. Diabetic ketoacidosis was diagnosed. She was admitted to the hospital and with appropriate management the ketoacidosis has quickly resolved. Over the past 10 hours, her glycemic control has been excellent on insulin infusion at a rate of 0.5 units per hour, and no adjustments to her insulin infusion rate have been required. The patient will begin eating solid food later this morning. Her renal function and serum electrolytes are now within normal limits. The patient has no personal or family history of diabetes. Laboratory testing for glutamic acid decarboxylase autoantibodies was negative when performed at the start of her course of antiviral therapy. She tolerated that therapy well, had an excellent response in her viral load, and was without evidence of hyperglycemia during treatment. The patient is afebrile and has normal physical examination findings. Her height is 66 in (167.6 cm), and weight is 160 lb (72.7 kg) (BMI = 25.8 kg/m2).

Which of the following is the best management plan for this patient? Transition from intravenous insulin infusion to a scheduled subcutaneous basal-bolus insulin regimen Transition from intravenous insulin infusion to twice-daily metformin therapy Discontinue all intravenous insulin infusion and continue to monitor blood glucose values Begin high-dosage glucocorticoid therapy and continue the intravenous insulin infusion Transition from intravenous insulin to sulfonylurea therapy Correct Answer: A Learning objective : Manage new-onset diabetes in a patient with interferon-treated hepatitis C infection.

Rationale: Interferon-α therapy for chronic hepatitis C infection and other conditions is associated with the development of pancreatic islet-cell autoantibodies and autoimmune diabetes. The onset of interferon-related type 1 diabetes has been reported to occur as early as 10 days and as long as 4 years after medication initiation and may occur subsequent to interferon discontinuation. Individuals with certain HLA haplotypes including DRB1*0405–DQB1*0401, DRB1*0802–DQB1*0302, and DRB1*0901–DQB1*0303 appear to be at increased risk of this complication. Patients receiving interferon should, at a minimum, have their glucose levels closely monitored. Some groups also recommend that islet-cell autoantibodies be assessed before and during interferon therapy to identify patients at increased risk for type 1 diabetes. This patient’s presentation with diabetic ketoacidosis occurring shortly after the discontinuation of interferon is consistent with interferon-related type 1 diabetes. Much less commonly, interferon therapy may be associated with hyperglycemia due to the development of anti-insulin receptor antibodies. This appears to be an extremely rare complication associated with either hypoglycemia or hyperglycemia and extreme insulin resistance. 

Rationale Continued: Because the patient has most likely developed new-onset type 1 diabetes, her hyperglycemia should be managed with transition from intravenous insulin infusion to a scheduled subcutaneous basal-bolus insulin regimen (Answer A). Given the high likelihood that she has type 1 rather than type 2 diabetes, a transition to metformin therapy (Answer B) or sulfonylurea therapy (Answer E) would not be a reasonable next step. Her relatively low insulin requirements after resolution of ketoacidosis further support a diagnosis of type 1, rather than type 2, diabetes. Measurement of pancreatic islet-cell autoantibodies would also be clinically helpful in confirming the diagnosis. A plan to observe the patient following discontinuation of antihyperglycemic therapy (Answer C) is not an appropriate choice because this would place the patient at risk for recurrent ketoacidosis. In general, it is recommended that interferon therapy be discontinued in patients who develop type 1 diabetes. In some cases, diabetes resolves following this intervention. However, there is no evidence that high-dosage glucocorticoid therapy (Answer D) will provide an additional clinical benefit.  

Reference(s): Popescu C, Popescu GA, Arama V. Type 1 diabetes mellitus with dual autoimmune mechanism related to pegylated interferon and ribavirin treatment for chronic HCV hepatitis. J Gastrointestin Liver Dis . 2013;22(1):101-104. PMID: 23539399Nakamura K, Kawasaki E, Imagawa A, et al; Research Committee on Type 1 Diabetes of the Japan Diabetes Society. Type 1 diabetes and interferon therapy: a nationwide survey in Japan. Diabetes Care. 2011;34(9):2084-2089. PMID: 21775762Noto H, Raskin P. Hepatitis C infection and diabetes. J Diabetes Complications. 2006;20(2):113-120. PMID: 16504840

ITE 2019 Question 52 A 22-year-old woman with type 1 diabetes mellitus is 7 weeks pregnant. Her most recent hemoglobin A1c value is 6.9% (52 mmol/mol). She is taking insulin detemir, 8 units in the morning and 12 units in the evening, in addition to prandial doses of insulin lispro based on an insulin-to-carbohydrate ratio of 1:10 and a sensitivity (or correction) factor of 1:40. Her overnight (3 AM) and fasting blood glucose levels range between 110 and 122 mg/dL (6.1-6.8 mmol/L), and her peak (1-hour) postprandial glucose levels range between 112 and 129 mg/dL (6.2-7.2 mmol/L).

Which of the following should you recommend during this pregnancy? Continue same regimen Change her insulin-to-carbohydrate ratio to 1:8 Increase morning insulin detemir dose to 12 units Increase evening insulin detemir dose to 14 units Change her sensitivity (or correction) factor to 1:30 Correct Answer: D Learning objective:Make basic recommendations for management of glycemia during pregnancy.

Rationale: During pregnancy, normalization of blood glucose levels reduces the risk of congenital malformations during the first 8 to 10 weeks of pregnancy and the risk of macrosomia and related comorbidities over the course of the pregnancy. The 2018 American Diabetes Association standards recommend that the hemoglobin A1c level be less than 6.0% to 6.5% (<42-48 mmol/mol) during early pregnancy, with a lower target level (<6.0% [<42 mmol/mol]) by the second and third trimesters if possible, as this decreases the risk of large-for-gestational-age infants. The American Diabetes Association also recommends maintaining fasting and preprandial glucose levels ≤95 mg/dL (≤5.0 mmol/L), peak (1-hour) postprandial levels ≤140 mg/dL (≤7.2-7.8 mmol/L), and 2-hour postprandial levels ≤120 mg/dL (≤6.7 mmol/L). Because this patient’s fasting blood glucose levels are higher than 95 mg/dL (>5.3 mmol/L), increasing the evening insulin detemir dose (Answer D) is the best course of action. Continuing the same regimen (Answer A) would not improve her high fasting blood glucose. Given that her peak postprandial glucose measurements are within the target range, changing her insulin-to-carbohydrate ratio (Answer B) is not indicated. Changing the morning insulin detemir dose (Answer C) would affect her evening blood glucose values, which are within range. Changing the sensitivity (or correction) factor (Answer E) would not affect her high fasting glucose levels.

Reference(s): American Diabetes Association. 13. Management of diabetes in pregnancy: standards of medical care in diabetes-2018. Diabetes Care . 2018;41(Suppl 1):S137-S143. PMID: 29222384 McCance DR. Pregnancy and diabetes. Best Pract Res Clin Endocrinol Metab. 2011;25(6):945-958. PMID: Kitzmiller JL, Block JM, Brown22115168 FM, et al. Managing preexisting diabetes for pregnancy: summary of evidence and consensus recommendations for care. Diabetes Care. 2008;31(5):1060-1079. PMID: 18445730

ITE 2019 Question 58 A 41-year-old woman with a 33-year history of type 1 diabetes mellitus complicated by end-stage kidney disease, proliferative retinopathy, and peripheral neuropathy underwent kidney-pancreas transplant 17 months ago due to severe hypoglycemia with unawareness. Since surgery, she has been treated with prednisone and the dosage was tapered to 5 mg daily 6 months after the transplant. She has been off insulin since 7 days after transplant. Her hemoglobin A1c level was 6.0% (42 mmol/mol) 6 months after transplant and 6.3% (45 mmol/mol) 12 months after transplant. Her medications include prednisone, tacrolimus, mycophenolate pregabalin, fosinopril , metoprolol, fluoxetine, and atorvastatin. On physical examination, her height is 67 in (170 cm) and weight is 153 lb (69.5 kg) (BMI = 24.0 kg/m2). Her blood pressure is 128/79 mm Hg, and pulse rate is 74 beats/min. On eye examination, laser scars are evident. She has reduced sensation to 10-g monofilament testing and reduced vibrational sense in each foot. The ankle and patellar reflexes are blunted.

Laboratory test results: Hemoglobin A 1c  = 7.3% (4.0%-5.6%) (56 mmol/mol [20-38 mmol/mol]) Fasting glucose = 139 mg/dL (<70-99 mg/dL) (SI: 7.7 mmol/L [3.9-5.5 mmol/L]) C-peptide = 1.9 ng/mL (0.9-4.3 ng/mL) (SI: 0.63 nmol/L [0.30-1.42 nmol/L]) Electrolytes, normal Creatinine = 1.5 mg/dL (0.6-1.1 mg/dL) (SI: 132.6 µmol/L [53.0-97.2 µmol/L])Estimated glomerular filtration rate = 53 mL/min per 1.73 m2 (>60 mL/min per 1.73 m2)Amylase = 56 U/L (26-102 U/L) (SI: 0.94 µkat/L [0.43-1.70 µkat/L])

Which of the following is the most likely explanation for her elevated fasting glucose and hemoglobin A 1c ? Pancreas rejection Transient hyperglycemia after transplant Posttransplant diabetes mellitus Autoimmune β-cell destruction (recurrence of type 1 diabetes)Correct Answer: C Learning objective : Diagnose posttransplant diabetes mellitus and summarize recommendations for appropriate treatment.

Rationale: Posttransplant diabetes mellitus (PTDM) (Answer C) is relatively common in patients following kidney and pancreas transplant. PTDM should only be diagnosed in patients who are on a maintenance immunosuppressive regimen, who have stable renal allograft function, and who do not have acute infection. Hyperglycemia must be persistent after transplant, and PTDM is diagnosed regardless of whether the patient had diabetes before undergoing transplant. Most reports on the outcomes of pancreas transplant rely on registry data, and definitions of graft failure differ from center to center. The 5-year adjusted pancreas graft survival rate for simultaneous pancreas and kidney recipients in the United States is 73%. The national 5-year pancreas graft survival in the United Kingdom is 75% (reported as insulin-independence). The following factors increase the risk for development of PTDM: older age (>40 years), obesity, history of diabetes mellitus, family history of diabetes, and certain immunosuppressive medications. The standard World Health Organization and American Diabetes Association definitions for the diagnosis of diabetes mellitus after transplant apply. An additional definition for new-onset diabetes after transplant is an absence of measured C-peptide. PTDM has been reported with glucocorticoid immunosuppression since the early days of kidney transplant. Glucocorticoids lead to weight gain and increased risk of the metabolic syndrome. Glucocorticoids promote hepatic glucose output and impairment of glucose uptake by adipocytes. Calcineurin inhibitors such as cyclosporine and tacrolimus also adversely affect β-cell function. These drugs lead to impairment in insulin production and secretion. Patients on modern-day immunosuppressive regimens following transplant generally develop insulin resistance over time.

Rationale Continued: The patient in this vignette is 17 months out from kidney-pancreas transplant and is on a stable immunosuppressive regimen. There is mild renal impairment, which is typical following transplant. She has no signs of pancreas rejection (Answer A), as the amylase level is in the normal range. Transient hyperglycemia after transplant occurs in up to 90% of patients in the first 6 weeks after transplant. Hyperglycemia typically resolves and most patients revert to near-normoglycemia during the first 6 months after surgery. This patient had evidence of impaired glucose tolerance 6 and 12 months after transplant, which is quite common, and this increases the risk for eventual development of overt diabetes. Due to the length of time since the transplant occurred, this does not represent transient hyperglycemia after transplant (Answer B). Recurrence of type 1 diabetes (Answer D) is possible following kidney-pancreas transplant, but this is unlikely. In a recent study, only 8% of patients with a history of type 1 diabetes developed recurrence of type 1 diabetes in the pancreas allograft. In this vignette, the C-peptide is in the normal range and there has been no evidence of autoimmune destruction of the pancreas graft. Therefore, this is unlikely to be the cause of the hyperglycemia in this vignette. The cause of the hyperglycemia in this case is PTDM. Treatment of PTDM is similar to the approach in nontransplant patients with diabetes. She should be referred to a nutritionist to review a diet treatment plan, with recommendations on losing weight and exercising. A decrease in the prednisone and/or tacrolimus dosage could be considered, but this must be balanced against the increased risk of graft rejection. Oral antihyperglycemic agents, including metformin, insulin secretogogues , and dipeptidyl-peptidase 4 inhibitors, can be used to treat PTDM. Unless the patient has marked hyperglycemia or requires hospitalization, insulin treatment can often be delayed as long as C-peptide is detectable.

Reference(s): Sharif A, Hecking M, de Vries AP, et al. Proceedings from an international consensus meeting of posttranplantation diabetes mellitus: recommendations and future directions. Am J Transplant . 2014;14:1992-2000. PMID: 25307034Dean PG, Kukla A, Stegall MD, Kudva YC. Pancreas transplantation. BMJ. 2017;357:j1321. PMID: 28373161Vendrame F, Hopfner Y-Y, Diamantopoulos S, et al. Risk factors for type 1 diabetes recurrence in immunosuppressed recipients of simultaneous pancreas-kidney transplants. Am J Transplant. 2016;16(1):235-245. PMID: 26317167Strom Halden TA, Asberg A, Vik K, Hartmann A, Jenssen T. Short-term efficacy and safety of sitagliptin treatment in long-term stable renal recipients with new-onset diabetes after transplantation. Nephrol Dial Transplant. 2014;29(4):926-933. PMID: 24452849

ITE 2019 Question 60 A 67-year-old woman with a 13-year history of type 2 diabetes mellitus, as well as hypertension, hyperlipidemia, and osteoporosis, presents for follow-up. She has no known diabetes-related complications. Her current medications are metformin, 1000 mg twice daily; lisinopril; atorvastatin; and alendronate. She does not smoke cigarettes, but does report drinking 1 to 2 glasses of red wine daily. Her mother has diabetes and osteoporosis, and her father has diabetes and coronary artery disease. On physical examination, her height is 66 in (167.5 cm) and weight is 192 lb (82.5 kg) (BMI = 31.0 kg/m2). Her blood pressure is 136/64 mm Hg. She has acanthosis nigricans on her neck and under her arms. Laboratory test results: Hemoglobin A1c = 8.6% (4.0%-5.6%) (70 mmol/mol [20-38 mmol/mol]) Creatinine = 1.1 mg/dL (0.6-1.1 mg/dL) (SI: 97.2 µmol/L [53.0-97.2 µmol/L])25-Hydroxyvitamin D = 31 ng/mL (30-80 ng/mL [optimal]) (SI: 77.4 µmol/L [74.9-199.7 µmol/L])Calcium = 9.1 mg/dL (8.2-10.2 mg/dL) (SI: 2.3 µmol/L [2.1-2.6 µmol/L])You counsel her to limit alcohol consumption and to increase weight-bearing exercise.

Which of the following medications should be added as the best next step in this patient’s diabetes management? Canagliflozin Liraglutide Sitagliptin Pioglitazone Glipizide Correct Answer: B Learning objective:Recommend the best treatment approach for type 2 diabetes mellitus in a patient with osteoporosis.

Rationale: Type 2 diabetes mellitus is associated with an increased risk of fractures. Direct effect of hyperglycemia on bone metabolism, effects of medications used for blood glucose management, and diabetes complications leading to increased risk of falls are all thought to contribute to this increased risk. As in the general population, there is an inverse correlation between bone mineral density and fracture risk in patients with diabetes. The FRAX score appears to underestimate the fracture risk in patients with type 2 diabetes. A recent clinical practice guideline reviews medications and surgical intervention and effect on bone mineral density and fracture risk, although data in this area are limited. Observational studies suggest that metformin may have a beneficial effect on bone mineral density and fracture risk. In the Osteoporotic Fractures in Men study, older men with type 2 diabetes who were taking sulfonylureas had an increased fracture risk; however, other studies have shown beneficial or neutral effects on fracture risk and neutral effects on bone mineral density. The authors do note that sulfonylureas (Answer E) and insulin must be used with caution to avoid hypoglycemia, falls, and fractures. Thiazolidinediones (Answer D) reduce bone mineral density and increase fracture risk in women and should be avoided in those at high fracture risk. Canagliflozin (Answer A) appears to have a negative effect on bone mineral density and fracture risk. Neither DPP-4 inhibitors (Answer C) nor GLP-1 receptor agonists (Answer B) have negative effects on bone mineral density or fracture, and both may be a reasonable choice from a bone health perspective. However, the hemoglobin A1c reduction required in this patient exceeds the 0.5% expected from use of DPP-4 inhibitors. Therefore, liraglutide is the better choice.

Reference(s): Paschou SA, Dede AD, Anagnostis PG, Vryonidou A, Morganstein D, Goulis DG. Type 2 diabetes and osteoporosis: a guide to optimal management. J Clin Endocrinol Metab. 2017;102(10):3621-3634. PMID: 28938433Giangregorio LM, Leslie WD, Lix LM, et al. FRAX underestimates fracture risk in patients with diabetes. J Bone Miner Res. 2012;27(2):301-308. PMID: 22052532

ITE 2017 Question 68 A 32-year-old woman with a 10-year history of type 1 diabetes mellitus is referred for a second opinion regarding her unexplained high hemoglobin A1c levels. Her diabetes is managed with insulin pump therapy. Self-monitoring of blood glucose performed 8 to 12 times daily shows values ranging between 80 and 110 mg/dL (4.4-6.1 mmol/L) before meals and between 100 and 130 mg/dL (5.6-7.2 mmol/L) 2 hours after meals. She does not report hypoglycemic episodes. Her review of systems is notable for recent fatigue and lightheadedness. Her periods have been regular but very heavy for the past year. Her last menstrual period was 3 weeks ago. Her medications include aspirin and ramipril. Her hemoglobin A1c level had been in the range of 5.5% to 6.2% (37-44 mmol/mol) for years, but 4 months ago it was 7.8% (62 mmol/mol) and a recent value was 8.2% (66 mmol/mol). Laboratory test results: Serum creatinine = 1.1 mg/dL (0.6-1.1 mg/dL) (SI: 97.2 µmol/L [53.0-97.2 µmol/L]) Liver enzymes, normal TSH, normal Urine albumin-to-creatinine ratio = 95 mg/g (<30 mg/g)

Which of the following is most likely the cause of her high hemoglobin A1c levels? Nocturnal hyperglycemia Laboratory error Hemolysis Pregnancy Iron deficiency Correct Answer: E Learning objective:Identify iron deficiency anemia as a cause of falsely high hemoglobin A1c values.

Rationale: Although the international standardization of the hemoglobin A1c assay has decreased potential technical errors in interpreting results, there are other biologic and patient-specific factors that may cause misleading results.Hemoglobin A1c values are influenced by red blood cell survival. Thus, falsely high values in relation to mean blood glucose values can be obtained when red blood cell turnover is low, resulting in a disproportionate number of older red cells. This problem can occur in patients with iron, vitamin B12, or folate deficiency anemia (thus, Answer E is correct).In contrast, rapid red blood cell turnover leads to a greater proportion of younger red cells and falsely low hemoglobin A1c values. Examples include patients with hemolysis (Answer C); patients treated for deficiencies of iron, vitamin B12, or folate; and patients treated with erythropoietin. It should be noted that hemoglobin A1c values tend to be lower in pregnancy (Answer D) because the average blood glucose concentration is about 20% lower in pregnant women than in nonpregnant women. In the first half of pregnancy, there is a rise in red cell mass and a slight increase in red blood cell turnover. Laboratory error (Answer B) is unlikely to happen twice—both of the patient’s hemoglobin A1c values were high at 7.8% (62 mmol/mol) and 8.2% (66 mmol/mol). High nighttime blood glucose values (Answer A) would not likely occur suddenly and without a rise in fasting blood glucose.

Reference(s): National Glycohemoglobin Standardization Program (NGSP) Web site. Factors that interfere with HbA1c test results . Available at: http://www.ngsp.org/factors.asp . Accessed for verification October 2018.Ahmad J, Rafat D. HbA1c and iron deficiency: a review. Diabetes Metab Syndr. 2013;7(2):118-122. PMID: 23680254Silva JF, Pimentel AL, Camargo JL. Effect of iron deficiency anaemia on HbA1c levels is dependent on the degree of anaemia. Clin Biochem. 2016;49(1):117-120. PMID: 26365695

ITE 2019 Question 70 A 45-year-old woman with a 20-year history of type 1 diabetes mellitus that has been complicated by nephropathy and retinopathy is referred for help achieving better glycemic control. Her current regimen consists of insulin glargine once daily and insulin lispro with meals. Self-monitoring of blood glucose (8 times daily) shows values ranging between 150 and 300 mg/dL (8.3-16.7 mmol/L). Her hemoglobin A1c level has been between 8.5% and 10.0% (69-86 mmol/mol). Her menses are regular. She has sickle cell disease without recent crises, but concern about the hemoglobinopathy prompted measurement of a fructosamine along with her routine laboratory tests. Laboratory test results: Hemoglobin A1c = 9.0% (4.0%-5.6%) (75 mmol/mol [20-38 mmol/mol]) Serum creatinine = 2.2 mg/dL (0.6-1.1 mg/dL) (SI: 194.5 µmol/L [53.0-97.2 µmol/L]) Urine albumin-to-creatinine ratio = 3886 mg/g (<30 mg/g)Liver function, normalTSH = 7.5 mIU/L (0.5-5.0 mIU/L)Serum fructosamine = 210 µmol/L (205-285 µmol/L)

The discrepancy between this patient’s hemoglobin A1c and fructosamine levels is most likely caused by which of the following? Laboratory error Sickle cell disease Hemolysis Hypothyroidism ProteinuriaCorrect Answer: ELearning objective:Identify nephrotic syndrome as a cause of falsely low fructosamine .

Rationale: The turnover of serum proteins, mainly albumin, is more rapid than that of hemoglobin; thus, serum fructosamine (glycated proteins, mostly albumin) values reflect mean blood glucose values over a much shorter period (1 to 2 weeks). There is generally a good correlation between serum fructosamine and hemoglobin A1c values. Fructosamine responds more rapidly with changes in blood glucose control than does hemoglobin A1c. Falsely low fructosamine values in relation to mean blood glucose values occur with rapid albumin turnover, for example, in patients with protein-losing enteropathy or nephrotic syndrome (Answer E).Sickle cell disease (Answer B), hemolysis (Answer C), and hypothyroidism (Answer D) do not falsely lower fructosamine. Laboratory errors (Answer A) do rarely occur, but in this case, the heavy proteinuria explains the normal fructosamine value.

Reference(s): Vetter SW. Glycated serum albumin and AGE receptors. Adv Clin Chem . 2015;72:205-275. PMID: 26471084 Koga M. Glycated albumin; clinical usefulness. Clin Chim Acta. 2014;433:96-104. PMID: 24631132Parrinello CM, Selvin E. Beyond HbA1c and glucose: the role of nontraditional glycemic markers in diabetes diagnosis, prognosis, and management. Curr Diab Rep. 2014;14(11):548. PMID: 25249070

ITE 2019 Question 71 An 18-year-old woman is seen for the recent development of erratic blood glucose values. Type 1 diabetes mellitus was diagnosed 2 years ago, and a regimen of once-daily insulin glargine and mealtime insulin lispro was initiated. Her glycemic control has always been adequate, with hemoglobin A1c values around 7.0% (53 mmol/mol). However, a few weeks ago, she started to notice recurrent symptomatic hypoglycemic episodes (blood glucose values between 45 and 60 mg/dL [2.5-3.3 mmol/L]) that occur mainly after meals. Despite eating more snacks to prevent hypoglycemia, she has lost 4 lb (1.8 kg) in the past 2 weeks. She has no gastrointestinal complaints or lightheadedness. Her menses are regular. Her BMI is 22 kg/m2, and blood pressure is 125/80 mm Hg. Findings on thyroid and abdominal examinations are unremarkable. No hyperpigmentation is noted. Laboratory test results: Hemoglobin A1c = 6.7% (4.0%-5.6%) (50 mmol/mol [20-38 mmol/mol]) Electrolytes, normalCreatinine, normalLiver function, normalComplete blood cell count, normal

An elevation in which of the following would most likely identify the underlying cause of her hypoglycemia? ACTH Free T 4 Tissue transglutaminase IgA antibodies 21-Hydroxylase antibodies Glutamic acid decarboxylase 65 antibodies Correct Answer: CLearning objective : Describe the association of celiac disease with type 1 diabetes mellitus and recognize its presentation.

Rationale: About 5% of persons with type 1 diabetes develop celiac disease. Only a minority of children and adolescents with type 1 diabetes and celiac disease present with gastrointestinal symptoms. More common initial findings include unpredictable blood glucose measurements and recurrent episodes of hypoglycemia because of erratic intestinal absorption of nutrients. Thus, elevated tissue transglutaminase IgA antibodies (Answer C) would most likely explain her hypoglycemia. Less than 1% of children with type 1 diabetes have autoimmune adrenalitis (Addison disease). Thus, measuring ACTH (Answer A) is unlikely to be helpful. In one report, about 2% of children with type 1 disease had circulating antibodies to steroid 21-hydroxylase (Answer D). Although less common than celiac disease, this condition is associated with decreased insulin requirement and increased frequency of hypoglycemia, as well as hyperpigmentation, hypotension, hyponatremia, and hyperkalemia (none of which is present in this patient). Hyperthyroidism is rare in patients with type 1 diabetes (1%-2%) and can lead to higher blood glucose values due to insulin resistance, not hypoglycemia. Thus, measuring free T4 (Answer B) is incorrect. In addition, except for weight loss, this patient has no symptoms or physical findings to suggest Graves disease. Antibodies to glutamic acid decarboxylase (a 65-kD protein) (Answer E) are found in about 70% of patients with type 1 diabetes at the time of diagnosis. They could very well be high in this patient, but this would not explain or be the cause of her hypoglycemia.

Reference(s): Khoury N, Semenkovich K, Arbeláez AM. Coeliac disease presenting as severe hypoglycaemia in youth with type 1 diabetes. Diabet Med. 2014;31(12):e33-e36. PMID: 24805141Abid N, McGlone O, Cardwell C, McCallion W, Carson D. Clinical and metabolic effects of gluten free diet in children with type 1 diabetes and coeliac disease. Pediatr Diabetes. 2011;12(4 Pt 1):322-325. PMID: 21615651Warncke K, Fröhlich-Reiterer EE, Thon A, Hofer SE, Wiemann D, Holl RW; DPV Initiative of the German Working Group for Pediatric Diabetology; German BMBF Competence Network for Diabetes Mellitus. Polyendocrinopathy in children, adolescents, and young adults with type 1 diabetes: a multicenter analysis of 28,671 patients from the German/Austrian DPV-Wiss database. Diabetes Care. 2010;33(9):2010-2012. PMID: 20551013

ITE 2019 Question 74 A 43-year-old man with an 18-year history of type 1 diabetes mellitus complicated by retinopathy, nephropathy, and gastroparesis schedules a visit to discuss evening hypoglycemia. He has been managed on an insulin pump for the past 2 years with overall improved control. He is using insulin aspart in his pump and reports no problems related to use of the device or with his infusion set sites. He has received education and is confident regarding estimation of his carbohydrate intake at meals. He reports consistently using his insulin pump bolus calculator to estimate his mealtime and correction insulin doses, which he administers at the start of meals. He reports several episodes of hypoglycemia that have occurred within 1 hour of bolusing for an evening meal. His appetite has been good and he denies postprandial nausea or bloating. He is eating a low-residue diet but notes that his evening meals are larger and higher in fat than the meals eaten earlier in the day.

His current insulin pump settings are as follows: Basal Rates Midnight:         1.0 units/h 6 AM:               0.8 units/h 6 PM:               1.0 units/h Insulin-to-Carbohydrate Ratio Midnight:         1 unit per 14 g carbohydrate 6 AM               1 unit per 12 g carbohydrate6 PM:               1 unit per 10 g carbohydrateAverage total daily insulin dose: 45 units; 48% basal, 52% bolusBlood glucose target: 120-140 mg/dL mg/dL (6.7-7.8 mmol/L)Insulin sensitivity (correction) factor: 40 mg/dL (2.2 mmol/L)Duration of insulin action: 5 hoursLaboratory test results:Hemoglobin A1c = 7.9% (4.0%-5.6%) (63 mmol/mol [20-38 mmol/mol]) Creatinine = 1.2 mg/dL (0.7-1.3 mg/dL) (SI: 106.1 µmol/L [61.9-114.96 µmol/L])

You review the results of a continuously monitored glucose profile showing the following results for 4 days of use:

Which of the following should you recommend now to address his glycemic instability in the evening and overnight hours? Extend evening-meal bolus delivery over a 4-hour period Increase to 1 unit of insulin per 8 g carbohydrate at 6 PM Administer mealtime bolus 1 hour after the start of evening meals Increase the midnight basal rate to 1.2 units/h Increase the 6-PM basal rate to 1.2 units/h Correct Answer: A Learning objective:Use the results of continuous glucose monitoring to adjust bolus insulin delivery.

Rationale: Although this patient’s glycemic control has improved since initiation of insulin pump therapy 2 years ago, as noted in his history, he has struggled to achieve adequate glycemic control in the evening and during the overnight hours. Review of the patient’s glucose sensor tracing recorded over a 4-day period is quite revealing. On 3 of the days recorded, the patient’s blood glucose drops shortly after delivery of the bolus administered to cover his evening meal. These episodes are followed by hyperglycemia late in the evening, which the patient addresses through the delivery of correction insulin.  The patient’s medical history and reported meal composition also provide information critical to his management. Individuals with gastroparesis experience delayed gastric emptying and absorption of nutrients following food intake. Compared with individuals without gastroparesis, affected persons often have a prolonged time to postmeal peak blood glucose and significantly higher blood glucose values many hours after meals. The delay in carbohydrate absorption may be most pronounced when meals are large and/or high in fat. As the patient reports ingesting meals of this type in the evening, this may certainly have contributed to his glycemic instability primarily following those meals.

Rationale Continued: Extension of the patient’s evening-meal bolus delivery over a 4-hour period (Answer A) is the best choice of the listed options to address his delayed digestion of nutrients. Delivery of insulin over an extended period will better synchronize mealtime insulin delivery with his absorption of carbohydrates. Although not listed as an option, alteration in the patient’s meal composition may also be helpful in addressing his postmeal glycemic instability. Simply increasing his bolus amount to 1 unit of insulin per 8 g carbohydrate at 6 PM (Answer B) will most likely increase his risk of hypoglycemia shortly after bolus administration. Delaying mealtime bolus delivery to 1 hour after the start of evening meals (Answer C) is often recommended for individuals with gastroparesis who experience early satiety or emesis after eating. However, that is not a problem for this patient and thus this modification does not represent the best intervention. Increases in the patient’s midnight (Answer D) or 6-PM (Answer E) basal rates would also not be the preferred next steps, as these interventions do not address his known risk for postbolus hypoglycemia. 

Reference(s): Ramzan Z, Duffy F, Gomez J, Fisher RS, Parkman HP. Continuous glucose monitoring in gastroparesis. Dig Dis Sci . 2011;56(9):2646-2655. PMID: 21735078 Shetty G, Wolpert H. Insulin pump use in adults with type 1 diabetes--practical issues. Diabetes Technol Ther. 2010;12(Suppl 1):S11-S16. PMID: 20515299

ITE 2019 Question 77 A 32-year-old woman in her second trimester of pregnancy presents to the ophthalmologist for a dilated eye examination. She has no known history of diabetic ophthalmopathy or any other diabetes-associated complications. The left image shows a picture of her dilated fundus from an examination several years ago, and the right image shows a picture from her current examination. Left                                     Right

The images shown depict which of the following: Correct Answer: B. B Learning objective : Characterize diabetic retinopathy and explain the effects of pregnancy on retinopathy progression. Answer Left Image Right Image A. Normal retina Microaneurysms B. Normal retina Severe nonproliferative diabetic retinopathy with retinal hemorrhages and hard exudates C. Nonproliferative diabetic retinopathy    Proliferative diabetic retinopathy D. Nonproliferative diabetic retinopathy Proliferative retinopathy with laser scars E. Soft exudates Hard exudates

Rationale: Pregnancy can cause clinically significant progression of retinopathy, both in patients with and without preexisting retinopathy. In the Diabetes Control and Complications Trial (DCCT), the likelihood of worsening retinopathy was much greater during pregnancy and the first year postpartum as compared with the outcomes of women who did not become pregnant during the study. However, despite the short-term risks, the long-term risk of progression, as observed in the DCCT, was not significantly different between women who did or did not become pregnant.   In addition to the importance of glycemic control as a contributing factor to retinopathy, lower retinal blood flow during pregnancy can exacerbate retinal ischemia and hypoxia, thereby causing progression. In addition, changes in hormones, growth factors, and systemic hemodynamics (such as a fall in systemic blood pressure) during pregnancy may also contribute. Given these risks, counseling women about the need for close monitoring during pregnancy and for up to 1 year postpartum is important. Diabetic retinopathy is divided into 2 main forms: nonproliferative and proliferative, named for the absence or presence of abnormal new blood vessels growing on the retina. Macular edema, characterized by retinal thickening and edema of the macula, can occur at any stage of diabetic retinopathy. 

Rationale Continued: Diabetic retinopathy can also be classified into 4 stages: Stage Description Mild nonproliferative retinopath Characterized by the presence of microaneurysms, which are small areas of balloon-like swelling in the retina's tiny blood vessels. Moderate nonproliferative retinopathy Progression to this stage causes some blood vessel flow to be obstructed. Severe nonproliferative retinopathy Characterized by several retinal hemorrhages all over the fundus, venous dilatation, cotton wool spots (soft exudates), and intraretinal microvascular abnormalities that are dilated shunt vessels carrying blood and bypassing small areas of capillary occlusion/nonperfusion. Proliferative retinopathy Progression to this stage triggers the growth of new blood vessels. These new blood vessels are abnormal and fragile, and they grow along the retina and along the surface of the clear, vitreous gel that fills the inside of the eye. By themselves, these blood vessels do not cause symptoms or vision loss. However, they have thin, fragile walls, which can cause severe vision loss and even blindness if they rupture.

Rationale Continued: The left image in this vignette shows a normal fundus revealing the bright yellow disc (called the optic disc) and the optic cup within it (thus, Answers C, D, and E are incorrect). Normal retinal veins are thin-walled, so they appear darker on imaging, and normal retinal arteries are thicker-walled, so they appear lighter. The right image (shown here) demonstrates nonproliferative diabetic retinopathy with retinal hemorrhages and hard exudates. The red areas show presence of retinal hemorrhage (white arrow) and the yellow areas are indicative of hard exudates, which are lipid deposits (black arrow). Thus, Answer B best characterizes the images.

Rationale Continued: Microaneurysms (Answer A) are balloon-like dilatations seen on retinal blood vessels and they are not present on the indicated image. Proliferative diabetic retinopathy (Answer C) is characterized by the presence of new abnormal blood vessels branching out from previously existing retinal vessels, which is not visualized in these images. Proliferative retinopathy with laser scars (Answer D) is associated with abnormal vasculature and multiple pigmented scars from panretinal laser therapy, none of which is present in these images.

Reference(s): Chew EY, Mills JL, Metzger BE, et al. Metabolic control and progression of retinopathy. The Diabetes in Early Pregnancy Study. National Institute of Child Health and Human Development Diabetes in Early Pregnancy Study. Diabetes Care. 1995;18(5):631-637. PMID: 8586000 ACCORD Study Group; ACCORD Eye Study Group; Chew EY, et al. Effects of medical therapies on retinopathy progression in type 2 diabetes [published corrections appear in N Engl J Med. 2011;364(2):190 and N Engl J Med. 2012;367(25):2458]. N Engl J Med. 2010;363(3):233-234. PMID: 20587587Heath V. Diabetes: progression of diabetic retinopathy found to be a potential risk during pregnancy. Nat Rev Endocrinol. 2010;6(7):354. PMID: 20583339

ITE 2019 Question 83 A 17-year-old woman with a history of pancreatic islet-cell tumors related to multiple endocrine neoplasia type 1 (MEN 1) is referred for evaluation. The diagnosis of MEN 1 was confirmed by genetic testing 7 years ago. Her father was also affected, and he died 2 years ago of metastatic islet-cell tumor. The patient has had multiple episodes of adrenergic symptoms with accompanying confusion over the past 6 months, and hypoglycemia with concomitant elevation of β-cell polypeptides has been documented on 2 occasions. She is known to have multiple tumors with 2 dominant lesions measuring 3.0 and 4.5 cm in the head of the pancreas. Over the past 3 years of observation, there has been a clear size increase in the larger of the 2 tumors. The patient is otherwise in good health with normocalcemia and no previous parathyroid surgeries. Her menses are regular, and she has no galactorrhea.

Which of the following should be recommended to this patient? Selective arterial calcium-stimulation test Enucleation of the lesions in the head of the pancreas An outpatient fast to document hypoglycemia Endoscopic ultrasonography No further testing now Correct Answer: A Learning objective:Manage functional islet-cell tumors in patients with multiple endocrine neoplasia type 1.

Rationale: Current management of multiple islet-cell tumors in the setting of multiple endocrine neoplasia type 1 usually entails an extended distal pancreatectomy and enucleation of remaining lesions where possible. Subsequently, surveillance of the remaining pancreatic head and body can be accomplished with a combination of invasive and noninvasive imaging. However, the timing of such surgery is uncertain. As such, one might argue that in the presence of 2 dominant tumors in the head of the pancreas, one of which is increasing in size, no further testing is indicated before surgery. However, in the presence of symptoms of hypoglycemia, it would be important to regionalize the locus of insulin hypersecretion before embarking on surgery to ensure that this is included in the scope of the resection (thus, Answers B and E are incorrect). Indeed, the patient did not exhibit a response to calcium in the gastroduodenal arterial territory (perfusing the head of the pancreas), while there was a positive response in the splenic artery suggesting the presence of an insulinoma in the tail of the pancreas. Although endoscopic ultrasonography (Answer D) is useful in localizing small islet-cell tumors, it provides no functional information and therefore is unlikely to guide the surgical management. An outpatient fast (Answer C) will not provide additional information in this patient because there are already strong indications to explore and enucleate functional lesions in the pancreas. In these circumstances, a selective arterial calcium-stimulation test (Answer A) will provide functional information ( ie , confirm or refute abnormal pancreatic insulin secretion and help regionalize this source).

Reference(s): O'Riordain DS, O'Brien T, van Heerden JA, Service FJ, Grant CS. Surgical management of insulinoma associated with multiple endocrine neoplasia type I. World J Surg . 1994;18(4):488-493; discussion 493-484. PMID: 7725733 Cryer PE, Axelrod L, Grossman AB, et al; Endocrine Society. Evaluation and management of adult hypoglycemic disorders: an Endocrine Society Clinical Practice Guideline. J Clin Endocrinol Metab. 2009;94(3):709-728. PMID: 19088155Brown CK, Bartlett DL, Doppman JL, et al. Intraarterial calcium stimulation and intraoperative ultrasonography in the localization and resection of insulinomas. Surgery. 1997;122(8):1189-1193; discussion 1193-1194. PMID: 9426437Doppman JL, Miller DL, Chang R, Shawker TH, Gorden P, Norton JA. Insulinomas: localization with selective intraarterial injection of calcium. Radiology. 1991;178(1):237-241. PMID: 1984311

FEMALE REPRODUCTION

ITE 2019 Question 10 A 25-year-old transgender man returns for follow-up after initiating masculinizing testosterone injections 9 months ago. He had regular monthly menses before initiation of testosterone. After 3 months of hormone therapy, menses ceased. However, for the past 2 months, bleeding has recurred intermittently despite no change in his hormone regimen. Laboratory test result (assessment just before the current visit, midway between injections): Total testosterone = 343 ng/dL (300-900 ng/dL [male]; (0.3-1.9 ng/dL [female]) (SI: 11.9 nmol/L [10.4-31.2 nmol/L (male)]; [0.01-0.07 nmol/L (female)] His testosterone dosage is 50 mg subcutaneously weekly. He has already had a gynecologic examination and ultrasonography, and there is no evidence of infection or uterine abnormality. He is not currently sexually active.

Which of the following is the best option to stop the uterine bleeding? Add oral estradiol Add oral progesterone Switch from subcutaneous to intramuscular testosterone therapy Insert a copper intrauterine device Increase the testosterone dosage Correct Answer: E Learning objective:Counsel transgender men taking masculinizing hormone therapy regarding management options for persistent or recurrent uterine bleeding.

Rationale: In this case, masculinizing hormone therapy induced cessation of menstrual bleeding within the expected time frame of the first 6 months on testosterone. Persistent or recurrent uterine bleeding should be evaluated for structural and nonstructural causes similar to the evaluation in a cisgender woman. For transgender patients, consideration of patient comfort and dysphoria might lead to selection of different exam and imaging techniques—for example, choosing transabdominal instead of transvaginal ultrasonography.   Management strategies for persistent or recurrent uterine bleeding in transgender men should also take into account patient preference and tolerance for different hormonal options. In cisgender women, use of oral estrogen (Answer A) can stop abnormal uterine bleeding. However, this option could be used only short term due to the risk of endometrial hyperplasia.   Progesterone (Answer B)—orally, injected, implanted, or by intrauterine system—would be another option to stop bleeding, and the injected or intrauterine device forms would also provide contraceptive benefit for those who are at risk of unintended pregnancy, as ovulation can still occur despite male physiologic testosterone levels. However, it is an additional medication that may not be needed.

Rationale Continued: A levonorgestrel-releasing intrauterine device would be a good option to decrease bleeding and induce amenorrhea, but it was not provided as a choice. The copper intrauterine device (Answer D) would not induce amenorrhea and could be associated with abnormal or heavy uterine bleeding.   Higher testosterone dosages and shorter intervals between doses are correlated with faster time to amenorrhea when initiating masculinizing therapy. Interestingly, physiologic male testosterone concentrations and low estradiol levels are not requirements for the development of amenorrhea. Increasing this patient’s testosterone dosage (Answer E) has the added benefit of improving gender dysphoria (compared with progesterone). With the midpeak testosterone concentrations on the low end of the male reference range, the best choice in this case would be to increase the testosterone dosage.   Testosterone injections administered subcutaneously are just as effective as intramuscular dosing in achieving therapeutic concentrations in cisgender men with hypogonadism and in transgender men. Advantages to subcutaneous dosing include decreased anxiety and less pain during and after the injection. Subcutaneous doses are initiated at 50% of doses used intramuscularly (50 mg instead of 100 mg weekly and 100 mg instead of 200 mg every 2 weeks).

Reference(s): Deutsch MB. Guidelines for the Primary and Gender-Affirming Care of Transgender and Gender Nonbinary People. 2nd ed. Center of Excellence for Transgender Health , Department of Family and Community Medicine, University of California San Fransisco . June 2016. Available at: www.transhealth.ucsf.edu/guidelines .Hembree WC, Cohen-Kettenis PT, Gooren L, et al. Endocrine treatment of gender-dysphoric/gender-incongruent persons: an Endocrine Society Clinical Practice Guideline. J Clin Endocrinol Metab. 2017;102(11):3869-3903. PMID: 28945902

ITE 2019 Question 15 A 20-year-old woman seeks additional evaluation of delayed puberty. She was first evaluated at age 16 years because of primary amenorrhea and lack of breast development. Her initial evaluation included a head MRI that showed no pituitary abnormality, as well as the laboratory results shown in the table. No hormone therapy was started at that time. At age 17 years, her examination findings were still consistent with Tanner stage 1 development, and test results showed no change in gonadotropin or estradiol concentrations. She began taking estradiol, 0.5 mg every other day. Every 6 months, therapy was discontinued for 3 months to see if spontaneous pubertal progression might occur. The initial 2 years of therapy were associated with development of pubic hair only. At a follow-up visit 6 months ago, the estradiol dosage was increased to 0.5 mg daily. She has noticed initial breast development with this most recent dosage change. Her sense of smell is normal. Her mother’s height is 66 in (167.6 cm) and her father’s height is 72 in (182.9 cm). On physical examination, her height is 62 in (157.5 cm) and weight is 105 lb (47.7 kg) (BMI = 19.2 kg/m2). Breasts and pubic hair distribution are consistent with Tanner stage 2 development. Pelvic ultrasonography shows a smaller-than-normal uterus and ovaries with tiny follicles.

Measurement Age 16 Years 17 Years 20 Years FSH 1.6 mIU/mL (SI: 1.6 IU/L) 1.5 mIU/mL (SI: 1.5 IU/L) 1.0 mIU/mL (SI: 1.0 IU/L) LH 0.5 mIU/mL (SI: 0.5 IU/L) 0.4 mIU/mL (SI: 0.4 IU/L) 0.5 mIU/mL (0.5 IU/L) Estradiol 17 pg/mL (SI: 62.4 pmol/L) 2 pg/mL (SI: 7.3 pmol/L) 29 pg/mL (SI: 106.5 pmol/L) Prolactin 6.8 ng/mL (SI: 0.30 nmol/L) 8.0 ng/mL (SI: 0.35 nmol/L) … TSH 2.1 mIU/L 2.0 mIU/L … Total T 4 … 6.7 µg/dL (SI: 86.2 nmol/L) … ACTH … … 23 pg/mL (SI: 5.1 pmol/L) Cortisol (8 AM) … … 12 µg/dL (SI: 331.1 nmol/L) IGF-1 … … 132 ng/mL (SI: 17.3 nmol/L) Bone age 13 years 13 years 14 years Reference ranges: FSH <3.0 mIU /mL (<3.0 IU/L) (prepubertal); LH <1.0 mIU /mL (<1.0 IU/L) (prepubertal); estradiol 10-180 pg /mL (36.7-660.8 pmol /L) (follicular, premenopausal); prolactin, 4-30 ng/mL (0.17-1.30 nmol/L); TSH, 0.5-5.0 mIU /L; total T4, 5.5-12.5 µg/dL (94.02-213.68 nmol/L); ACTH, 10-60 pg /mL (2.2-13.2 pmol /L); cortisol (8 AM), 5-25 µg/dL (137.9-689.7 nmol/L); IGF-1, 122-384 ng/mL (16.0-50.3 nmol/L).

Which of the following is the best next step in this patient’s management? Change to an oral contraceptive Add micronized progesterone, 100 mg daily Start GH therapy Increase the estradiol dosage Discontinue estradiol and observe for 3 months Correct Answer: D Learning objective:Select the appropriate hormone therapy when a woman with delayed puberty and idiopathic hypogonadotropic hypogonadism presents as an adult.

Rationale: Estrogen therapy in a female with delayed puberty should be initiated slowly and with dosages lower than those used in adults to optimize breast and uterine development. In older adolescents and young adults, estrogen therapy might start as low as 25 to 50 mcg daily transdermally or 0.5 mg to 1 mg daily with oral estradiol and would be gradually increased every 6 months over the course of 2 years. The best step in this patient’s management is to increase the estradiol dosage (Answer D). Progesterone is initiated only after significant breast development is achieved, as progesterone might interfere with optimal breast growth. Cyclic progesterone therapy is generally initiated after about 2 years of estrogen therapy or when breakthrough bleeding occurs. Therefore, since she has only advanced to Tanner stage 2, it would be too early to advance to oral contraceptives (Answer A) or to add micronized progesterone (Answer B). Progesterone would also be added only for days 1 through 12 of a 28-day cycle, not daily as listed in Answer B.

Rationale Continued: Girls with delayed puberty due to Turner syndrome often present with short stature, so delaying bone maturation might be important to improve growth potential in those cases. In contrast, the steady linear growth rate in idiopathic hypogonadotropic hypogonadism is associated with taller than normal achieved height. In this case, although she has not achieved mid-parental height, this patient is only showing signs of early pubertal development with a bone age younger than chronologic age. Even in cases of constitutional delay of puberty, serum GH and IGF-1 concentrations might be low but will increase in response to estrogen (or testosterone therapy in males). Patients with idiopathic hypogonadotropic hypogonadism are not typically GH deficient and will continue to grow when treated with sex steroids alone. While GH (Answer C) is less likely to lead to epiphyseal closure than estrogen replacement, there is no evidence that patients with idiopathic hypogonadotropic hypogonadism benefit from GH therapy. Before age 18 years, estrogen therapy might be discontinued periodically after optimal breast development has occurred to determine whether spontaneous ovulatory cycles occur as in constitutional delay. Persistent hypogonadotropic hypogonadism is highly likely if spontaneous pubertal progression does not occur by age 18. Because this patient is older than 18 and her breast development has not progressed beyond Tanner stage 2, discontinuing estradiol (Answer E) is not recommended.

Reference(s): Gravholt CH, Andersen NH, Conway GS, et al; International Turner Syndrome Consensus Group. Clinical practice guidelines for the care of girls and women with Turner syndrome: proceedings from the 2016 Cincinnati International Turner Syndrome Meeting. Eur J Endocrinol . 2017;177(3):G1-G70. PMID: 28705803 Klein KO, Rosenfield RL, Saten RJ, et al. Estrogen replacement in Turner syndrome: literature review and practical considerations. J Clin Endocrinol Metab. 2018;103(5):1790-1803. PMID: 29438552

ITE 2019 Question 28 A 16-year-old girl presents with primary amenorrhea. She underwent thelarche at age 10 years but had no axillary or pubic hair development. She has had no hot flashes, acne, hirsutism, galactorrhea, or symptoms of thyroid disease, and she is not sexually active. On physical examination, her blood pressure is 110/80 mm Hg. Her weight is 147 lb (66.8 kg), and height is 69 in (175.3 cm) (BMI = 21.7 kg/m2). On skin examination, she has no axillary or pubic hair. Her breasts are Tanner stage 4. No masses are noted on abdominal examination. Pelvic examination reveals a vaginal length of 1.5 cm. A mass is palpated in the right inguinal area.

Laboratory test results: TSH = 3.72 mIU /L (0.5-5.0 mIU /L) Prolactin = 8.6 ng/mL (4-30 ng/mL) (SI: 0.37 nmol/L [0.17-1.30 nmol/L]) FSH = 3.2 mIU/mL (<3.0 mIU/mL [prepubertal]; 2.0-12.0 mIU/mL [follicular]; 4.0-36.0 mIU/mL [midcycle]; 1.0-9.0 mIU/mL [luteal]; >30 mIU/mL [postmenopausal]) (SI: 3.2 IU/L [<3.0 IU/L (prepubertal); 2.0-12.0 IU/L (follicular); 4.0-36.0 IU/L (midcycle); 1.0-9.0 IU/L (luteal); >30 IU/L (postmenopausal)])LH = 15.0 mIU/mL (<1.0 mIU/mL [prepubertal]; 1.0-18.0 mIU/mL [follicular]; 20.0-80.0 mIU/mL [midcycle]; 0.5-18.0 mIU/mL [luteal]; >30 mIU/mL [postmenopausal]) (SI: 15.0 IU/L [<1.0 IU/L (prepubertal); 1.0-18.0 IU/L (follicular); 20.0-80.0 IU/L (midcycle); 0.5-18.0 IU/L (luteal); >30 IU/L (postmenopausal)])Testosterone = 390 ng/dL (8-60 ng/dL) (SI: 13.5 nmol/L [0.3-2.1 nmol/L])Estradiol = 45 pg/mL (10-180 pg/mL [follicular]; 100-300 pg/mL [midcycle]; 40-200 pg/mL [luteal]; <20 pg/mL [postmenopausal]) (SI: 165.2 pmol/L [36.7-660.8 pmol/L (follicular); 367.1-1101.3 pmol/L (midcycle); 146.8-734.2 pmol/L (luteal); <73.4 pmol/L (postmenopausal)])β-hCG = <3.0 mIU/mL (<3.0 mIU/mL) (SI: <3.0 IU/L [<3.0 mIU/mL])Karyotype: 46,XY

Which of the following is the best next step in this patient’s management? Prescribe transdermal estradiol Refer for gonadectomy Prescribe low-dosage oral contraceptive Prescribe spironolactone Prescribe micronized progesterone Correct Answer: B Learning objective:Recommend management for a woman with complete androgen insensitivity syndrome.

Rationale: Androgen insensitivity syndrome results from pathogenic variants in the gene encoding the androgen receptor and can be complete or partial. In 46,XY individuals with complete androgen insensitivity, there is no androgen-mediated genital development in the fetus and there is absence of male external genitalia. There is also absence of androgen-mediated hair growth from adrenal androgens (pubic and axillary hair) and testicular androgens (male facial/body hair pattern). The testes are present and may be found in the inguinal canal, presenting as hernias, or as masses in the labioscrotal folds, but they are most commonly found intra-abdominally. The testes make antimullerian hormone. Therefore, there is regression of the mullerian structures: the uterus, tubes, and upper two-thirds of the vagina. Testosterone is also produced and aromatized to estradiol, resulting in breast development. Patients with complete androgen insensitivity have female features and no sexual hair. Partial androgen insensitivity syndrome is caused by a less deleterious pathogenic variant in the androgen receptor gene that results in partial activity. Male external genitalia development can be partial and incomplete, leading to ambiguous genitalia.Persons with androgen insensitivity syndrome are at increased risk for gonadoblastoma development in the testes if they are not removed. The risk of developing a germ-cell tumor before puberty is only 0.8%, but it increases to an estimated 5% to 22% after puberty. While still somewhat controversial, some studies have shown that gonadectomy (Answer B) can safely be delayed until after puberty when female secondary sexual characteristics are fully developed. Still, gonadectomy will need to be performed at some stage for this patient. From the hormonal perspective, she will not need estradiol therapy until that time (thus, Answer B is correct and Answer A is incorrect).

Rationale Continued: Persons with complete androgen insensitivity have “normal” male testosterone levels and androgens are converted by aromatization to estradiol. Following gonadectomy, the initial step is to begin replacement with estradiol. The uterus is absent, so progesterone (Answer E) (which is typically given along with estrogen for endometrial protection) is not needed. Patients with complete androgen insensitivity are infertile and therefore do not need hormonal contraceptives (Answer C). Hormonal contraceptives would also be inappropriate for hormone replacement in these patients. Spironolactone (Answer D) is used off-label to block androgen action at the androgen receptor for hyperandrogenic disorders in women, such as polycystic ovary syndrome. Because the androgen receptor is not active in complete androgen insensitivity, there is no need for androgen receptor blockade.

Reference(s): Ulbright TM, Young RH. Gonadoblastoma and selected other aspects of gonadal pathology in young patients with disorders of sex development. Semin Diagn Pathol. 2014;31(5):427-440. PMID: 25129544Cools M, Looijenga LH. Tumor risk and clinical follow-up in patients with disorders of sex development. Pediatr Endocrinol Rev. 2011;9(Suppl 1):519-524. PMID: 22423509Chaudhry S, Tadokoro-Cuccaro R, Hannema SE, Acerini CL, Hughes IA. Frequency of gonadal tumours in complete androgen insensitivity syndrome (CAIS): a retrospective case-series analysis. J Pediatr Urol. 2017;13(5):498.e1-498.e6. PMID: 28351649

ITE 2019 Question 34 A 50-year-old woman presents to your office with concerns of hirsutism and male-pattern balding. She had normal menarche at age 12 years and regular menses with 2 uncomplicated pregnancies. Her periods stopped 1 year ago, and she has noticed increased facial hair in a beard-like distribution over the past 18 months, some male-pattern balding, and increased periareolar hair. On physical examination, her BMI is 27 kg/m2 and blood pressure is 150/90 mm Hg. She is very muscular. She has terminal hairs on her chin in a full-beard distribution, 15 hairs on her areolae, and hair above and below the umbilicus. Her clitoris measures 2.8 x 1.7 cm. There are no striae. Laboratory test results: LDL cholesterol = 151 mg/dL (<100 mg/dL [optimal]) (SI: 3.91 mmol/L [<2.59 mmol/L]) Testosterone = 350 ng/dL (8-60 ng/dL) (SI: 12.1 nmol/L [0.3-2.1 nmol/L])DHEA-S = 120 µg/dL (15-200 µg/dL) (SI: 3.3 µmol/L [0.41-5.42 µmol/L])Prolactin = 15 ng/mL (4-30 ng/mL) (SI: 0.65 nmol/L [0.17-1.30 nmol/L])FSH = 19 mIU/mL (>30 mIU/mL [postmenopausal]) (SI: 19 IU/L [>30 IU/L])LH = 18 mIU/mL (>30 mIU/mL [postmenopausal]) (SI: 18 IU/L [>30 IU/L])Hemoglobin A1c = 5.8% (4.0%-5.6%) (40 mmol/mol [20-38 mmol/mol])

Which of the following is the best next test to evaluate this patient? Transvaginal ultrasonography Dexamethasone suppression test Pituitary MRI Ovarian venous sampling Fasting glucose and C-peptide Correct Answer: A Learning objective:Evaluate postmenopausal hyperandrogenism.

Rationale: In postmenopausal women, hirsutism or virilization is most commonly associated with obesity or hyperthecosis . However, the rapid onset and severity of the symptoms and signs in this patient raise concern for an ovarian tumor (usually Sertoli-Leydig–cell tumor, arrhenoblastoma , or hilus -cell tumor) that is secreting testosterone and causing virilization. Transvaginal ultrasonography (Answer A) is the first test to look for a tumor or asymmetry of the ovaries since the tumors are usually small. If only testosterone is elevated and the ultrasound is negative, performing ultrasonography again in 3 months may be helpful. In addition, there have been reports of rare testosterone-secreting tumors of the adrenal gland, so CT of the abdomen and pelvis might be considered (but this was not given as a choice). CT is not optimal imaging for evaluating ovarian morphology and transvaginal ultrasonography is preferred.  

Rationale Continued: A dexamethasone suppression test (Answer B) might be indicated if an adrenal mass were detected and/or the DHEA-S concentration were elevated. Pituitary MRI (Answer C) is appropriate in the workup for a pituitary tumor and is not indicated in this case. The gonadotropins are actually relatively suppressed from postmenopausal levels. Ovarian venous sampling (Answer D) was in vogue in the 1980s, but multiple studies have shown that it cannot distinguish between an ovarian tumor and an adrenal tumor. Hyperthecosis is a more common cause of postmenopausal hirsutism in which high gonadotropin levels drive androgen production from the ovarian theca cells. Whether women who present with hyperthecosis in menopausal years had polycystic ovary syndrome before menopause has not been clarified. Obesity can cause hirsutism; adipose tissue can have increased 5α-reductase activity, as well as local aromatase activity, which can cause androgenic and estrogenic effects. However, obesity alone does not usually result in virilization. Thus, the documentation of insulin resistance (Answer E) is not helpful in establishing a diagnosis. Exposure to exogenous androgens might also be in the differential diagnosis because many men now use testosterone gel. Testosterone therapy for women is not approved for hypoactive sexual desire disorder in the United States. If it were administered, a physiologic, not pharmacologic, level would be the goal. In addition, with exogenous supraphysiologic testosterone administration, FSH and LH levels are usually suppressed. 

Reference(s): Alpañés M, González- Casbas JM, Sánchez J, Pián H, Escobar- Morreale HF. Management of postmenopausal virilization. J Clin Endocrinol Metab. 2012;97(8):2584-2588. PMID: 22669303Rothman MS, Wierman ME. How should postmenopausal androgen excess be evaluated? Clin Endocrinol (Oxf). 2011;75(2):160-164. PMID: 21521309

ITE 2019 Question 46 A 42-year-old woman with polycystic ovary syndrome presents for follow-up. At the time of diagnosis, laboratory evaluation documented normal TSH, prolactin, FSH, and urinary free cortisol values. She was treated with an oral contraceptive pill, which she stopped 1 year ago. Since then, she has noted worsening hirsutism and acne, has had no menses, and has gained 22 lb (10 kg). On physical examination, her blood pressure is 130/80 mm Hg. Her height is 67 in (170.2 cm), and weight is 260 lb (118.2 kg) (BMI = 40.7 kg/m2). Cystic acne is present on her back. Her Ferriman-Gallwey score is 10, and acanthosis nigricans is present on her neck, in the axillae and groin, and on her knuckles. Pelvic examination reveals a normal uterus and ovaries, but is limited by obesity.Laboratory test results:Testosterone = 89 ng/dL (8-60 ng/dL) (SI: 3.1 nmol/L [0.3-2.1 nmol/L])DHEA-S = 122 µg/dL (18-244 µg/dL) (SI: 3.3 µmol/L [1.19-9.00 µmol/L])Hemoglobin A1c = 7.5% (4.0%-5.6%) (58 mmol/mol [20-38 mmol/mol])β-hCG = <3.0 mIU/mL (<3.0 mIU/mL) (SI: <3.0 IU/L [<3.0 IU/L])

Pelvic ultrasonography reveals an endometrial stripe with a cystic appearance and a thickness of 10 mm (see image). She is given a course of medroxyprogesterone, 10 mg daily for 10 days, which does not trigger a withdrawal bleed.

Which of the following assessments should you order next? Another pelvic ultrasound in 2 weeks FSH measurement Papanicolaou test Medroxyprogesterone challenge at a higher dose Endometrial biopsy Correct Answer: E Learning objective:ORDER THE APPROPRIATE EVALUATION FOR AN OBESE WOMAN WITH POLYCYSTIC OVARY SYNDROME WHO PRESENTS WITH LONGSTANDING AMENORRHEA IN THE ABSENCE OF TREATMENT.

Rationale: A woman with polycystic ovary syndrome, obesity, and type 2 diabetes mellitus is at increased risk for endometrial hyperplasia and cancer. Further, amenorrhea lasting longer than 6 months in a woman younger than 45 years with anovulation is an indication to evaluate the endometrial lining. The criterion standard test is endometrial biopsy (Answer E), although ultrasonography can also be used as an initial evaluation. A thick endometrial lining, greater than 4 mm, is concerning for endometrial abnormalities. In this patient, endometrial biopsy revealed endometrial cancer. The question arises whether women with polycystic ovary syndrome should have routine ultrasound monitoring given their 3-fold increased risk for endometrial cancer. Most women with endometrial cancer present with vaginal bleeding (75%-90%). Therefore, routine ultrasound surveillance of the uterine lining for detection of endometrial cancer is not recommended. The only exception is for patients with Lynch syndrome (familial early-onset colorectal cancer [age <50 years] and endometrial cancer), caused by a germline pathogenic variant in one of several DNA mismatch repair genes.

Rationale Continued: In the absence of a withdrawal bleed, there is no need to reassess the endometrial lining with ultrasonography (Answer A) because it most likely remains thick. If elevated, an FSH level (Answer B) would indicate menopause or perimenopause. However, this patient has no symptoms of menopause and the result would not explain the abnormal endometrial lining. A medroxyprogesterone challenge at a higher dose (Answer D) would not be the most appropriate choice—the dose already given should have been sufficient to trigger a withdrawal bleed, so a pathologic assessment is needed. A Papanicolaou test (Answer C) with endometrial cells can be indicative of endometrial pathology, but an endometrial biopsy offers direct assessment of endometrial pathology.

Reference(s): Smith RA, Manassaram -Baptiste D, Brooks D, et al. Cancer screening in the United States, 2014: a review of current American Cancer Society guidelines and current issues in cancer screening. CA Cancer J Clin. 2014;64(1):30-51. PMID: 24408568 Legro RS, Arslanian SA, Ehrmann DA, et al; Endocrine Society. Diagnosis and treatment of polycystic ovary syndrome: an Endocrine Society clinical practice guideline. J Clin Endocrinol Metab. 2013;98(12):4565-4592. PMID: 24151290Haoula Z, Salman M, Atiomo W. Evaluating the association between endometrial cancer and polycystic ovary syndrome. Hum Reprod. 2012;27(5):1327-1331. PMID: 22367984Gottschau M, Kjaer SK, Jensen A, Munk C, Mellemkjaer L. Risk of cancer among women with polycystic ovary syndrome: a Danish cohort study. Gynecol Oncol. 2015;136(1):99-103. PMID: 25451694

ITE 2019 Question 57 A 21-year-old woman presents for evaluation of secondary amenorrhea. Menarche occurred at age 13 years. Menses were regular until age 15 when they stopped for a year. Her pediatrician thought it was due to low body fat percentage from running cross-country, and he prescribed oral contraceptives. She had regular withdrawal bleeds on the pill, but stopped treatment on her own 6 months ago and has had no periods since. Micronized progesterone (for 10 days) was prescribed on 2 separate occasions; she had no withdrawal bleeding with the initial lower dose, but did have withdrawal bleeding after the higher dose. The patient has no hot flashes, vaginal dryness, or galactorrhea. She runs 2 to 4 miles 3 to 5 times a week. There is no history of eating disorders. She has never been pregnant and has never had a pelvic infection or gynecologic procedure. On physical examination, her blood pressure is 110/74 mm Hg. Her height is 65 in (165 cm), and weight is 115 lb (52.3 kg) (BMI = 19.1 kg/m2). She has no terminal hair growth or acne. The rest of her examination findings are unremarkable.

Recent laboratory test results:  TSH = 2.3 mIU /L (0.5-5.0 mIU /L) Prolactin = 12 ng/mL (4-30 ng/mL) (SI: 0.17-1.30 nmol/L)  LH = 13.0 mIU/mL (1.0-18.0 mIU/mL [follicular]; 20.0-80.0 mIU/mL [midcycle]; 0.5-18.0 mIU/mL [luteal]; >30 mIU/mL [postmenopausal]) (SI: 13.0 IU/L [<1.0 IU/L (prepubertal); 1.0-18.0 IU/L (follicular); 20.0-80.0 IU/L (midcycle); 0.5-18.0 IU/L (luteal); >30 IU/L (postmenopausal)])FSH = 6.6 mIU/mL (2.0-12.0 mIU/mL [follicular]; 4.0-36.0 mIU/mL [midcycle]; 1.0-9.0 mIU/mL [luteal]; >30 mIU/mL [postmenopausal]) (SI: 6.6 IU/L [<3.0 IU/L (prepubertal); 2.0-12.0 IU/L (follicular); 4.0-36.0 IU/L (midcycle); 1.0-9.0 IU/L (luteal); >30 IU/L (postmenopausal)])Estradiol = 62 pg/mL (10-180 pg/mL [follicular]; 100-300 pg/mL [midcycle]; 40-200 pg/mL [luteal]; <20 pg/mL [postmenopausal]) (SI: 227.6 pmol/L [36.7-660.8 pmol/L (follicular); 367.1-1101.3 pmol/L (midcycle); 146.8-734.2 pmol/L (luteal); <73.4 pmol/L (postmenopausal)]) Androstenedione = 322 ng/dL (80-240 ng/dL) (SI: 11.2 nmol/L [2.79-8.38 nmol/L])  Cortisol (8 AM) = 12 µg/dL (5-25 µg/dL) (SI: 331.1 nmol/L [137.9-689.7 nmol/L])  Antimullerian hormone = 10.0 ng/mL (0.9-9.5 ng/mL) (SI: 71.4 pmol /L [6.4-67.9 pmol /L])  Pelvic ultrasonography reveals an ovarian volume of 5 mL bilaterally with a uterine lining thickness of 6 mm.

Which of the following is the most likely diagnosis? Premature ovarian insufficiency Polycystic ovary syndrome Functional hypothalamic amenorrhea Empty sella syndrome Asherman syndromeCorrect Answer: B Learning objective : Distinguish between polycystic ovary syndrome and hypothalamic amenorrhea in a woman with secondary amenorrhea.

Rationale: This patient presents with secondary amenorrhea. Premature ovarian insufficiency (Answer A) is diagnosed by the presence of elevated FSH and low estradiol. Instead, this patient has a profile of FSH and estradiol concentrations that could be considered normal. Functional hypothalamic amenorrhea (Answer C) is associated with estrogen deficiency, presumably from loss of pulsatile GnRH secretion, which is not the case here.  An elevated LH-to-FSH ratio is not diagnostic of polycystic ovary syndrome (Answer B), but it can be suggestive of or consistent with LH-driven excess androgen production. Androstenedione and antimullerian hormone concentrations were both elevated in this patient. Taken together, the evidence in this case is suggestive of polycystic ovary syndrome. Metformin was prescribed, which can improve menstrual regularity and increase ovulatory frequency, even in lean women with polycystic ovary syndrome. Within 3 months, she resumed monthly menses. With this diagnosis, she preferred to restart oral contraceptives for birth control, and metformin was discontinued. 

Rationale Continued: Asherman syndrome (Answer E) describes amenorrhea resulting from intrauterine adhesions or fibrosis of the endometrium associated with a uterine procedure, such as dilation and curettage, uterine infection, or severe postpartum hemorrhage. This patient has no history of pelvic procedures, infection, or pregnancy.  Empty sella syndrome (Answer D) describes the appearance of a flattened pituitary gland on the floor of the sella , and in some cases the pituitary might not be visible. Pituitary function is usually normal.

Reference(s): Gordon CM, Ackerman KE, Berga SL, et al. Functional hypothalamic amenorrhea: an Endocrine Society Clinical Practice Guideline. J Clin Endocrinol Metab . 2017;102(5):1413-1439. PMID: 28368518 Legro RS, Arslanian SA, Ehrmann DA, et al; Endocrine Society. Diagnosis and treatment of polycystic ovary syndrome: an Endocrine Society Clinical Practice Guideline. J Clin Endocrinol Metab. 2013;98(12):4565-4592. PMID: 24151290 Wang JG, Lobo RA. The complex relationship between hypothalamic amenorrhea and polycystic ovary syndrome. J Clin Endocrinol Metab. 2008;93(4):1394-1397. PMID: 18230664

ITE 2019 Question 89 A 27-year-old woman seeks evaluation for a 6-month history of amenorrhea, dyspareunia, vaginal dryness, and night sweats. She has a history of goiter, and a recent TSH measurement was 2.0 mIU /L. She is sexually active. Her family history is notable for her mother and maternal aunt having early menopause at age 39 years and 40 years, respectively. She has a 19-year-old brother with autism. On physical examination, the thyroid is about twice normal size and she has atrophic vaginal lining on pelvic examination. Laboratory test results: FSH = 90.0 mIU/mL (2.0-12.0 mIU/mL [follicular]; 4.0-36.0 mIU/mL [midcycle]; 1.0-9.0 mIU/mL [luteal]; >30 mIU/mL [postmenopausal]) (SI: 90.0 IU/L [2.0-12.0 IU/L (follicular); 4.0-36.0 IU/L (midcycle); 1.0-9.0 IU/L (luteal); >30 IU/L (postmenopausal)])Estradiol = 20 pg/mL (10-180 pg/mL [follicular]; 100-300 pg/mL [midcycle]; 40-200 pg/mL [luteal]; <20 pg/mL [postmenopausal]) (SI: 73.4 pmol/L [36.7-660.8 pmol /L (follicular); 367.1-1101.3 pmol /L (midcycle); 146.8-734.2 pmol /L (luteal); <73.4 pmol /L (postmenopausal)]) Progesterone = 1.1 ng/mL (≤1.0 ng/mL) (SI: 3.5 nmol/L [<3.2 nmol/L]) Prolactin = 7 ng/mL (4-30 ng/mL) (SI: 0.30 nmol/L [0.17-1.30 nmol/L]) Karyotype: 46,XX

Which of the following tests is most likely to explain this patient’s clinical presentation? Qualitative β- hCG Measurement of ovarian antibodies Measurement of TPO antibodies Fragile X (FMR1) testing Pituitary MRI Correct Answer: D Learning objective : Explain the differential diagnosis of premature ovarian insufficiency and appropriately recommend fragile X carrier testing.

Rationale: Fragile X premutation screening (Answer D) should be performed in women with premature ovarian insufficiency, especially in those who have a family history of male relatives with learning disorders, autism, or mental retardation or family members with ataxia and/or dementia (suggestive of fragile X–related ataxia). The screening is accomplished by FMR1 genetic testing. In fragile X carriers, CGG repeats are in the premutation range.   Pregnancy would not elevate FSH and would be associated with much higher levels of estradiol and progesterone, so qualitative β- hCG (Answer A) is not necessary. Ovarian antibodies (Answer B) have not been identified as a cause of this type of premature ovarian insufficiency. As currently developed, their measurement is not clinically useful and has no predictive value. TPO antibodies (Answer C) are very useful in predicting thyroid disease but are not relevant to predicting autoimmune ovarian failure. There is no indication for pituitary MRI (Answer E) given the patient’s presentation, because an FSH-secreting pituitary tumor is rare and would not cause estrogen deficiency. Gonadotrope -secreting pituitary tumors are more common in men than in women. In premenopausal women, they may present with ovarian hyperstimulation due to biologically active FSH and persistently elevated estradiol levels. In postmenopausal women, they present with mass effects, headaches, vision changes, and often pituitary dysfunction because of tumor size. Usually, the tumors in postmenopausal women blunt the rise in gonadotropins ( ie, postmenopausal FSH and LH levels are instead in the premenopausal range).  

Reference(s): European Society for Human Reproduction and Embryology (ESHRE) Guideline Group on POI, Webber L, Davies M, et al. ESHRE guideline: management of women with premature ovarian insufficiency . Hum Reprod . 2016;31(5):926-937. PMID: 27008889    Wang T, Bray SM, Warren ST. New perspectives on the biology of fragile X syndrome. Curr Opin Genet Dev. 2012;22(3):256-263. PMID: 22382129 

MALE REPRODUCTION

ITE 2019 Question 13 A 27-year-old man seeks evaluation of infertility. The patient and his wife have been trying for pregnancy for the past 2 years without success. His wife has normal menstrual cycles and findings from a thorough gynecologic evaluation are normal. The couple has been timing sexual intercourse around her ovulation. The patient underwent puberty at age 13 years and started shaving at the same time as his peers. He recalls some increase in testicular size at puberty along with development of bilateral gynecomastia, which has remained stable. His sense of smell is normal. He reports normal libido and erectile function. He had one previous partner but always used contraception during intercourse. He has psychosocial problems such as difficulty making new friends and he had difficulty learning in high school. He takes no medications. On physical examination, he is a well-developed man without eunuchoid habitus. His blood pressure is 120/72 mm Hg, and BMI is 26.6 kg/m2. He has normal axillary and chest hair. There is bilateral, nontender gynecomastia, 6 x 4 cm on the right side and 5 x 5 cm on the left side. There is no galactorrhea, nipple retraction, or skin discoloration. His cardiac auscultation is normal. His phallus and scrotum are normal. His testes are firm and 4 mL bilaterally. His muscle bulk is slightly reduced, but tone and power are normal.

Laboratory test results:  Morning total testosterone (by tandem mass spectrometry) = 349 ng/dL (300-900 ng/dL) (SI: 12.1 nmol/L [10.4-31.2 nmol/L]) TSH = 2.13 mIU /L (0.5-5.0 mIU /L) Azoospermia is documented on semen analysis (2 specimens, each taken after 3 days of abstinence). Another morning total testosterone measurement (1 week later) is 364 ng/dL (12.6 nmol/L).

Which of the following should be the next diagnostic step in this patient’s evaluation? Measurement of dihydrotestosterone Testicular ultrasonography Measurement of gonadotropins Measurement of the testosterone-to-epitestosterone ratio Mammography Correct Answer: C Learning objective:Describe the clinical presentation of a man with mosaic Klinefelter syndrome and recommend appropriate diagnostic tests.

Rationale: This patient's clinical presentation of bilateral gynecomastia, very small testes, azoospermia, and psychosocial issues is consistent with Klinefelter syndrome. Klinefelter syndrome was first described by Harry Klinefelter in 1942 as a clinical entity characterized by gynecomastia, small testes, absent spermatogenesis, and variable Leydig-cell function. In 1959, the etiology of the condition was discovered to be a supernumerary X chromosome. Klinefelter syndrome is the most common sex chromosome abnormality with an estimated prevalence of 1 in 600 male newborns. It is also one of the most common genetic causes of primary hypogonadism (both testosterone deficiency and infertility) and is present in 11% of azoospermic men and in 4% of infertile men. Approximately 80% of men with Klinefelter syndrome have a 47,XXY karyotype while the remaining men have 46,XY/47,XXY mosaicism. Although small testes and azoospermia are almost universal in men with Klinefelter syndrome, the degree of androgen deficiency is variable with some men presenting with castrate testosterone levels and a eunuchoid habitus, while the others are well androgenized (such as this patient). Indeed, approximately 40% of men with Klinefelter syndrome have total testosterone levels in the normal range (although mean levels are still below those of age-matched controls). However, despite normal serum testosterone levels in some patients, both LH and FSH concentrations are always elevated, confirming inadequate negative feedback. Thus, measurement of LH and FSH levels (Answer C) is required to confirm the diagnosis of primary testicular failure.

Rationale Continued: Men with mosaic Klinefelter syndrome (usually 47,XXY/46,XY karyotype) are less severely affected than nonmosaic men, and many present with a normal male phenotype as a result of adequate androgenization. Although infertility remains an issue, the chance of finding sperm in the ejaculate is also significantly higher in mosaic patients. Studies have shown that on average, men with mosaic Klinefelter syndrome have lower LH levels, larger testicular volumes, and higher total sperm count than men with nonmosaic Klinefelter syndrome. The constitutively active genes from the extra X chromosome are posited to be the etiology behind most of the pathology that is seen in nonmosaic men; this might explain the milder phenotype in mosaic patients. This patient's karyotype analysis showed 47,XXY/46,XY mosaicism.Dihydrotestosterone is derived from testosterone via 5α-reduction and is likely to be normal in this patient (as his testosterone levels are normal). Hence, its measurement (Answer A) is not helpful in making the diagnosis. Determination of the testosterone-to-epitestosterone ratio (Answer D) is performed in men who are suspected of doping with exogenous testosterone and is also not indicated. This patient has bilateral gynecomastia with no suspicious masses, nipple discharge, or nipple discoloration, so mammography (Answer E) is unlikely to be helpful. Men with Klinefelter syndrome have small testes due to atrophy of seminiferous tubules. There is no suspicious mass palpable in this patient’s testes. Therefore, testicular ultrasonography (Answer B) is also not indicated.

Reference(s): Wikström AM, Dunkel L. Klinefelter syndrome. Best Pract Res Clin Endocrinol Metab . 2011;25(2):239-250. PMID: 21397196Aksglaede L, Link K, Giwercman A, Jørgensen N, Skakkebaek NE, Juul A. 47,XXY Klinefelter syndrome: clinical characteristics and age-specific recommendations for medical management. Am J Med Genet C Semin Med Genet. 2013;163C(1):55-63. PMID: 23345262

ITE 2019 Question 18 An 85-year-old man comes to the clinic for evaluation of a 3-year history of decreased libido, loss of morning erections, fatigue, and loss of muscle mass. His exercise tolerance has also decreased. He saw his primary care physician who checked an afternoon serum total testosterone concentration, which was 84 ng/dL (300-900 ng/dL) (SI: 2.9 nmol/L [10.4-31.2 nmol/L]). The patient is retired and lives with his girlfriend. He has no history of androgen disorders. He has 2 biologic children and 5 grandchildren. He has no gynecomastia and has not noticed decreased shaving frequency. His medical history is notable for hypertension, benign prostatic hypertrophy, and bilateral epididymal cysts. He takes a calcium-channel blocker and a 5α-reductase inhibitor. On physical examination, the patient looks younger than his stated age. His blood pressure is 132/79 mm Hg, pulse rate is 74 beats/min, and BMI is 28.6 kg/m2. He does not have a cushingoid face. His visual fields are normal to confrontation. There is no gynecomastia. There are no abdominal striae. His testes are 25 mL bilaterally. His prostate is symmetric and moderately enlarged without any nodules or induration. Muscle power is slightly decreased.

Laboratory test results (8 AM): Testosterone (by tandem mass spectrometry) = 52 ng/dL (300-900 ng/dL) (SI: 1.8 nmol/L [10.4-31.2 nmol/L]) LH = 3.3 mIU /mL (1.0-9.0 mIU /mL) (SI: 3.3 IU/L [1.0-9.0 IU/L]) Prolactin = 6.8 ng/mL (4-23 ng/mL) (SI: 0.3 nmol/L [0.17-1.00 nmol/L])TSH = 2.58 mIU/L (0.5-5.0 mIU/L)Free T4 = 1.2 ng/dL (0.8-1.8 ng/dL) (SI: 15.4 pmol/L [10.30-23.17 pmol/L])Cortisol = 10.7 µg/dL (5-25 µg/dL) (SI: 295.2 nmol/L [137.9-689.7 nmol/L])IGF-1 = 41 ng/mL (53-162 ng/mL) (SI: 5.4 nmol/L [6.9-21.2 nmol/L])PSA = 2.4 ng/mL (<7.2 ng/mL) (SI: 2.4 µg/L [<7.2 µg/L])

Which of the following is the best next step in the evaluation of this androgen-deficient patient? Inhibin B measurement GH measurement Scrotal ultrasonography Free testosterone measurement MRI of the sella Correct Answer: ELearning objective:Recommend MRI of the sella in patients with profound secondary hypogonadism

Rationale: The best screening test for the diagnosis of hypogonadism is measurement of serum total testosterone in a morning sample, preferably using mass spectrometry. In men deemed hypogonadal on the basis of the total testosterone level, the next step is to measure gonadotropins to determine whether hypogonadism is primary or secondary. Elevated gonadotropins suggest primary testicular failure. Low or inappropriately normal levels of gonadotropins are indicative of secondary hypogonadism. Prolactin and other pituitary hormones (if indicated) should be measured. MRI of the sella (Answer E) is indicated in patients with hyperprolactinemia and symptoms of mass effect (headaches, bitemporal hemianopia). Even in the absence of hyperprolactinemia and mass effect symptoms, pituitary MRI is indicated in the setting of severe secondary hypogonadism. Indeed, a few studies have shown that the likelihood of finding a sellar lesion is higher if secondary hypogonadism is profound (total testosterone <150 ng/dL [<5.21 nmol/L]). This patient has profound secondary hypogonadism. The next step in his care is therefore to perform a dedicated MRI of the pituitary. Indeed, this patient's MRI showed a macroadenoma ( nonsecreting ) that was compressing the infundibulum and abutting the chiasm (see image). His visual fields were normal on perimetry. After neurosurgical consultation, it was decided to follow him conservatively. He was educated regarding symptoms of apoplexy. Androgen replacement with a transdermal testosterone gel improved his symptoms.

Rationale Continued: Coronal view.            Sagittal view. Because GH is secreted in pulses, its measurement (Answer B) is not helpful in diagnosing GH deficiency and will not add information beyond what is learned from measuring IGF-1 (which is more appropriate to assess because it reflects average GH secretion over time). This patient has GH deficiency as reflected by his low IGF-1 level. Measurement of free testosterone (Answer D) is indicated in patients with conditions that result in alterations of serum SHBG concentrations (such as obesity, liver disease, HIV), none of which this patient has. Inhibin B (Answer A) is a product of Sertoli cells of the testes and its measurement is helpful during evaluation of infertility. It has no role in the evaluation of this patient. Similarly, scrotal ultrasonography (Answer C) is unlikely to be helpful in his evaluation.

Reference(s): Basaria S. Male hypogonadism. Lancet. 2014;383(9924):1250-1263. PMID: 24119423 Rhoden EL, Estrada C, Levine L, Morgentaler A. The value of pituitary magnetic resonance imaging in men with hypogonadism. J Urol. 2003;170(3):795-798. PMID: 12913700

ITE 2019 Question 22 A 62-year-old transgender woman is referred for management of gender dysphoria. The patient reports being in denial of her gender identity for much of her life. At age 53 years, she established care with both a psychologist and psychiatrist after suffering an emotional breakdown. At that time, she came out as transgender to these mental health professionals. She then came out to family and friends 2 years later. At age 56, she began cross-sex hormone therapy with estrogen and antiandrogens. She has been on numerous estrogen formulations, including oral, sublingual, transdermal, and intramuscular. Despite attempts with different doses and formulations of estrogen, her testosterone and estradiol levels have never reached target. She was advised to discontinue hormone therapy for 3 weeks to reassess baseline testosterone, after which treatment was resumed as outlined in the table. On physical examination, her height is 69 in (175 cm) and weight is 185 lb (84 kg) (BMI = 27.3 kg/m2). Her blood pressure is 145/88 mm Hg, and pulse rate is 85 beats/min. Her physical examination findings are notable for a testicular volume of 15 mL bilaterally and sparse body hair.

Laboratory test results: Treatment Regimen Measurement None Spironolactone: 100 mg twice daily   Estradiol: 2 mg daily Spironolactone: 200 mg twice daily   Estradiol: 4 mg daily Spironolactone: 100 mg twice daily   Serum estradiol Not measured Not measured 37 pg/mL (SI: 135.8 pmol/L) 35 pg/mL (SI: 128.5 pmol/L) Serum testosterone 977 ng/dL (SI: 33.9 nmol/L)   677 ng/dL (SI: 23.5 nmol/L) 506 ng/dL (SI: 17.6 nmol/L) 580 ng/dL (SI: 20.1 nmol/L) Reference ranges (female): estradiol: 10-180 pg/mL (SI: 36.7-660.8 pmol/L); testosterone: 8-60 ng/dL (SI: 0.3-2.1 nmol/L).

Which of the following is the best next step in this patient’s management? Increase the estradiol dosage Increase the estradiol and spironolactone dosages Switch the mode of estradiol delivery from oral to sublingual Recommend the addition of progesterone Recommend elective orchiectomy Correct Answer: E Learning objective:Identify appropriate treatment strategies to normalize testosterone levels in transgender women.

Rationale: The goal of hormone therapy for transgender women (male-to-female) is to achieve serum levels of sex steroids in the female range. The route of estrogen administration can be oral, sublingual, transdermal, or intramuscular. Which formulation to recommend to a particular patient should be personalized based on factors such as age, medical comorbidities, risk of venous thromboembolism, ease of monitoring, cost, and patient preference. Many patients request the intramuscular formulation due to the high levels of estrogen obtained a few days after an injection and due to advice from others in the community who believe that the levels of estrogen obtained with the intramuscular route promote greater feminization. The disadvantages of the intramuscular formulation are the peaks and troughs of serum estradiol levels, fear of needles for some patients, and stronger potential for abuse than with other formulations. Ethinyl estradiol is a semisynthetic estrogen given orally that should be avoided due to increased risk of venous thromboembolism. The nonoral formulations of estrogen are presumed to have a lower risk of venous thromboembolism, as they avoid the first-pass metabolism in the liver. A disadvantage of conjugated oral estrogens is that serum estradiol levels cannot be followed to allow for dosage titration. Options to lower the androgens include antiandrogens such as spironolactone and cyproterone acetate, as well as GnRH agonists given every 1 to 3 months. The Endocrine Society guidelines recommend achieving estradiol and testosterone levels in the normal female range (100-200 pg /mL and <50 ng/dL, respectively).

Rationale Continued: Some transgender women do not achieve target levels of sex hormones despite adequate or high dosages of estrogen and spironolactone. The most effective method to lower this patient’s androgens is an elective orchiectomy (Answer E), which is a low-risk surgery. Further increasing the dosages of spironolactone and/or estrogen (Answers A and B) is incorrect, as this approach will most likely not adequately lower her testosterone and will increase the risk of greater adverse effects. While switching from oral to sublingual estradiol (Answer C) might increase the estrogen level, it would most likely not adequately lower her testosterone. It is also uncertain whether progesterone (Answer D) would adequately lower her testosterone level and this medication may carry adverse effects.

Reference(s): Hembree WC, Cohen- Kettenis PT, et al. Endocrine treatment of gender-dysphoric/gender-incongruent persons: an Endocrine Society clinical practice guideline. J Clin Endocrinol Metab . 2017;102(11):3869-3903. PMID: 28945902Tangpricha V, den Heijer M. Oestrogen and anti-androgen therapy for transgender women. Lancet Diabetes Endocrinol. 2017;5(4):291-300. PMID: 27916515

ITE 2019 Question 55 A 58-year-old man with hypertension, hyperlipidemia, and obesity presents for follow-up. Ten months ago, he was documented to have 2 low testosterone measurements when he sought evaluation for symptoms of fatigue, poor sleep, worsening erectile function, and mild lower urinary symptoms. Secondary hypogonadism was diagnosed, and he was prescribed a topical testosterone gel. While on the testosterone therapy, he noticed a modest increase in his energy but a mild worsening of his urinary symptoms. He had only modest improvement in erectile function, so a phosphodiesterase inhibitor was added. He also takes atorvastatin and lisinopril. On physical examination, his height is 67 in (170 cm) and weight is 210 lb (95.5 kg) (BMI = 32.9 kg/m2). His blood pressure is 144/87 mm Hg, and pulse rate is 71 beats/min. Examination findings are normal. Testicular volume is 12 to 15 mL bilaterally.

Measurement Baseline 10 Months on Testosterone Total testosterone (300-900 ng/dL [SI: 10.4-31.2 nmol/L]) 238 and 261 ng/dL (SI: 8.3 and 9.1 nmol/L) 672 ng/dL(SI: 23.3 nmol/L)PSA (<3.8 ng/mL [<3.8 µg/L]) 1.2 ng/mL (SI: 1.2 µg/L) 2.9 ng/mL (SI: 2.9 µg/L) Hematocrit (41%-50% [0.41-0.50]) 47% (0.47) 51% (0.51) Chemistry panel Normal Normal DXA scan Hip T-score, –1.2 Spine T-score, –1.0 Hip T-score, –1.2 Spine T-score, –1.1 Laboratory test results:

Which of the following is the best next step in this patient’s management? Continue testosterone therapy at the current dosage Decrease the testosterone dosage due to elevated hematocrit Decrease the testosterone dosage to target testosterone <500 ng/dL (<17.4 nmol/L) given his age Increase the testosterone dosage due to lack of increase in T-scores Refer for urologic evaluation due to increase in PSA Correct Answer: E Learning objective:In a patient on testosterone therapy, determine when a rise in prostate-specific antigen should prompt urologic evaluation.

Rationale: Men on testosterone replacement therapy should be regularly monitored to determine whether their serum testosterone concentrations are at goal and whether they have developed any adverse effects related to the therapy. Our understanding of the relationship between testosterone therapy and prostate cancer is complex and limited due to a lack of adequately powered randomized controlled trials of long duration. Testosterone therapy given to men with metastatic prostatic cancer can worsen their clinical course and is therefore contraindicated. In fact, androgen deprivation therapy is a common treatment modality for men with prostate cancer. Nonetheless, there is no solid evidence that testosterone therapy causes new prostate cancer to develop. It is well established that men treated with testosterone typically experience an average rise in PSA of 0.3 to 0.5 ng/mL (0.3-0.5 µg/L). The Endocrine Society’s clinical practice guideline has a strong recommendation based on very low-quality evidence that clinicians should assess prostate cancer risk in men being considered for testosterone therapy. It recommends against testosterone therapy without further urologic evaluation in men with a palpable prostate nodule or PSA level greater than 4 ng/mL (>4 µg/L) or greater than 3 ng/mL (>3 µg/L) in men at high risk of prostate cancer. The authors of the guideline also recommend a urologic consultation if the PSA level rises more than 1.4 ng/mL (>1.4 µg/L) within any 1-year period after initiation of testosterone therapy. Thus, this patient’s PSA level should be addressed (Answer E).

Rationale Continued: Continuing therapy at the same dosage (Answer A) does not address the significant rise in the PSA level. Decreasing the testosterone dosage (Answer C) would be appropriate in an elderly man, but not in a middle-aged man. The patient in this case most likely has benign prostatic hypertrophy, as he presented with lower urinary symptoms before starting testosterone therapy. The presence of lower urinary symptoms on therapy is not concerning. It is well established that testosterone therapy increases hemoglobin and hematocrit. This phenomenon occurs more frequently in older men and more commonly occurs with the intramuscular testosterone esters (enanthate, cypionate), as this formulation is associated with peak and trough levels of testosterone. According to the Endocrine Society’s clinical practice guideline on androgen deficiency, a baseline hematocrit level greater than 50% is a relative contraindication to therapy and should prompt an evaluation for potential causes. The authors recommend that testosterone therapy be discontinued if the hematocrit level reaches 54% and the drug be held until the hematocrit decreases to a safe level. In patients where this occurs, management options include trying a lower testosterone dosage, switching to a formulation with a lower risk of erythrocytosis, and/or performing phlebotomy. Addressing this patient’s hematocrit level by lowering his testosterone dosage (Answer B) is incorrect, as his hematocrit remains less than 54% while on testosterone therapy. Increasing the testosterone dosage (Answer D) due to a lack of improvement in bone density would be inappropriate, as the testosterone dosage should be based primarily on serum levels of testosterone rather than on signs or symptoms that may not improve with testosterone therapy.

Reference(s): Bhasin S, Cunningham GR, Hayes FJ, et al; Endocrine Society. Testosterone therapy in men with androgen deficiency syndromes: an Endocrine Society clinical practice guideline. J Clin Endocrinol Metab . 2010;95(6):2536-2559. PMID: 20525905

ITE 2019 Question 65 A 33-year-old man presents with low libido and erectile dysfunction with loss of morning erections for the last 5 months. There is no history of head or testicular trauma. He is married and has 2 biologic children. His medical history is notable for rheumatoid arthritis diagnosed 9 months ago. He was started on methotrexate, 7.5 mg weekly, but had persistent painful and swollen joints, particularly his knees. He did not want to start oral glucocorticoid therapy, so he has had injections of methylprednisolone and triamcinolone every 6 to 8 weeks for knee pain. On physical examination, his BMI is 32 kg/m2, phallus is normal, and testes are 15 mL bilaterally and without masses. The patient has swelling of the distal interphalangeal joints. Laboratory test results: Total testosterone (8 AM) = 151 ng/dL (300-900 ng/dL) (SI: 5.2 nmol/L [10.4-31.2 nmol/L]) LH = 2.9 mIU /mL (1.0-9.0 mIU/mL) (SI: 2.9 IU/L [1.0-9.0 IU/L])FSH = 3.3 mIU/mL (1.0-13.0 mIU/mL) (SI: 3.3 IU/L [1.0-13.0 IU/L])Prolactin = 13 ng/mL (4-23 ng/mL) (SI: 0.6 nmol/L [0.17-1.00 nmol/L])TSH = 1.1 mIU/L (0.5-5.0 mIU/L)Free T4 = 1.4 ng/dL (0.8-1.8 ng/dL) (SI: 18.0 pmol/L [10.30-23.17 pmol/L])IGF-1 = 134 ng/mL (113-297 ng/mL) (SI: 17.6 nmol/L [14.8-38.9 nmol/L])Glucose (fasting) = 116 mg/dL (70-99 mg/dL) (SI: 6.4 mmol/L [3.9-5.5 mmol/L])Transferrin saturation = 32% (14%-50%) Pituitary MRI shows empty sella .

Which of the following is responsible for the patient’s androgen deficiency? Empty sella Methotrexate Methylprednisolone Hemochromatosis Correct Answer: C Learning objective:Diagnose glucocorticoid-induced hypogonadism.

Rationale: This patient has glucocorticoid-induced hypogonadism (Answer C). Both endogenous and exogenous hypercortisolism are associated with hypogonadism in both sexes. Studies have shown an inverse association between the dosage of glucocorticoids and serum testosterone levels, and suppression of gonadal hormones can occur as early as 3 days after initiation of glucocorticoids. Although both primary and secondary forms of hypogonadism have been described, most studies report low or inappropriately normal gonadotropins along with low serum testosterone, suggesting a secondary process (as seen in the present case). Some studies have shown that glucocorticoids inhibit synthesis and secretion of GnRH. Recovery of the gonadal axis may take months after discontinuation of glucocorticoids. Studies have shown that testosterone replacement in men with glucocorticoid-induced hypogonadism improves bone mass, muscle mass and strength, and quality of life. This patient has an “empty sella ” (Answer A) on MRI. The appearance of an empty sella on MRI is due to the prolapse of the arachnoid into the sella and compression of the pituitary gland. The pituitary gland may be partially visualized or not visible at all. However, pituitary function is normal in most cases of empty sella. This patient’s other pituitary hormones (TSH, free T4, IGF-1, and prolactin) are normal, making central hypogonadism due to compression of the pituitary unlikely. Methotrexate (Answer B) is used in the treatment of rheumatoid arthritis and is not associated with hypogonadism. Although hemochromatosis (Answer D) is a well-established cause of secondary hypogonadism, this patient's transferrin saturation is normal (generally >45% in men with hemochromatosis).

Reference(s): Yeap BB, Grossmann M, McLachlan RI, et al. Endocrine Society of Australia position statement on male hypogonadism (part 1): assessment and indications for testosterone therapy. Med J Aust. 2016;205(4):173-178. PMID: 27510348

ITE 2019 Question 72 A 67-year-old man presents with fatigue, low libido, poor erectile function, and a decrease in sexual activity with his partner that is causing problems in their relationship. He has hypertension controlled on lisinopril and hydrochlorothiazide and class I obesity (BMI = 34 kg/m2). He wants to start testosterone replacement if it will improve his sexual function. He has no known cardiovascular disease. Laboratory test results: Total testosterone = 230 ng/dL (300-900 ng/dL) (SI: 8.0 nmol/L [10.4-31.2 nmol/L]) FSH = 2.0 mIU /mL (1.0-13.0 mIU/mL) (SI: 2.0 IU/L [1.0-13.0 IU/L])LH = 3.0 mIU/mL (1.0-9.0 mIU/mL) (SI: 3.0 IU/L [1.0-9.0 IU/L])

Which of the following patterns would best represent the effects of testosterone replacement in this older man with late-onset hypogonadism? Answer Libido Erectile Function Sexual Activity A. Increased ImprovedIncreased B. Increased No change No change C. No change Improved No change D. No change No change No change Correct Answer: A Learning objective : Counsel patients regarding the benefits of testosterone therapy on sexual function in symptomatic older men with late-onset hypogonadism and unequivocally low testosterone levels

Rationale: Hypogonadism is strongly associated with symptoms of sexual dysfunction including low sexual desire, erectile dysfunction, decreased satisfaction with sexual activity, and a reduction in overall sexual activity. The prevalence of hypogonadism increases with age, as well as with the presence of other comorbidities including obesity, type 2 diabetes mellitus, chronic kidney disease, liver disease, and cardiovascular disease. Many of these comorbid conditions, particularly diabetes and cardiovascular disease, also contribute to erectile dysfunction through direct effects on the vasculature, which may not respond to testosterone therapy. The benefits of treating younger men with hypogonadism with testosterone are clear, but the benefit-risk profile in older men (≥65 years) is less clear. The recent conclusion of the Sexual Function Trial substudy of the Testosterone Trial in Older Men showed that testosterone therapy for 1 year in older men with hypogonadism (defined by unequivocally low total testosterone levels [≤275 ng/dL (≤9.5 nmol/L)]) significantly improved sexual symptoms with increases in libido, erectile function, and overall sexual activity (thus, Answer A is correct and Answers B, C, and D are incorrect).

Rationale Continued: Furthermore, a higher increase in testosterone level on therapy was associated with a higher sexual activity score. Most of the participants in the trial had comorbid illnesses—obesity in 63%, hypertension in 72%, and history of myocardial infarction in 15% (more than 3 months before the start of the study). No participants were using phosphodiesterase inhibitors. There was no significant difference in cardiovascular outcomes during the 1-year study between testosterone and placebo groups. These benefits were realized in patients with high rates of obesity and hypertension, so the presence of these comorbidities does not alter expectation of clinical improvements in sexual symptoms. Testosterone therapy has been shown to be beneficial in older men with unequivocally low testosterone levels

Reference(s): Wu FC, Tajar A, Beynon JM, et al; EMAS Group. Identification of late-onset hypogonadism in middle-aged and elderly men. N Engl J Med . 2010;363(2):123-135. PMID: 20554979Snyder PJ, Bhasin S, Cunningham GR, et al; Testosterone Trials Investigators. Effects of testosterone treatment in older men. N Engl J Med. 2016;374(7):611-624. PMID: 26886521

ITE 2019 Question 75 A 62-year-old man with a 10-year history of type 2 diabetes mellitus seeks help for erectile dysfunction. He reports a normal libido. He has a sedentary desk job and does not have time to exercise. He has hypertension and dyslipidemia. He takes atorvastatin, 40 mg daily; ramipril; and metformin. On physical examination, his blood pressure is 128/72 mm Hg, pulse rate is 68 beats/min, and BMI is 37 kg/m2. He is well virilized , and he has 20-mL testes bilaterally. Findings on cardiopulmonary, abdominal, and neurologic examinations are normal. Laboratory test results (sample drawn when fasting): Total testosterone = 280 ng/dL (300-900 ng/dL) (SI: 9.7 nmol/L [10.4-31.2 nmol/L]) Total cholesterol = 215 mg/dL (<200 mg/dL [optimal]) (SI: 5.57 nmol/L [<5.18 mmol/L])HDL cholesterol = 35 mg/dL (>60 mg/dL [optimal]) (SI: 0.91 nmol/L [>1.55 mmol/L])LDL cholesterol = 115 mg/dL (<100 mg/dL [optimal]) (SI: 2.98 nmol/L [<2.59 mmol/L])Triglycerides = 200 mg/dL (<150 mg/dL [optimal]) (SI: 2.26 nmol/L [<1.70 mmol/L])Hemoglobin A1c = 7.2% (4.0%-5.6%) (55 mmol/mol [20-38 mmol/mol])

Which of the following is the best next step in the evaluation and management of his erectile dysfunction? Measurement of free testosterone Exercise tolerance test Intensification of his glucose-lowering regimen Addition of a second drug to treat his dyslipidemia Trial of an oral phosphodiesterase inhibitor Correct Answer: B Learning objective:Determine whether a patient needs formal cardiovascular assessment before starting an oral phosphodiesterase inhibitor for erectile dysfunction

Rationale: Erectile dysfunction is a strong marker of cardiovascular disease. Before initiating pharmacotherapy for erectile dysfunction, the clinician must confirm that this man has the exercise capacity to safely have sexual intercourse. Sexual intercourse is typically about 3 to 4 metabolic equivalents. Walking a mile in 20 minutes is about 4 metabolic equivalents. For sedentary men with clinically significant cardiovascular risk factors such as obesity, diabetes mellitus, and hyperlipidemia, an exercise tolerance test should be considered before initiation of an oral phosphodiesterase inhibitor (thus, Answer B is correct and Answer E is incorrect). Most experts recommend testing for hypogonadism in men who have erectile dysfunction. Eugonadism is necessary for normal male sexual function, including normal tumescence, but serum total testosterone concentrations must be very low (<150 ng/dL [<5.2 nmol/L]) to cause erectile dysfunction. In this man with normal libido and a slightly low total testosterone concentration, it is very unlikely that hypogonadism is the cause of his erectile dysfunction. Thus, measurement of calculated free testosterone (Answer A) is unlikely to be useful in determining the cause. Optimizing his glycemic control (Answer C) and lipid control (Answer D) might provide microvascular benefits, but both are under reasonable control currently. Neither will improve erectile dysfunction that is due to extant diabetic neuropathy and vasculopathy.

Reference(s): McVary KT. Erectile dysfunction. N Engl J Med. 2007;357(24):2472-2481. PMID: 18077811 Miner M, Seftel AD, Nehra A, et al. Prognostic utility of erectile dysfunction for cardiovascular disease in younger men and those with diabetes. Am Heart J. 2012;164(1):21-28. PMID: 22795278Nehra A, Jackson G, Miner M, et al. The Princeton III Consensus recommendations for the management of erectile dysfunction and cardiovascular disease. Mayo Clin Proc. 2012;87(8):766-778. PMID: 22862865

LIPID / OBESITY

ITE 2019 Question 7 A 36-year-old woman with a peak lifetime BMI of 46 kg/m2 had a laparoscopic gastric bypass operation in another state 8 weeks ago. She initially did well, but over the last 3 weeks she began to experience episodes of vomiting. Over the last 5 days, she has been vomiting almost everything she eats. Over the last 2 days, her husband says that she has become increasingly confused, dysarthric, and unsteady on her feet. On neurologic examination, she is clearly confused, has nystagmus, is unsteady on standing, has decreased sensation on her lower extremities, and has a right third nerve palsy.

This patient most likely has a deficiency of which of the following? Vitamin B 12 Folate Thiamine Zinc Correct Answer: C Learning objective:Differentiate among the vitamin deficiencies that can occur after gastric bypass surgery.

Rationale: The symptoms displayed by this patient are characteristic of Wernicke encephalopathy, which is caused by thiamine deficiency (Answer C). Thiamine deficiency causes neuronal death due to metabolic dysfunction of astrocytes within the central nervous system. The classic triad of this condition is confusion, ataxia, and nystagmus. A wide range of other abnormalities can be seen, including cranial nerve dysfunction, peripheral neuropathies, seizures, and psychosis. Because thiamine is a water-soluble vitamin, body stores can be depleted within days to weeks of inadequate intake. The condition typically presents 4 to 12 weeks after bariatric surgery but can occur as early as 2 weeks and as late as 18 months after surgery. Although most commonly reported following gastric bypass surgery, Wernicke encephalopathy can occur after any type of bariatric surgery. The most common antecedent is persistent vomiting, which then severely limits thiamine intake. Other less common precipitating factors are intravenous glucose or parenteral nutrition administration without thiamine supplementation. The condition is important to recognize, as treatment with parenteral thiamine (100 mg daily for 7 to 14 days, or 500 mg 3 times daily for 3 days) must be administered to prevent serious morbidity. Although vitamin B12 deficiency (Answer A) can cause neurologic symptoms and signs, body stores of B12 are sizable, so deficiency does not usually occur until 6 to 24 months after bariatric surgery. Folate deficiency (Answer B) is uncommon and typically presents as anemia. Zinc deficiency (Answer D) is rare; it is associated with skin and hair findings and is primarily seen after biliary pancreatic diversion.

Reference(s): Aasheim ET. Wernicke encephalopathy after bariatric surgery, a systematic review. Ann Surg . 2008;248(5):714-720. PMID: 18948797 Serra A, Sechi G, Singh S, Kumar A. Wernicke encephalopathy after obesity surgery: a systematic review. Neurology. 2007;69(6):615. PMID: 17679686Mechanick JI, Kushner RF, Sugerman HJ, et al; American Association of Clinical Endocrinologists; Obesity Society; American Society for Metabolic & Bariatric Surgery. American Association of Clinical Endocrinologists, The Obesity Society, and American Society for Metabolic & Bariatric Surgery medical guidelines for clinical practice for the perioperative nutritional, metabolic, and nonsurgical support of the bariatric surgery patient [published correction appears in Obesity (Silver Spring). 2010;18(3):649]. Obesity (Silver Spring). 2009;17(Suppl 1):S1-S70. PMID: 19319140

ITE 2019 Question 14 A 49-year-old man with a history of severe obesity and type 2 diabetes mellitus had a gastric bypass operation 3 years ago. His preoperative BMI was 42 kg/m2. After surgery, his diabetes resolved and his insulin therapy was stopped. His weight decreased and stabilized at a BMI of 29 kg/m2. He has had no follow-up for the last 16 months. Over the past month, he has had several episodes in which he felt shaky, sweaty, and irritable. Family members brought him to the emergency department for confusion that developed after he had a large breakfast at a buffet restaurant. In the emergency department, his glucose concentration was documented to be 35 mg/dL (1.9 mmol/L).

After treating his acute decompensation, which of the following is the most appropriate next treatment? Dietary modification Partial pancreatectomy Octreotide Acarbose Correct Answer: A Learning objective :Manage hyperinsulinemic hypoglycemia that develops after gastric bypass surgery.

Rationale: Gastric bypass surgery has a dramatic effect on carbohydrate metabolism. Forty to fifty percent of patients who have diabetes mellitus preoperatively do not require medication for diabetes after surgery. Diabetes resolves in many of these individuals within weeks of the operation. It appears that the exposure of the distal bowel to food results in exaggerated secretion of GLP-1, which may facilitate the improvement in glucose control seen after surgery. Postprandial hypoglycemia is an uncommon late complication of gastric bypass surgery that is increasingly recognized. It appears that in some individuals, perhaps in response to ongoing stimulation by GLP-1, β-cell proliferation occurs, resulting in islet hyperplasia associated with excessive insulin secretion and endogenous hyperinsulinemic hypoglycemia. Some patients may develop multiple small insulinomas. The management of this condition is controversial. While partial pancreatectomy (Answer B) was suggested in the initial series, other authors have suggested that many of these patients can be managed by reducing the intake of carbohydrates, consuming low–glycemic index carbohydrates, and always eating carbohydrates in the context of a mixed meal. While acarbose (Answer D), octreotide (Answer C), calcium-channel blockers, and diazoxide have all been used as treatments, diet alone (Answer A) alleviates symptoms in 50% to 70% of affected individuals. Thus, dietary modification is the initial treatment choice. It appears that the condition recurs in many individuals who have a subtotal pancreatectomy, and those who have more aggressive pancreatic surgery can develop pancreatic diabetes. For this reason, pancreatectomy is currently used only if other treatments fail and the patient remains debilitated by frequent episodes of hypoglycemia that limit functional capacity.

Reference(s): Service GJ, Thompson GB, Service FJ, Andrews JC, Collazo-Clavell ML, Lloyd RV. Hyperinsulinemic hypoglycemia with nesidioblastosis after gastric bypass surgery. N Engl J Med. 2005;353(3):249-254. PMID: 16034010 Rariy CM, Rometo D, Korytkowski M. Post-gastric bypass hypoglycemia. Curr Diab Rep. 2016;16(2):19. PMID: 26868861

ITE 2019 Question 20 A 23-year-old man seeks evaluation of high cholesterol. He was seen at a local health fair where he was told that his cholesterol was extremely high and that he should seek care. On physical examination he has small yellowish papules on his abdomen, lower back, and the extensor surfaces of his arms. Fasting lipid panel: Total cholesterol = 325 mg/dL (<200 mg/dL [optimal]) (SI: 8.42 mmol/L [<5.18 mmol/L]) Triglycerides = 3450 mg/dL (<150 mg/dL [optimal]) (SI: 38.99 mmol/L [<1.70 mmol/L]) HDL cholesterol = 30 mg/dL (>60 mg/dL [optimal]) (SI: 0.78 mmol/L [>1.55 mmol/L]) LDL cholesterol, cannot be calculated

Which of the following abnormalities does this man most likely have? ATP-binding cassette A1 (ABCA1) deficiency LDL-receptor deficiency Apolipoprotein E2/E2 Lipoprotein lipase deficiency Overproduction of apolipoprotein B Correct Answer: D Learning objective:Identify the clinical features of genetic hyperlipidemias.

Rationale: This man has marked hypertriglyceridemia. Lipoprotein lipase (LPL) is the enzyme that is responsible for the catabolism of triglyceride-rich lipoprotein particles. Deficiency of LPL (Answer D) therefore will produce this phenotype. LPL has a cofactor: apolipoprotein C2. Deficiency of apolipoprotein C2 can also produce this picture. Individuals with severe hypertriglyceridemia develop eruptive xanthoma, which are the skin lesions described in this patient. The apolipoprotein E2/E2 phenotype (Answer C) is present in patients with dysbetalipoproteinemia , which is also referred to as type III hyperlipidemia, or broad-beta disease. Classic skin manifestations include tuberoeruptive lesions at the elbows and palmar xanthomas. The apolipoprotein E2/E2 phenotype occurs in about 1 per 100 persons, but the development of characteristic dyslipidemia is infrequent and usually appears later in life due to acquired medical conditions such as hypothyroidism, obesity, diabetes mellitus, or estrogen replacement therapy. ATP-binding cassette A1 (ABCA1) is a protein involved in moving cholesterol from peripheral tissues onto HDL particles. Deficiency of this protein (Answer A) results in the condition known as Tangier disease, characterized by very low HDL-cholesterol levels and the classic physical examination finding of orange tonsils, which this patient does not have.

Rationale Continued: LDL-receptor deficiency (Answer B) results in the condition known as familial hypercholesterolemia. Patients with familial hypercholesterolemia have very high LDL-cholesterol levels, tendinous xanthomas, and premature coronary artery disease. Overproduction of apolipoprotein B (Answer E) is the underlying problem in patients with familial combined hyperlipidemia. These individuals can have modest elevations in either triglycerides or LDL cholesterol or both. This condition is associated with atherosclerotic cardiovascular disease; however, the degree of triglyceride elevation seen in this condition is not as high as it is in this patient and it is not associated with the eruptive xanthomas.

Reference(s): Garg A, Simha V. Update of dyslipidemia. J Clin Endocrinol Metab . 2007;92(5):1581-1589. PMID: 17483372Berglund L, Brunzell JD, Goldberg AC, et al; Endocrine Society. Evaluation and treatment of hypertriglyceridemia: an Endocrine Society clinical practice guideline. J Clin Endocrinol Metab. 2012;97(9):2969-2989. PMID: 22962670

ITE 2019 Question 26 A 59-year-old man is 8 months status post non–q-wave myocardial infarction. After atorvastatin, 80 mg daily, was prescribed, his LDL-cholesterol level decreased dramatically. However, he developed pain in his lower extremities and a rise in his creatine kinase level twice the upper normal limit. He stopped the atorvastatin and the pain resolved. His TSH level is normal. His triglyceride level has been in the range of 220 to 270 mg/dL (2.49-3.05 mmol/L) each time it has been checked since his myocardial infarction.

Which of the following medications should be started as the best next step in this patient’s management? Rosuvastatin, 10 mg daily Fenofibrate, 145 mg daily Ezetimibe, 10 mg daily Evolocumab , 420 mg subcutaneously once monthly Niacin, 1000 mg daily Correct Answer: ALearning objective:Recommend options for managing statin-associated muscle pain.

Rationale: As many as 5% to 10% of patients who are prescribed statin therapy experience myalgias. This is a much greater prevalence than statin-induced myopathy, which the US FDA defines as a creatine phosphokinase level greater than 10 times the upper normal limit, or rhabdomyolysis, defined as a creatine phosphokinase level greater than 10,000 U/L (>167 µ kat /L). Rhabdomyolysis is an emergent condition that warrants the immediate cessation of statin therapy to prevent renal failure. However, this condition occurs in only 3 per 100,000 person-years. Deciding what to do when patients have muscle pain, weakness, or cramps when taking a statin is a difficult clinical problem. A number of options are available and no clear clinical trial data define the best approach. Statins are currently the single best class of medications for cardiovascular disease prevention. As a result, the first goal is to try to keep a patient such as this one on the highest tolerated statin dosage possible. The best first approach is to prescribe a lower dosage of a statin that the patient has not tried before. For the described patient, that would be rosuvastatin, 10 mg daily (Answer A). This patient does not necessarily need a PCSK9 inhibitor (Answer D) at this time, and attempting an alternative statin is the best next step. While fenofibrate (Answer B), niacin (Answer E), and ezetimibe (Answer C) all can produce favorable effects on serum lipid levels, none has been shown to produce reductions in cardiovascular endpoints when used as a single agent, so prescribing any one of them would not be the next step until various approaches to prescribing statins have failed.   

Reference(s): Joy TR, Hegele RA. Narrative review: statin-related myopathy. Ann Intern Med . 2009;150(12):858-868. PMID: 19528564 Cornier MA, Eckel RH. Non-traditional dosing of statins in statin-intolerant patients-is it worth a try? Curr Atheroscler Rep. 2015;17(2):475. PMID: 25432858Ahmad Z. Statin intolerance. Am J Cardiol. 2014;113(10):1765-1771. PMID: 24792743

ITE 2019 Question 32 A 57-year-old postmenopausal woman is referred for management of cardiovascular risk. Eleven months ago, she had advanced lipid testing through her naturopathic primary care provider’s office and was found to have elevated lipoprotein (a) levels. At the time, she was told that her cardiovascular risk was very high and was prescribed a medication, which she did not start. Recently, she developed chest tightness; cardiac workup was unrevealing with no evidence of clinical cardiovascular disease. She takes alprazolam for generalized anxiety disorder and several dietary supplements, but no other medications. She does not smoke cigarettes, and she drinks 2 to 4 alcoholic beverages a week. She exercises regularly. Her father had a stroke at age 57 years, and a paternal uncle had a stroke at age 60 years. Her paternal grandmother underwent 2-vessel coronary bypass surgery and aortic valve replacement at age 72 years. On physical examination, she is a healthy, anxious-appearing woman. Her blood pressure is 106/68 mm Hg. Her height is 68 in (172.7 cm), and weight is 163 lb (74 kg) (BMI = 24.8 kg/m2). The rest of her examination findings are normal.

Laboratory test results (sample drawn while fasting, no treatment): Total cholesterol = 197 mg/dL (<200 mg/dL [optimal]) (SI: 5.10 mmol/L [<5.18 mmol/L]) Triglycerides = 65 mg/dL (<150 mg/dL [optimal]) (SI: 0.73 mmol/L [<1.70 mmol/L]) HDL cholesterol = 86 mg/dL (>60 mg/dL [optimal]) (SI: 2.23 mmol/L [>1.55 mmol/L]) LDL cholesterol = 98 mg/dL (<100 mg/dL [optimal]) (SI: 2.54 mmol/L [<2.59 mmol/L]) Non-HDL cholesterol = 111 mg/dL (<130 mg/dL [optimal]) (SI: 2.87 mmol/L [<3.37 mmol/L]) Apolipoprotein B = 83 mg/dL (50-110 mg/dL) (SI: 0.83 g/dL [0.5-1.1 g/dL]) Lipoprotein (a) = 50 mg/dL (≤30 mg/dL) (>95th percentile for Caucasians) (SI: 1.79 µmol/L [≤1.07 µmol/L])Hemoglobin A1c = 5.5% (4.0%-5.6%) (37 mmol/mol [20-38 mmol/mol])TSH = 2.38 mIU/L (0.5-5.0 mIU/L)Fasting plasma glucose = 90 mg/dL (70-99 mg/dL) (SI: 5.0 mmol/L [3.9-5.5 mmol/L])Her 10-year atherosclerotic cardiovascular disease risk is 1.1% based on the American College of Cardiology/American Heart Association cardiovascular risk calculator.

Which of the following is the best next step in this patient’s management? Start low-dosage aspirin Start a statin Start niacin Start a PCSK9 inhibitor No therapy is necessary now Correct Answer: B Learning objective:Manage increased cardiovascular risk in an individual with elevated lipoprotein (a) levels.

Rationale: This patient is seeking advice on cardiovascular risk management. Her lipid panel is fairly unremarkable, but advanced lipoprotein testing obtained in view of her family history of atherosclerotic cardiovascular disease revealed elevated levels of lipoprotein (a).  The 2013 American College of Cardiology/American Heart Association guidelines for cholesterol lowering, which have been adopted widely in the United States, focus on global cardiovascular risk assessment. For primary prevention of atherosclerotic cardiovascular disease in adults, as in this patient, the cardiovascular risk calculator uses age, sex, race, systolic blood pressure, diabetes status, total cholesterol level, and HDL-cholesterol level as variables to calculate 10-year cardiovascular disease risk. Moderate- to high-intensity statin therapy is recommended for patients with a 10-year cardiovascular risk greater than 7.5%. Per the American College of Cardiology/American Heart Association risk calculator, this patient’s risk is estimated to be less than 5% and thus per current guidelines she does not necessarily need to be treated. However, family history and other advanced lipid measurements are not considered in the risk calculator.

Rationale Continued: Tricyclic antidepressants are well recognized to be associated with weight gain, although the amount of weight gained varies markedly. In this patient, the initiation of amitriptyline is temporally associated with the onset of her weight gain (thus, Answer A is correct). Assessing for other therapeutic options in the management of her headaches represents an important opportunity. Weight loss is observed upon initiation of thyroid hormone replacement in hypothyroid adults. Karmisholt et al reported an average weight loss of 9.5 lb (4.3 kg) after a year of levothyroxine therapy and achieving an average TSH concentration of 2.2 mIU/L (compared with a concentration of 102 mIU/L at treatment initiation). However, most of the weight loss observed was related to a decrease in lean mass compartment and not in bone and fat mass as measured by DXA. Investigators suggest most of the weight loss is related to excretion of excess body water. Although this patient is appropriately concerned about the adequacy of her thyroid hormone supplementation, her TSH level suggests that her levothyroxine dosage is appropriate and adjustments are unlikely to offer additional benefit to her weight-loss efforts (thus, Answer B is incorrect).

Rationale Continued: Lipoprotein (a) is characterized as an LDL-like particle in which apolipoprotein B is covalently bound by a single disulfide bond to apolipoprotein (a). It has high homology to plasminogen. Plasma concentrations of lipoprotein (a) are genetically determined and vary markedly among individuals. Lipoprotein (a) has a longer plasma residence time than LDL, and mechanisms by which lipoprotein (a) is cleared from the circulation are unclear. On average, persons of African descent have 2- to 3-fold higher lipoprotein (a) plasma concentrations than most persons of European and Asian descent. The exact physiologic function of lipoprotein (a) is still elusive, but it has been associated with wound healing and interaction with vascular wall matrix. Several lines of evidence, both epidemiologic and genetic, now conclusively suggest that lipoprotein (a) is independently associated with cardiovascular disease risk. This increased risk is irrespective of LDL concentration and increases linearly with increasing lipoprotein (a) concentration. Elevated lipoprotein (a) mediates myocardial infarction, stroke, and peripheral arterial disease. Genetic variation in the LPA gene is also strongly associated with aortic valve calcification and stenosis. For routine clinical care, currently available assays are considered fairly accurate for separating low-risk patients from high-risk patients. Because circulating lipoprotein (a) levels are genetically determined, there is little influence from diet and environment and plasma levels do not fluctuate significantly from a preset baseline over a lifetime.

Rationale Continued: Whether lipoprotein (a) is a risk factor when LDL cholesterol is controlled or low as it is in the patient in this vignette is not known. Also, there is no consensus on when to measure lipoprotein (a). However, recent studies suggest that elevated lipoprotein (a) remains a risk factor even when LDL-cholesterol levels less than 70 mg/dL (<1.81 mmol/L) are achieved. No current therapies are approved to lower lipoprotein (a). In this patient, the elevated lipoprotein (a) is inherited and most likely explains the history of cardiovascular disease in her father and uncle and valvular disease in her grandmother. It is reasonable to conclude that her cardiovascular risk is underestimated by the risk calculator, as neither her family history nor her elevated lipoprotein (a) is considered by this tool. There is robust clinical trial evidence that lowering LDL cholesterol decreases cardiovascular risk, even in individuals without particularly elevated LDL cholesterol, such as this patient. Thus, treatment with an LDL-cholesterol–lowering agent, preferably statin therapy, is indicated (thus, Answer B is correct and Answer E is incorrect). Aspirin (Answer A), an antiplatelet agent, has not been shown to be beneficial in individuals with elevated lipoprotein (a) levels. Among currently approved drugs, niacin (Answer C), PCSK9 inhibitors (Answer D), and mipomersen can modestly lower lipoprotein (a) levels, but their use is limited.

Reference(s): Tsimikas S. A test in context: lipoprotein (a): diagnosis, prognosis, controversies, and emerging therapies. J Am Coll Cardiol . 2017;69(6):692-711. PMID: 28183512 Nordestgaard BG, Langsted A. Lipoprotein (a) as a cause of cardiovascular disease: insights from epidemiology, genetics, and biology. J Lipid Res. 2016;57(11):1953-1975. PMID: 27677946

ITE 2019 Question 39 A 43-year-old woman makes an appointment to discuss weight-loss medications. Her BMI is 33 kg/m2. She has a history of multiple deep vein thromboses treated with warfarin and a seizure disorder that is controlled on phenytoin.

Which of the following US FDA–approved weight-loss medications would be the best choice for this patient? Orlistat, 120 mg 3 times daily Liraglutide, 3 mg daily Topiramate, titrated to 50 mg twice daily Naltrexone-bupropion, 8 mg/90 mg tablets titrated to 32 mg/360 mg daily Correct Answer: B Learning objective :List the contraindications for specific weight-loss medications.

Rationale: Several US FDA–approved weight-loss medications are now available. The range of options makes it important for clinicians to tailor the specific weight-loss medication that they prescribe to the unique needs of each patient. The patient in this vignette qualifies for a weight-loss medication because her BMI is greater than 30 kg/m2. However, she has a few medical conditions that complicate the choice. Orlistat (Answer A) would not be ideal because it can alter the levels of fat-soluble vitamins including vitamin K, which could affect the degree of anticoagulation that she is getting from her usual dose of warfarin. There are concerns about using liraglutide (Answer B) and naltrexone-bupropion (Answer D) in patients with known cardiovascular disease or uncontrolled hypertension because these medications can increase pulse rate due to sympathomimetic effects. Fortunately, this patient does not have either of these conditions. However, naltrexone-bupropion is relatively contraindicated because bupropion can lower the seizure threshold. Topiramate (Answer C) is not US FDA approved for weight loss as a single agent, and the recent Endocrine Society guidelines specifically recommend that it not be used for weight loss as there are other medications that are FDA approved. Phentermine-topiramate extended-release (not listed) should be used with caution in women of reproductive age because it has been associated with birth defects. Women taking this combination should have a documented negative pregnancy test before starting the medication and then pregnancy tests every month while taking it. Of the listed medications, the best choice for this patient is liraglutide (Answer B).

Reference(s): Apovian CM, Aronne LJ, Bessesen DH, et al; Endocrine Society. Pharmacological management of obesity: an Endocrine Society clinical practice guideline. J Clin Endocrinol Metab. 2015;100(2):342-362. PMID: 25590212Yanovski SZ, Yanovski JA. Long-term drug treatment for obesity: a systematic and clinical review. JAMA. 2014;311(1):74-86. PMID: 24231879Yanovski SZ, Yanovski JA. Naltrexone extended-release plus bupropion extended-release for treatment of obesity. JAMA. 2015;313(12):1213-1214. PMID: 25803343

ITE 2019 Question 54 A 27‑year-old woman with a history of hypertriglyceridemia and pancreatitis has controlled her hypertriglyceridemia reasonably well with diet and fenofibrate. Her triglyceride levels on this program have ranged from 595 to 880 mg/dL (6.72-9.94 mmol/L). She has 2 children, aged 4 and 7 years, and she developed gestational diabetes during her last pregnancy. She was not prescribed oral contraceptive pills because of her hypertriglyceridemia. She now returns 8 months after her last visit and reports that she is 12 weeks pregnant and still taking fenofibrate. Laboratory test results: Total cholesterol = 300 mg/dL (<200 mg/dL [optimal]) (SI: 7.77 mmol/L [<5.18 mmol/L]) Triglycerides = 815 mg/dL (<150 mg/dL [optimal]) (SI: 9.21 mmol/L [<1.70 mmol/L]) HDL cholesterol = 31 mg/dL (>60 mg/dL [optimal]) (SI: 0.80 mmol/L [>1.55 mmol/L]) Fasting glucose = 105 mg/dL (70-99 mg/dL) (SI: 5.8 mmol/L [3.9-5.5 mmol/L]) Hemoglobin A1c = 7.0% (4.0%-5.6%) (53 mmol/mol [20-38 mmol/mol])

Which of the following is the most reasonable strategy now? Continue fenofibrate Substitute atorvastatin for fenofibrate Substitute nicotinic acid for fenofibrate Substitute omega-3 fatty acids for fenofibrate Correct Answer: A Learning objective :Develop an approach to treating severe hypertriglyceridemia during pregnancy.

Rationale: Serum total cholesterol, HDL-cholesterol, LDL-cholesterol, and triglyceride levels increase during pregnancy (typically, the increases are by 75%, 40%, 70%, and 33%, respectively). The mean values for total cholesterol and triglycerides during pregnancy are 317 mg/dL (8.21 mmol/L) and 300 mg/dL (3.39 mmol/L), respectively. After delivery, lipid levels slowly return to prepartum levels. In women with underlying disorders of triglyceride metabolism, levels may rise during pregnancy to a degree that puts the mother at risk for pancreatitis, which could have serious implications for both the mother and the fetus. In addition, the development of gestational diabetes could increase the risk of marked hypertriglyceridemia. Drugs used for the treatment of lipid disorders should generally be stopped before conception. Statins (Answer B) are teratogenic and contraindicated in pregnancy. Ideally, all medications should be avoided during pregnancy, particularly during the first trimester when embryogenesis and tissue differentiation occur. This patient continued fenofibrate during the critical first trimester. Few data are available on the treatment of hypertriglyceridemia during pregnancy. Omega-3 fatty acids (Answer D) have been used to treat hypertriglyceridemia during pregnancy, but the available data suggest that they are not very effective. However, omega-3 fatty acids may be the treatment of choice in the first trimester. Observational studies and case reports suggest that fibrates may be used safely and effectively during pregnancy. Fenofibrate (Answer A) is more potent than omega-3 fatty acids, so it is most likely the best choice in the second and third trimesters and it should be continued in this patient. Nicotinic acid (Answer C) is less effective than fenofibrate in reducing triglyceride levels.

Rationale Continued: When pancreatitis due to hypertriglyceridemia develops during pregnancy, a few treatment approaches have been used. The standard approach of fasting, fluid administration, and pain control is the best first step. If hyperglycemia is present, then intravenous insulin can be administered. Other treatments that have been tried include intravenous heparin, plasma exchange, lipoprotein apheresis, and cesarean delivery if the pregnancy is far enough along.

Reference(s): Amin T, Poon LC, Teoh TG, et al. Management of hypertriglyceridaemia -induced acute pancreatitis in pregnancy. J Matern Fetal Neonatal Med . 2015:28(8):954-958. PMID: 25072837Crisan LS, Steidl ET, Rivera-Alsina ME. Acute hyperlipidemic pancreatitis in pregnancy. Am J Obstet Gynecol. 2008;198(5):e57-e59. PMID: 18359475Whitten AE, Lorenz RP, Smith JM. Hyperlipidemia-associated pancreatitis in pregnancy managed with fenofibrate. Obstet Gynecol. 2011;117(2 Pt 2):517-519. PMID: 21252809Nakao J, Ohba T, Takaishi K, Katabuchi H. Omega-3 fatty acids for the treatment of hypertriglyceridemia during the second trimester. Nutrition. 2015;31(2):409-412. PMID: 25592021

ITE 2019 Question 59 A 51-year-old man comes to see you for follow-up of type 2 diabetes mellitus. He takes no lipid-lowering medications. His fasting lipid panel reveals the following: LDL cholesterol = 98 mg/dL (<100 mg/dL [optimal]) (SI: 2.54 mmol/L [<2.59 mmol/L]) HDL cholesterol = 38 mg/dL (>60 mg/dL [optimal]) (SI: 0.98 mmol/L [>1.55 mmol/L]) Triglycerides = 290 mg/dL (<150 mg/dL [optimal]) (SI: 3.28 mmol/L [<1.70 mmol/L])

Which of the following would be the most appropriate medication(s) to start? Gemfibrozil Rosuvastatin Omega-3 fatty acids Niacin Rosuvastatin plus fenofibrate Correct Answer: B Learning objective:Recommend a treatment approach for moderate hypertriglyceridemia in patients with type 2 diabetes mellitus

Rationale: Cardiovascular disease remains the primary cause of death for persons with diabetes mellitus, and aggressive approaches to prevent disease are warranted. The most common lipid abnormalities seen in patients with diabetes are increased triglyceride and reduced HDL-cholesterol levels. While it would therefore make sense that lowering serum triglyceride levels might reduce cardiovascular disease risk, studies examining this hypothesis have to date not demonstrated any benefit of triglyceride-lowering with fibrates (Answer A) or omega-3 fatty acids (Answer C) in patients with diabetes. Treating individuals with diabetes with a fibrate failed to show a reduction in primary cardiovascular endpoints in the Bezafibrate Infarction Prevention (BIP) trial and more recently in the Action to Control Cardiovascular Risk in Diabetes (ACCORD) trial. The ACCORD trial specifically assessed whether adding a fibrate to a statin (Answer E) conferred any additional cardiovascular benefits. The results were negative. These studies have been criticized for not selecting individuals with elevated triglycerides at baseline, and post hoc analyses suggest a benefit among those with high baseline triglyceride levels. The Fenofibrate Intervention and Event Lowering in Diabetes (FIELD) study enrolled nearly 10,000 patients with type 2 diabetes, of which about 3650 were women. This study also failed to demonstrate that fenofibrate reduced cardiovascular events.

Rationale Continued: The AIM-HIGH study included many patients with diabetes and examined the effectiveness of adding niacin (Answer D) to aggressive statin therapy. Even though niacin reduced triglyceride levels and increased HDL-cholesterol levels, the trial was stopped early because of a failure of niacin treatment to reduce cardiovascular endpoints and an unexpected increase in stroke rates. Conversely, statins have been unequivocally shown to reduce coronary heart disease risk in patients with diabetes in randomized controlled trials and in post hoc analyses of such trials. A few ongoing studies are addressing this important question, and new insights may emerge over time, but the currently available data suggest that the single best approach to cardiovascular risk reduction in patients with diabetes is reduction of LDL-cholesterol levels with a statin (Answer B). As a result, the focus of the current cholesterol guidelines is almost exclusively on patients for whom statins should be prescribed.

Reference(s): Stone NJ, Robinson JG, Lichtenstein AH, et al; American College of Cardiology/American Heart Association Task Force on Practice Guidelines. 2013 ACC/AHA guideline on the treatment of blood cholesterol to reduce atherosclerotic cardiovascular risk in adults: a report of the American College of Cardiology/American Heart Association Task Force on Practice Guidelines [published correction appears in Circulation . 2014;129(25 Suppl 2):S46-S48]. Circulation . 2014;129(25 Suppl 2):S1-S45. PMID: 24222016 Cholesterol Treatment Trialists’ (CTT) Collaboration, Baigent C, Blackwell L, et al. Efficacy and safety of more intensive lowering of LDL cholesterol: a meta-analysis of data from 170,000 participants in 26 randomised trials. Lancet. 2010;376(9753):1670-1681. PMID: 21067804ACCORD Study Group, Ginsberg HN, Elam MB, et al. Effects of combination lipid therapy in type 2 diabetes mellitus [published correction appears in N Engl J Med. 2010;362(18):1748]. N Engl J Med. 2010;362(17):1563-1574. PMID: 20228404Jun M, Foote C, Lv J, et al. Effects of fibrates on cardiovascular outcomes: a systematic review and meta-analysis. Lancet. 2010;375(9729):1875-1884. PMID: 20462635Nordestgaard BG, Varbo A. Triglycerides and cardiovascular disease. Lancet. 2014;384(9943):626-635. PMID: 25131982

ITE 2019 Question 63 A 41-year-old woman comes to the emergency department with severe abdominal pain and is admitted to the hospital for acute pancreatitis. This is her first episode of pancreatitis. She does not drink alcohol or smoke cigarettes. She started oral contraceptives 3 months ago for irregular menses but otherwise takes no medications. On physical examination, she appears acutely ill. Her blood pressure is 142/84 mm Hg, and pulse rate is 112 beats/min. Her abdomen is diffusely tender on examination. Laboratory test results (sample drawn in the emergency department): Total cholesterol = 414 mg/dL (<200 mg/dL [optimal]) (SI: 10.72 mmol/L [<5.18 mmol/L]) HDL cholesterol = 21 mg/dL (>60 mg/dL [optimal]) (SI: 0.54 mmol/L [>1.55 mmol/L]) LDL cholesterol, not determined Triglycerides = 2743 mg/dL (<150 mg/dL [optimal]) (SI: 31.00 mmol/L [<1.70 mmol/L])Amylase = 804 U/L (26-102 U/L) (SI: 13.43 µkat/L [0.43-1.70 µkat/L])ALT = 37 U/L (10-40 U/L) (SI: 0.62 µkat/L [0.17-0.67 µkat/L])Hemoglobin A1c = 5.1% (4.0%-5.6%) (32 mmol/mol [20-38 mmol/mol])Glucose = 220 mg/dL (70-99 mg/dL) (SI: 12.2 mmol/L [3.9-5.5 mmol/L])

In addition to stopping her oral contraceptive pill, ordering nothing by mouth except new medications, and providing intravenous fluids and pain control, which of the following should be started now? Fish oil Metformin Fenofibrate Insulin drip Pioglitazone Correct Answer: D Learning objective:Recommend insulin in the setting of hypertriglyceridemia-induced pancreatitis, even in the absence of known diabetes mellitus, to increase lipoprotein lipase synthesis and activity.

Rationale: This patient has hypertriglyceridemia-induced pancreatitis. Given the recent initiation of oral contraceptives, estrogen was most likely a “second hit” triggering this severe hypertriglyceridemia (most individuals who have this degree of triglyceride elevations have an underlying genetic predisposition). Reducing her triglycerides is a major goal to clear this current episode of pancreatitis; maintaining triglycerides concentrations less than 500 mg/dL (<5.65 mmol/L) is important to prevent further attacks. High triglycerides come from intestinal absorption, so preventing any oral intake is a key step in limiting further chylomicron and VLDL production. However, this alone is rarely sufficient. Acute interventions to resolve the hypertriglyceridemia include apheresis, insulin infusion, or heparin infusion. Apheresis/plasma exchange can rapidly clear the lipids and should be considered if available. Insulin stimulates lipoprotein lipase, which metabolizes chylomicrons and VLDL to glycerol and free fatty acids. Heparin causes a rapid rise in lipoprotein lipase, but the lipoprotein lipase is then rapidly cleared by depletion of stores; the transient benefit of heparin has limited use. Even though this patient is not diabetic (her hemoglobin A1c level is 5.1% [32 mmol/mol] despite her current glucose concentration of 220 mg/dL [12.2 mmol/L]), infusion of insulin (Answer D) along with dextrose to prevent hypoglycemia will enhance resolution of the hypertriglyceridemia and will resolve the pancreatitis.

Rationale Continued: Once the current episode of pancreatitis is resolved, the treatment plan should focus on prevention of further episodes. In addition to recommending a low-fat diet, discontinuation of oral contraceptives alone may return her triglyceride level to less than 500 mg/dL (<5.65 mmol/L). Additional options include prescribing a fibrate (Answer C), which can lower triglycerides 20% to 50%, or fish oil (Answer A) (≥3 g daily), which can lower triglycerides up to 50%. Should the patient meet diagnostic criteria for diabetes mellitus, metformin (Answer B) or pioglitazone (Answer E) could be considered, but there is no indication for the use of these agents in this acute setting.

Reference(s): Ewald N, Hardt PD, Kloer HU. Severe hypertriglyceridemia and pancreatitis: presentation and management. Curr Opin Lipidol. 2009;20(6):497-504. PMID: 19770656

ITE 2019 Question 85 A 39-year-old man seeks evaluation for a documented elevated LDL-cholesterol level. His father died of a myocardial infarction at age 29 years and his brother developed angina at age 32 years. The patient has intermittent chest pain that is consistent with angina, but he has not had any diagnostic testing. He is taking atorvastatin, 40 mg daily, and on this medication his fasting LDL-cholesterol level is 245 mg/dL (6.35 mmol/L). On physical examination, he has thickened Achilles tendons and nodules on the extensor tendons of his hands.

Which of the following mediations should be added as the best next step? Ezetimibe Fenofibrate Evolocumab Niacin Correct Answer: C Learning objective :Summarize the utility of PCSK9 inhibitors in the treatment of patients with familial hypercholesterolemia.

Rationale: This patient’s clinical picture is consistent with familial hypercholesterolemia. Fenofibrate (Answer B) and niacin (Answer D) have not been shown to reduce cardiovascular event rates when added to statin therapy. However, data from the IMPROVE-IT trial recently demonstrated that ezetimibe (Answer A) added to a regimen of simvastatin, 40 mg daily, resulted in a modest but statistically significant 6.4% reduction in a composite cardiovascular endpoint. This might be a reasonable option. However, given this patient’s dramatic LDL-cholesterol elevation, the best next step is to add evolocumab (Answer C), which is a monoclonal antibody that targets and degrades PCSK9. PCSK9 circulates in the blood and alters the liver’s handling of LDL cholesterol. Persons with pathogenic gene variants that lower PCSK9 levels have lower LDL-cholesterol levels. A few pharmaceutical approaches to lowering PCSK9 levels have been developed. Evolocumab was approved in August 2015. It is specifically approved for use, in addition to diet and maximally tolerated statin therapy, in adult patients with heterozygous or homozygous familial hypercholesterolemia. The other US FDA–approved medication in this class is alirocumab. An additional treatment option for patients with familial hypercholesterolemia is lipopheresis , although this is not widely available.

Reference(s): Cannon CP, Blazing MA, Giugliano RP, et al; IMPROVE-IT Investigators. Ezetimibe added to statin therapy after acute coronary syndromes. N Engl J Med . 2015;372(25):2387-2397. PMID: 26039521Sabatine MS, Giugliano RP, Wiviott SD, et al; Open-Label Study of Long-Term Evaluation against LDL Cholesterol (OSLER) Investigators. Efficacy and safety of evolocumab in reducing lipids and cardiovascular events. N Engl J Med. 2015;372(16):1500-1509. PMID: 25773607Ajufo E, Rader DJ. Recent advances in the pharmacological management of hypercholesterolaemia. Lancet Diabetes Endocrinol. 2016;4(5):436-446. PMID: 27012540

ITE 2019 Question 90 A 45-year-old man in the intensive care unit is found to have new-onset hypertriglyceridemia. He was admitted to the hospital 8 days ago with gallstone pancreatitis. Four days ago, his clinical course was complicated by respiratory failure requiring intubation and transfer to the intensive care unit. He developed an ileus and there is concern about pancreatic pseudocyst formation. As a result, enteral feedings have been deferred and he has been on central parenteral nutrition the past 3 days. This has been associated with some hyperglycemia currently managed with insulin infusion. He requires multiple intravenous medications provided in either saline or D5W riders. He also requires intravenous propofol as a sedative. He has no history of diabetes or lipid abnormalities. His current central parenteral nutrition formula is providing 70% of basal calories as determined by the Harris Benedict equation with 1.5 g/kg of protein and 30% of calories from lipid emulsion. On physical examination, he is sedated and intubated. He has a nasogastric tube for intermittent suction. His abdomen is distended with hypoactive bowel sounds. There are no peritoneal signs. His admission weight was 202 lb (91.8 kg) and height was 68.5 in (174 cm) (BMI = 30.3 kg/m2).

Laboratory test results: Sodium = 135 mEq /L (136-142 mEq /L) (SI: 135 mmol/L [136-142 mmol/L]) Creatinine = 1.4 mg/dL (0.7-1.3 mg/dL) (SI: 123.8 µmol/L [61.9-114.9 µmol/L]) Potassium = 3.2 mg/dL (3.5-5.0 mEq/L) (SI: 3.2 mEq/L [3.5-5.0 mmol/L])Triglycerides = 835 mg/dL (<150 mg/dL [optimal]) (SI: 9.44 mmol/L [<1.70 mmol/L])Free T4 = 0.6 ng/dL (0.8-1.8 ng/dL) (SI: 7.7 pmol/L [10.30-23.17 pmol/L])Metered glucose values over the course of 6 hours have ranged from 125 mg/dL (6.9 mmol/L) to 137 mg/dL (7.6 mmol/L) His insulin infusion is stable at 2 units/h.

Which of the following is the best next step in the management of this patient’s hypertriglyceridemia? Increase the rate of insulin infusion to maintain glucose levels between 80 and 110 mg/dL (4.4-6.1 mmol/L) Change central parenteral nutrition to peripheral parenteral nutrition Add a fibrate agent via the nasogastric tube Stop the lipid emulsion in central parenteral nutrition; continue the dextrose–amino acid solution Start intravenous levothyroxine Correct Answer: D Learning objective:Identify factors that contribute to hypertriglyceridemia in patients receiving central parenteral nutrition.

Rationale: The goal of nutrition support is to provide the necessary nutrients to meet the metabolic requirements of a patient who is unable to meet those needs with oral intake. Acute critical illness is a catabolic state, and in the absence of adequate nutrition, it is associated with breakdown of proteins into amino acids to support gluconeogenesis. Enteral nutrition is the preferred route to provide nutritional support in the critically ill patient. It is associated with lower risk for infection, especially when initiated early (within 48 hours) in the course of illness. In the presence of contraindications to enteral feedings (small-bowel obstruction, ileus, etc ), central parenteral nutrition is often considered. Yet, central parenteral nutrition has not been shown to have the same benefits as enteral nutrition when started early in the course of critical illness. In fact, a higher risk for metabolic derangements and infection has been reported. Hyperglycemia and hypertriglyceridemia are common metabolic complications of parenteral nutrition. Initiation of insulin therapy is recommended and can often be included in the parenteral nutrition preparation. However, there are limitations regarding the amount of insulin that can be included, and some patients require additional insulin administration to achieve desired glucose targets. Improving hyperglycemia with insulin therapy can also lower triglycerides. However, the initial benefits reported with intensive insulin therapy in the intensive care unit setting with reduced sepsis, intensive care unit length of stay, and hospital mortality have not been confirmed. Aggressive insulin therapy has also been plagued by concerns of hypoglycemia. As a result, intensive insulin therapy with tight glycemic control (glucose concentrations of 80 to 110 mg/dL [4.4 to 6.1 mmol/L]) has been largely abandoned. This patient’s glycemia is well controlled on his current insulin infusion; more aggressive insulin therapy (Answer A) is unlikely to significantly improve his severe hypertriglyceridemia and could put him at risk for hypoglycemia.

Rationale Continued: Parenteral nutrition via a peripheral vein (Answer B) is limited by the osmolarity of the dextrose–amino acid preparation causing phlebitis. As a result, peripheral parenteral nutrition formulas require large volumes to provide the calories desired or a large percentage of calories to be provided as fat emulsion. In this critically ill patient with hypertriglyceridemia, neither large volumes nor large amounts of fat (which can worsen hypertriglyceridemia) are desirable. A fibrate (Answer C) is effective in the management of hypertriglyceridemia in the ambulatory setting. However, administration via a nasogastric tube in the setting of an ileus is ill advised. Several factors must be considered when determining an optimal parenteral nutrition formula while minimizing the risk for metabolic complications in the critically ill patient. One is to avoid overfeeding when providing excess calories. In the critically ill patient, “permissive underfeeding” is recommended by providing basal calorie requirements as determined by simplistic equations (25 kcal/kg of actual body weight per day), predictive formulas (Harris Benedict equation), or measured metabolic rate via indirect calorimetry. In the obese patient (BMI >30 kg/m2), hypocaloric feeding is advised (11 to 14 kcal/kg of actual body weight per day), with 60% to 70% of basal requirements as determined by predictive formulas or indirect calorimetry. This patient’s parenteral nutrition is providing 70% of basal requirements as recommended in view of his BMI of 30 kg/m2.

Rationale Continued: It is unclear why free T4 was measured in this patient. Although the value is abnormal, it is unlikely to be contributing significantly to the metabolic abnormalities noted. TSH could be measured, but its interpretation is also influenced by critical illness. The most likely culprit of the thyroid function abnormalities in this setting is sick euthyroid syndrome, and observation with repeated thyroid function testing when the patient is clinically improved is advised. Initiation of intravenous levothyroxine therapy (Answer E) would not be the most appropriate management step. Generally, 30% of calories are provided as fat calories via a lipid emulsion. In patients with hypertriglyceridemia, calories provided via lipid emulsion should be limited or avoided. Discontinuation of the lipid emulsion in the central parenteral nutrition (Answer D) would be associated with the most clinically significant decrease in triglyceride levels. Of note, this patient is receiving fat calories through another source—propofol. Propofol is a short-acting hypnotic agent frequently used in the intensive care unit setting. It is administered as a lipid emulsion and has been shown to contribute to abnormalities in lipid metabolism, most frequently hypertriglyceridemia. Although discontinuation of propofol would also be expected to improve triglyceride levels, stopping the lipid emulsion in the central parenteral nutrition would be the easiest to accomplish. 

Reference(s): McClave SA, Martindale RG, Vanek VW, et al; A.S.P.E.N. Board of Directors; American College of Critical Care Medicine; Society of Critical Care Medicine. Guidelines for the provision and assessment of nutrition support therapy in the adult critically ill patient: Society of Critical Care Medicine (SCCM) and American Society for Parenteral and Enteral Nutrition (A.S.P.E.N.). JPEN J Parenter Enteral Nutr . 2009;33(3):277-316. PMID: 19398613NICE-SUGAR Study Investigators, Finfer S, Chittock DR, et al. Intensive versus conventional glucose control in critically ill patients. N Engl J Med. 2009;360:1283-1297. PMID: 26088909

PITUITARY

ITE 2019 Question 8 A 21-year-old woman with a history of optic nerve glioma diagnosed at age 11 years would like to establish care with an adult endocrinologist. At diagnosis, the optic nerve glioma was treated surgically and she had postoperative radiotherapy (cumulative dose, 40 Gy ) impacting the sellar region. She also developed complex partial epilepsy, which is now well controlled. She was treated with recombinant human GH from age 12 to 19 years. She feels well and has no concerns. Her periods are regular. She takes lamotrigine for epilepsy, but is on no other medications. On physical examination, her height is 61 in (155 cm) and weight is 110 lb (50 kg) (BMI = 20.8 kg/m2). Breasts and pubic hair are Tanner stage 4. Her blood pressure is 102/68 mm Hg.Her most recent laboratory testing was performed 2 years ago (when she was still on GH replacement):TSH = 1.4 mIU/L (0.5-5.0 mIU/L)Free T4 = 1.2 ng/dL (0.8-1.8 ng/dL) (SI: 15.4 pmol/L [10.30-23.17 pmol/L])Serum cortisol (8 AM) = 15.0 μg/dL (5-25 μg/dL) (SI: 413.8 nmol/L [137.9-689.7 nmol/L])IGF-1 = 205 ng/mL (116-341 ng/mL) (SI: 26.9 nmol/L [15.2-44.7 nmol/L])FSH = 8.0 mIU/mL (2.0-12.0 mIU /mL [follicular]) (SI: 8.0 IU/L [2.0-12.0 IU/L]) LH = 12.0 mIU /mL (1.0-18.0 mIU /mL [follicular]) (SI: 12.0 IU/L [1.0-18.0 IU/L]) Estradiol = 110 pg /mL (10-180 pg /mL [follicular]) (SI: 403.8 pmol /L [36.7-660.8 pmol /L])

Which of the following management strategies would you advise for this patient? Annual evaluation of full pituitary function indefinitely Annual evaluation of full pituitary function if symptomatic of hormonal deficiency Annual screening of gonadotropins only GHRH-arginine test Insulin tolerance test Correct Answer: A Learning objective:Counsel patients who have undergone cranial irradiation regarding the need for long-term (usually lifelong) screening of pituitary function.

Rationale: As a result of the significant improvement in survival outcomes after childhood cancer, it is increasingly recognized that treatment of malignancy in childhood can lead to lifelong consequences. In particular, survivors of childhood cancer are at risk of developing a spectrum of neuroendocrine abnormalities, primarily because of the effect of radiation therapy on the hypothalamus. Indeed, hypothalamic and pituitary endocrinopathies occur in up to 80% of adults following radiotherapy to these areas; even doses as low as 20 Gy can cause pituitary dysfunction. All pituitary hormones can be affected, but GH production is most commonly impaired as is the case in this vignette. For example, in one study of 748 childhood cancer survivors treated with cranial irradiation and observed for a mean of 27.3 years, 46.5% developed GH deficiency, 10.8% developed LH/FSH deficiency, 7.5% developed TSH deficiency, and 4% developed ACTH deficiency. In this study and others, cumulative incidence of pituitary dysfunction increased throughout the follow-up period. Therefore, it is recommended that individuals who have undergone pituitary radiation at a dose greater than 30 Gy undergo lifelong annual evaluation of full pituitary function even if they are symptom-free (thus, Answer A is correct and Answers B and C are incorrect).

Rationale Continued: In this vignette, the patient has an early-morning cortisol level of 15.0 µg/dL (413.8 nmol/L), which probably indicates sufficient ACTH secretion, although a level greater than 18.0 µg/dL (>496.6 nmol/L) would be required to be entirely reassuring. In contrast, a serum cortisol value less than 3.0 µg/dL (<82.8 nmol/L) is highly suggestive of cortisol deficiency. Therefore, it is reasonable for this patient to undergo further evaluation of ACTH reserve, although there is no definitive recommendation as to the gold standard stimulation test to use. The ACTH-stimulation test is done by many; however, it can miss cases of partial or early ACTH deficiency that may respond normally to cosyntropin . Other options include the metyrapone stimulation test or the insulin-induced hypoglycemia test (insulin tolerance test) (Answer E). The latter test is performed by administering insulin (0.1 units/kg) and measuring serum glucose and cortisol at regular intervals for 120 minutes after the injection. A normal response is characterized by a serum glucose value less than 50 mg/dL (<2.8 mmol/L) and a cortisol value greater than 18.0 µg/dL (>496.6 mmol/L). However, the insulin tolerance test is unpleasant, and hypoglycemia can be dangerous in elderly patients and in those with an underlying seizure disorder as this patient has. Therefore, it would not be recommended in this scenario. This patient developed GH deficiency after pituitary irradiation, which is the most common endocrine abnormality in this setting. GH replacement therapy was started after completion of cancer treatment in order to optimize skeletal growth during puberty and was discontinued some time after growth had ceased. There is good evidence that continuation of GH therapy into adulthood in such individuals can provide further benefits for bone density and could be considered in this case (especially if age-adjusted bone mineral density is low). IGF-1 should be measured now, but no further evaluation of the GH axis with GHRH-arginine (Answer D) or any other modality is required, as GH deficiency in this situation is most likely to be permanent.

Reference(s): Darzy KH. Radiation-induced hypopituitarism after cancer therapy: who, how and when to test. Nat Clin Pract Endocrinol Metab . 2009;5(2):88-99. PMID: 19165221Children’s Oncology Group. Children’s Oncology Group long-term follow-up guidelines for survivors of childhood, adolescent, and young adult cancer. Version 4.0, October 2013. Available at: www.survivorshipguidelines.org.Molitch ME, Clemmons DR, Malozowski S, Merriam GR, Vance ML; Endocrine Society. Evaluation and Treatment of Adult Growth Hormone Deficiency: an Endocrine Society Clinical Practice Guideline. J Clin Endocrinol Metab. 2011;96(6):1587-1609. PMID: 21602453Sklar CA, Antal Z, Chemaitilly W, et al. Hypothalamic-pituitary and growth disorders in survivors of childhood cancer: an Endocrine Society clinical practice guideline. J Clin Endocrinol Metab. PMID: 29982476

ITE 2019 Question 19 A 36-year-old man is referred for evaluation of abnormal results on thyroid function tests that were obtained during the workup of palpitations. He has no relevant medical history. His family history is notable for his mother having had “a thyroid problem.” He has no siblings. He takes no medications. On physical examination, his blood pressure is 128/85 mm Hg and pulse rate is 90 beats/min. He has very mild tremors when his hands are extended. His thyroid is palpable, but not obviously enlarged. There is no stare or lid lag. Free T4 = 1.9 ng/dL (0.8-1.8 ng/dL) (SI: 24.5 pmol /L [10.30-23.17 pmol/L])Total T3 = 210 ng/dL (70-200 ng/dL) (SI: 3.2 nmol/L [1.08-3.08 nmol/L])TSH = 5.3 mIU/L (0.5-5.0 mIU/L)You confirm these tests and order the following:SHBG = 7.5 µg/mL (1.1-6.7 µg/mL) (SI: 66.7 nmol/L [10-60 nmol/L])α-Subunit = 5.2 ng/mL (<1.2 ng/mL) (SI: 5.2 µg/L [<1.2 µg/L])Pituitary MRI shows an 8-mm right-sided pituitary adenoma.

Which of the following is the best next step in this patient’s management? Initiation of somatostatin analogue as long-term therapy Initiation of methimazole as long-term therapy Genetic testing for a pathogenic variant in the thyroid hormone receptor β gene T 3  suppression test Referral for neurosurgeryCorrect Answer: ELearning objective:Distinguish between a TSH-secreting adenoma and thyroid hormone resistance and recommend surgery as the primary therapy for TSH-secreting microadenomas.

Rationale: The differential diagnosis of elevated free thyroid hormone and inappropriately normal or high TSH includes thyroid hormone resistance syndrome and a TSH-secreting adenoma. Other causes are the co-existence of primary hyperthyroidism and heterophilic antibodies to mouse immunoglobulins that cause artifactual high TSH levels and the use of amiodarone that reduces T4 to T3 deiodination (T3 is not high in this setting). In this patient, the main differential diagnosis is between thyroid hormone resistance and a TSH-secreting adenoma.   The effects of thyroid hormone are mediated by thyroid hormone receptors, which are encoded by the thyroid hormone receptor α gene (THRA) and the thyroid hormone receptor β gene (THRB). Patients with inherited THRB pathogenic variants present with resistance to thyroid hormone. This disorder is characterized by elevated levels of thyroid hormones, normal or elevated TSH levels, and often thyroid enlargement. Patients with THRA pathogenic variants have only recently been identified. If a patient with thyroid hormone resistance is asymptomatic, the mainstay in management is to recognize the correct diagnosis and to avoid antithyroid treatment.

Rationale Continued: TSH-secreting adenomas represent about 1% of pituitary adenomas. Approximately 25% co-secrete other pituitary hormones, including GH, prolactin, or gonadotropins. Most of them are macroadenomas at the time of presentation. The distinction between thyroid hormone resistance and a TSH-secreting pituitary adenoma can be challenging. The following features, when present, favor a TSH-secreting adenoma: (1) An increased α pituitary glycoprotein subunit-to-TSH molar ratio (>1), which is calculated as follows: [ α subunit (ng/mL)/TSH (mIU/L) x 10](2) The absence of thyroid test abnormalities compatible with thyroid hormone resistance in other family members(3) An elevated serum SHBG concentration, reflecting a functionally true hyperthyroid state(4) Inability to suppress serum TSH with supraphysiologic doses of T3 (T3 suppression test)

Rationale Continued: This patient has a mild elevation of free T 4  and T 3 , an elevated SHBG and an elevated α subunit (with ratio of 12.0 [5.2/4.3 x 10]). These findings suggest the diagnosis of a TSH-secreting adenoma. The family history of a “thyroid problem” is not helpful given the high prevalence of hypothyroidism in the general population. TSH-secreting adenomas are rarely diagnosed at an early stage, as occurred in this vignette. More commonly, they are larger than 1 cm. Although TSH-secreting adenomas respond well to first-generation somatostatin analogues (with a hormonal response rate that is higher than that of GH-secreting adenomas), the high surgical curability of microadenomas makes surgery (Answer E) the best option here. Somatostatin analogue therapy (Answer A) is reserved in case of surgical failure. In general, antithyroid medications (Answer B) are not used to treat TSH-secreting adenomas definitively (except in some cases for short periods to control hyperthyroidism before surgery), as they may cause growth of the adenoma. At this point, the diagnosis is established and a T 3 suppression test (Answer D) is not necessary. In countries where thyrotropin-releasing hormone stimulation testing is available, this assessment may also help because a marked TSH response to thyrotropin-releasing hormone is less likely in TSH-secreting adenomas. Ultimately, genetic testing for pathogenic variants in the THRA and THRB genes (Answer C) may help, but surgery is the best next step in this case.

Reference(s): Ortiga -Carvalho TM, Sidhaye AR, Wondisford FE. Thyroid hormone receptors and resistance to thyroid hormone disorders. Nat Rev Endocrinol. 2014;10(10):582-591. PMID: 25135573Amlashi FG, Tritos NA. Thyrotropin-secreting pituitary adenomas: epidemiology, diagnosis, and management. Endocrine. 2016;52(3):427-440. PMID: 26792794Beck-Peccoz P, Persani L, Mannavola D, Campi I. Pituitary tumours: TSH-secreting adenomas. Best Pract Res Clin Endocrinol Metab. 2009;23(5):597-606. PMID: 19945025

ITE 2019 Question 36 A 32-year-old woman comes in for evaluation of secondary amenorrhea. She underwent menarche at age 14 years and previously had normal menstrual cycles. She has 2 children aged 4 and 2 years, but has not had a period since she stopped breastfeeding her youngest child 1 year ago. She has not resumed using contraception since her periods had not yet returned but does not desire more children. She has a history of mild-moderate depression, but currently takes no regular medication. On physical examination, she appears well with normal secondary sexual characteristics and normal breast exam with no evidence of galactorrhea. Her height is 68.5 in (174 cm), and weight is 150 lb (68.2 kg) (BMI = 22.5 kg/m2). Her blood pressure is 118/78 mm Hg. Laboratory test results: Prolactin = 95 ng/mL (4-30 ng/mL) (SI: 4.1 nmol/L [0.17-1.30 nmol/L]) LH = 2.0 mIU/mL (1.0-18.0 mIU/mL [follicular]; 20.0-80.0 mIU/mL [midcycle]; 0.5-18.0 mIU/mL [luteal]) (SI: 2.0 IU/L [1.0-18.0 IU/L (follicular); 20.0-80.0 IU/L (midcycle); 0.5-18.0 IU/L (luteal)])FSH = 1.8 mIU/mL (2.0-12.0 mIU/mL [follicular]; 4.0-36.0 mIU/mL [midcycle]; 1.0-9.0 mIU/mL [luteal]) (SI: 1.8 IU/L [2.0-12.0 IU/L (follicular); 4.0-36.0 IU/L (midcycle); 1.0-9.0 IU/L (luteal)])Estradiol = 25 pg/mL (10-180 pg /mL [follicular]; 100-300 pg /mL [midcycle]; 40-200 pg /mL [luteal]) (SI: 91.8 pmol /L [36.7-660.8 pmol /L (follicular); 367.1-1101.3 pmol /L (midcycle); 146.8-734.2 pmol /L (luteal)]) Pituitary-directed MRI shows a 4-mm, low-attenuation lesion consistent with an adenoma.

Which of the following pharmacotherapies would be the best treatment strategy for this patient? Combined oral contraceptive pill Bromocriptine Cabergoline Cabergoline and oral contraceptive pill No treatment required Correct Answer: A Learning objective:Recommend the oral contraceptive pill as an acceptable treatment of amenorrhea caused by microprolactinoma when fertility is not desired.

Rationale: The vignette describes a fairly straightforward case of a microprolactinoma presenting with secondary amenorrhea. Indications for use of dopamine agonists in the management of prolactinoma are tumor shrinkage in the setting of macroprolactinoma, amelioration of symptoms such as galactorrhea, and restoration of menstrual cycle and fertility in premenopausal women. This patient has developed amenorrhea and secondary hypogonadism due to her microprolactinoma . Given that this patient does not desire more children and that her only symptom is secondary amenorrhea, treatment with the combined oral contraceptive pill (Answer A) is reasonable. There has previously been some reluctance to use estrogen therapy in the setting of microprolactinoma without concomitant dopamine agonist therapy because of concerns about tumor growth, but there is no robust evidence to support this and the combined oral contraceptive pill is endorsed by the Endocrine Society’s clinical practice guideline on hyperprolactinemia as an acceptable treatment under these circumstances. Although there has not been a randomized controlled trial directly exploring the effects of estrogen on growth of microprolactinoma, retrospective analysis of patients treated with oral contraceptives and estrogen/progesterone replacement for 2 years did not document an increase in tumor size. However, there are no convincing safety studies exploring the use of oral contraceptives in the management of macroprolactinoma. Thus, if the pituitary adenoma were larger than 1 cm, use of oral contraceptives without dopamine agonists would not be appropriate or recommended.

Rationale Continued: Although the use of dopamine agonists (Answers B and C) in microprolactinoma is not incorrect under the circumstances of this vignette, they are more expensive than oral contraceptive pills and they are associated with more adverse effects (such as mood disorder); thus, dopamine agonists are less desirable for this patient, especially with a history of depression and in the absence of galactorrhea. Moreover, use of a dopamine agonist will render her fertile and she will require an additional method of contraception. The use of a dopamine agonist plus the oral contraceptive pill (Answer D) is a common method of managing such patients. It would certainly be correct if the patient had marked galactorrhea, as this symptom would not be treated by the oral contraceptive pill. Occasionally, galactorrhea may develop once oral contraceptive pills have been commenced and, in that setting, the addition of a dopamine agonist is entirely reasonable. However, in the absence of galactorrhea and no desire for fertility, it is less appropriate to treat her from the outset with 2 medications when only 1 (oral contraceptive pill) is necessary. If this patient were of an age commensurate with menopause, then no treatment of her microprolactinoma (Answer E) would be correct. However, she has secondary hypogonadism at age 32 years, which would result in an unacceptable increased risk of osteoporosis if untreated.

Reference(s): Melmed S, Casaneva FF, Hoffman AR, et al; Endocrine Society. Diagnosis and treatment of hyperprolactinema : an Endocrine Society clinical practice guideline. J Clin Endocrinol Metab. 2011;96(2):273-288. PMID: 21296991Testa G, Vegetti W, Motta T, et al. Two-year treatment with oral contraceptives in hyperprolactinemic patients. Contraception. 1998;58(2):69-73. PMID: 9773260

ITE 2019 Question 40 A 58-year-old man with metastatic melanoma being treated medically is admitted to the hospital with lethargy, altered mental status, and hypotension. In addition to his anticancer therapy, he takes levothyroxine for hypothyroidism (Hashimoto thyroiditis was diagnosed many years ago). Laboratory test results: Random cortisol = 2.1 μ g/dL (5-25 μ g/dL) (SI: 57.9 nmol/L [137.9-689.7 nmol/L])ACTH = <5 pg/mL (10-60 pg/mL) (SI: <1.1 pmol/L [2.2-13.2 pmol/L])Testosterone = 21 ng/dL (300-900 ng/dL) (SI: 0.7 nmol/L [10.4-31.2 nmol/L])LH = 0.4 mIU/mL (1.0-9.0 mIU/mL) (SI: 0.4 IU/L [1.0-9.0 IU/L])FSH = 2.1 mIU/mL (1.0-13.0 mIU/mL) (SI: 2.1 IU/L [1.0-13.0 IU/L])IGF-1 = 45 ng/mL (78-220 ng/mL) (SI: 5.9 nmol/L [10.2-28.8 nmol/L])Prolactin = 1.3 ng/mL (4-23 ng/mL) (SI: 0.06 nmol/L [0.17-1.00 nmol/L])TSH = 0.3 mIU/L (0.5-5.0 mIU /L) MRI shows homogeneous enlargement of the pituitary gland and stalk, which was not present on MRI 2 months ago.

Which of the following medications is the most likely cause of this patient’s pituitary abnormalities? Prednisone Ipilimumab Temozolomide Sunitinib Correct Answer: B Learning objective :Identify medications that can cause hypophysitis.

Rationale: Ipilimumab (Answer B) is a monoclonal antibody used in the treatment of patients with metastatic melanoma. Hypophysitis has been reported in 10% to 15% of treated patients; pituitary enlargement can occur within 2 months of treatment initiation, and corticotrophs and thyrotrophs are the most common cell types affected. This form of hypophysitis is different from lymphocytic hypophysitis that occurs peripartum in women. Ipilimumab is one agent in a new class of immunotherapy drugs that modulate immune checkpoint proteins, such as cytotoxic T-lymphocyte antigen-4 (CTLA4) and programmed death-1 (PD-1). Ipilimumab is an anti-CTLA4 antibody, and nivolumab and pembrolizumab are anti-PD-1 antibodies. Less commonly, these drugs can also cause thyroiditis leading to hypothyroidism and hyperthyroidism and, rarely, adrenalitis leading to adrenal insufficiency.Although prednisone (Answer A) can certainly suppress ACTH secretion, it does not cause pituitary enlargement. High-dosage steroids are not effective in treating this hypophysitis. Temozolomide (Answer C) is an alkylating agent used in the treatment of gliomas and has been useful in the treatment of some patients with pituitary carcinomas and very aggressive macroadenomas. Sunitinib (Answer D) is a tyrosine kinase inhibitor that has been used to treat thyroid cancer, among other cancers, but it has not been implicated as a cause of hypophysitis.

Reference(s): Corsello SM, Barnabei A, Marchetti P, De Vecchis L, Salvatori R, Torino F. Endocrine side effects Induced by immune checkpoint inhibitors. J Clin Endocrinol Metab. 2013;98(4):1361-1375. PMID: 23471977Faje AT, Sullivan R, Lawrence D, et al. Ipilimumab-induced hypophysitis: a detailed longitudinal analysis in a large cohort of patients with metastatic melanoma. J Clin Endocrinol Metab. 2014;99(11):4078-4085. PMID: 25078147Albarel F, Gaudy C, Castinetti F, et al. Long-term follow-up of ipilimumab-induced hypophysitis, a common adverse event of the anti-CTLA-4 antibody in melanoma. Eur J Endocrinol. 2015;172(2):195-204. PMID: 25416723

ITE 2019 Question 42 A 51-year-old man presents with decreased libido and erectile dysfunction and is found to have the following laboratory values: Testosterone = 197 ng/mL (300-900 ng/dL) (SI: 6.8 nmol/L [0.31-1.04 nmol/L]) Prolactin = 39 ng/mL (4-23 ng/mL) (SI: 1.7 nmol/L [0.17-1.00 nmol/L]) A 14-mm pituitary adenoma is identified on MRI. While taking cabergoline, 0.5 mg twice weekly, his prolactin and testosterone levels normalize and his libido and erectile function improve. Two years later, he develops headaches and erectile dysfunction again despite maintaining a normal prolactin level on cabergoline therapy.

Which of the following should be the next management step? Discontinue cabergoline Switch cabergoline to bromocriptine Increase the cabergoline dosage to 1.0 mg twice weekly Perform another pituitary-directed MRI Correct Answer: D Learning objective :Distinguish prolactinomas from clinically nonfunctioning adenomas.

Rationale: Of concern in this patient is the discrepancy between his prolactin level of only 39 ng/mL (1.7 nmol/L) and the size of the adenoma—14 mm. Although this discrepancy could be due to inefficient production of prolactin by a prolactinoma, it is more likely due to stalk dysfunction caused by a nonfunctioning adenoma or some other mass lesion such as a meningioma or Rathke cleft cyst. A dopamine agonist could indeed normalize prolactin levels and, if the hypogonadism were the result of hyperprolactinemia, correct decreased libido and erectile dysfunction but have no effect on the growth of a mass lesion that is not a prolactinoma. However, he now has symptoms suggestive of hypogonadism with a normal prolactin, raising concerns about increased mass effect. Therefore, performing a pituitary-directed MRI (Answer D) to assess for an increase in tumor size should be the next management step, not stopping treatment (Answer A). The concern in this vignette is ensuring that this is not a prolactinoma; increasing the cabergoline dosage (Answer C) would not help distinguish a prolactinoma from a nonsecreting tumor. The patient’s prolactin level has normalized with cabergoline at the current dosage, so there is no reason to switch to bromocriptine (Answer B). 

Reference(s): Melmed S, Casanueva FF, Hoffman AR, et al; Endocrine Society. Diagnosis and treatment of hyperprolactinemia: an Endocrine Society clinical practice guideline. J Clin Endocrinol Metab . 2011;96(2):273-288. PMID: 21296991Karavitaki N, Thanabalasingham G, Shore HC, et al. Do the limits of serum prolactin in disconnection hyperprolactinaemia need re-definition? A study of 226 patients with histologically verified non-functioning pituitary macroadenoma. Clin Endocrinol (Oxf). 2006;65(4):524-529. PMID: 16984247

ITE 2019 Question 47 A 28-year-old woman is referred for evaluation of a pituitary adenoma that was incidentally discovered on cranial MRI performed to investigate headache. She reports no change in appearance or weight. Her menstrual cycles are regular (last menstrual period was 2 weeks ago), and she is currently sexually active. She wishes to become pregnant as soon as possible. On physical examination, she has no obvious features of GH or cortisol excess. Her height is 62 in (157.5 cm), and weight is 123 lb (56 kg) (BMI = 22.5 kg/m2). Her blood pressure is 118/69 mm Hg. Laboratory test results: TSH = 1.8 mIU/L (0.5-5.0 mIU/L)Free T4 = 1.1 ng/dL (0.8-1.8 ng/dL) (SI: 14.2 pmol/L [10.30-23.17 pmol/L])Cortisol (8 AM) = 18 μg/dL (5-25 μg/dL) (SI: 496.6 nmol/L [137.9-689.7 nmol/L])Estradiol = 315 pg/mL (10-180 pg/mL [follicular]) (SI: 1156.4 pmol/L [36.7-660.8 pmol/L]) IGF-1 = 168 ng/mL (117-321 ng/mL) (SI: 22.0 nmol/L [15.3-42.1 nmol/L]) Prolactin = 28 ng/mL (4-30 ng/mL) (SI: 1.22 nmol/L [0.17-1.30 nmol/L])

MRI shows a 9-mm, low-attenuation lesion in the center and right side of the pituitary gland (coronal image on the left and sagittal image on the right). It elevates the pituitary gland overlying it, abutting and minimally compressing the left side of the optic chiasm and deviating the pituitary stalk to the left. Findings on visual field assessment are normal.

Which of the following is the best management option? Refer for consideration of pituitary surgery Commence cabergoline therapy Commence octreotide therapy Perform another pituitary-directed MRI in 2 years Advise routine pituitary-directed MRI in the first trimester of pregnancy Correct Answer: A Learning objective:Devise an appropriate follow-up plan of an incidentally discovered pituitary microadenoma and explain the impact of pregnancy on pituitary volume.

Rationale: This is a case of a nonfunctioning pituitary incidentaloma. Although it can be classified as a microadenoma because of its size (<10.0 mm), it is very close to the optic chiasm. The main issue identified in this vignette is that the patient would like to become pregnant. Thus, the real determination is identifying which of the offered options would maximize her chances of a safe pregnancy. This is relevant because the anterior lobe of the pituitary gland enlarges up to 3-fold during gestation due to hyperplasia and hypertrophy of lactotrophs. In addition, these changes do not completely resolve in the immediate postpartum period; however, pituitary MRI demonstrates a return to normal volume within 6 months after delivery. The current Endocrine Society clinical practice guideline for the management of pituitary incidentaloma recommends that pituitary surgery be considered (Answer A) in patients such as those described in this vignette to reduce the risk of visual compromise during pregnancy. However, any discussion of pituitary surgery in these patients must also include the small risk of postoperative hypopituitarism and its implications for subsequent fertility.

Rationale Continued: While medical treatment of pituitary incidentalomas has not been systematically studied, some studies have demonstrated tumor shrinkage with the use of dopamine agonist therapy. However, this is highly variable and most of these studies were conducted in patients who had previously undergone debulking surgery. This is also true of somatostatin analogues such as octreotide. Therefore, there is currently insufficient evidence to routinely recommend the use of medical therapy (Answers B and C) in the management of pituitary incidentalomas. Moreover, cabergoline should usually be discontinued once it is known that a patient is pregnant (however, there is a case to continue cabergoline in “high-risk” pregnancies). Although there is no convincing evidence of teratogenic effects of octreotide, its use in pregnancy should be limited to scenarios where the benefits outweigh the risks ( eg , acromegaly with pituitary macroadenoma). Regardless of pregnancy in this patient, she does require radiologic follow-up of her pituitary tumor, particularly given its proximity to the optic chiasm. However, waiting 2 years (Answer D) is incorrect because a subsequent MRI should be considered 6 to 12 months after the initial MRI. Although pituitary MRI (Answer E) can be performed in pregnancy, this should not be routinely advised and should be reserved for pregnant patients with pituitary tumors who develop a new visual field defect or worsening headache. Therefore, it would be more appropriate to recommend close monitoring of visual fields in this patient if she becomes pregnant and recommend pituitary MRI only if there is concern.

Reference(s): Freda PU, Beckers AM, Katznelson L, et al; Endocrine Society. Pituitary incidentaloma: an Endocrine Society clinical practice guideline. J Clin Endocrinol Metab . 2011;96(4):894-904. PMID: 21474686Gonzalez JG, Elizando G, Saldivar D, Nanez H, Todd LE, Villarreal JZ. Pituitary gland growth during normal pregnancy: an in vivo study using magnetic resonance imaging. Am J Med. 1988;85(2):217-220. PMID: 3400697Melmed S, Casanueva FF, Hoffman AR, et al; Endocrine Society. Diagnosis and treatment of hyperprolactinema: an Endocrine Society clinical practice guideline. J Clin Endocrinol Metab. 2011;96(2):273-288. PMID: 21296991

ITE 2019 Question 50 A 32-year-old pregnant woman with a history of acromegaly comes for a visit at 18 weeks’ gestation. She was treated by debulking pituitary surgery 3 years ago, but a small tumor remnant remained in the cavernous sinus. As a result, IGF-1 and GH levels failed to normalize after surgery and she required management with intramuscular lanreotide with the subsequent addition of pegvisomant . She feels well and has no concerns. On physical examination, she has mild acromegalic facies. Her blood pressure is 112/78 mm Hg, and pulse rate is 100 beats/min. Visual fields are normal. Laboratory test results 2 months before pregnancy:GH = 1.8 ng/mL (0.01-3.61 ng/mL) (SI: 1.8 µg/L [0.01-3.61 µg/L])IGF-1 = 195 ng/mL (113-297 ng/mL) (SI: 25.5 µg/L [14.8-38.9 µg/L])Prolactin = 19 ng/mL (4-30 ng/mL) (SI: 0.83 µg/L [0.17-1.30 µg/L])Current laboratory test results:GH = 4.0 ng/mL (SI: 4.0 µg/L)IGF-1 = 320 ng/mL (SI: 41.9 µg/L)Prolactin = 220 ng/mL (SI: 9.6 µg/L)

Pituitary-directed MRI is shown (see image). Residual tumor is visible in the left side of the pituitary (white arrow), extending into cavernous sinus (stable compared with MRI 1 year ago).

Which of the following should you recommend as the best next management step for this patient? Continue current therapy Stop pegvisomant and continue lanreotide Stop pegvisomant and lanreotide; start cabergolineStop current therapy; restart if IGF-1 continues to riseStop current therapy; do not routinely monitor IGF-1 during pregnancy Correct Answer: E Learning objective : Manage acromegaly during pregnancy and counsel patients about the difficulty of monitoring GH status in this setting.

Rationale: In pregnancy, the placenta produces a biologically active placental variant of GH, which is indistinguishable from pituitary GH on conventional assays. This GH variant stimulates IGF-1 production, which can result in increased levels, particularly in the second half of pregnancy, but it is counterbalanced by increased estrogen. Therefore, normal reference ranges for GH and IGF-1 levels cannot be applied in pregnancy. This patient had well-controlled acromegaly both biochemically and radiologically before her pregnancy. This, along with her lack of significant symptoms, means that the current higher GH and IGF-1 levels can be attributed to her pregnancy and not to disease progression. Somatostatin analogues such as lanreotide cross the placenta and their safety in pregnancy has not been established, although no apparent adverse effects have been confirmed (albeit in very small numbers). Similarly, safety data on the GH receptor antagonist pegvisomant are lacking. One global safety database describes 30 pregnancies exposed to pegvisomant and 3 pregnancies in which pegvisomant was used throughout; no adverse outcomes were described in either circumstance. However, it is recommended that, when possible, somatostatin analogues and pegvisomant be discontinued before conception and not taken during pregnancy.

Rationale Continued: This is a case of stable disease without evidence of optic chiasm compromise on a prepregnancy pituitary MRI. Therefore, the optimal and safest course of action is to stop all therapy and to not monitor IGF-1 (Answer E). This patient will require close monitoring during pregnancy with regular assessment for symptoms and visual field testing. Medical therapy of acromegaly may need to be reinstituted in the setting of tumor growth or worsening headaches. In this situation, cabergoline (Answer C) has been shown to be safe when used in the management of macroprolactinoma in pregnancy and could be an option in this case only if she had genuine evidence of disease progression. Because GH and IGF-1 levels cannot be used to guide assessment of disease activity in pregnancy for reasons described above, stopping therapy and restarting if IGF-1 levels rise (Answer D) is incorrect. Continuation of current therapy (Answer A) is not required and it would potentially expose the fetus to unnecessary risk. There are no safety data that favor pegvisomant over somatostatin analogues, so it is not necessary to stop one drug and continue the other (Answer B).

Reference(s): Katznelson L, Laws ER Jr, Melmed S, et al; Endocrine Society. Acromegaly: an Endocrine Society clinical practice guideline. J Clin Endocrinol Metab . 2014;99(11):3933-3951. PMID: 25356808Neggers SJ, Franck SE, de Rooij FW, et al. Long-term efficacy and safety of pegvisomant in combination with long-acting somatostatin analogs in acromegaly. J Clin Endocrinol Metab. 2014;99(10):3644-3652. PMID: 24937542Fleseriu M. Medical treatment of acromegaly in pregnancy, highlights on new reports. Endocrine. 2015;49(3):577-579. PMID: 25931411

ITE 2019 Question 78 A 36-year-old woman has developed hypertension, hirsutism, and wide, purple striae on her abdomen as she enters her second trimester of pregnancy. Laboratory test results: Serum cortisol (8 AM) = 33 μ g/dL (5-25 μ g/dL) (SI: 910.4 nmol/L [137.9-689.7 nmol/L])ACTH = 145 pg/mL (10-60 pg/mL) (SI: 31.9 pmol/L [2.2-13.2 pmol/L])Urinary free cortisol = 375 µg/24 h (4-50 µg/24 h) (SI: 1035 nmol/d [11-138 nmol/d])MRI shows a 6-mm pituitary adenoma.

Which of the following options should you recommend? Defer therapy until after delivery Begin ketoconazole Begin mifepristone Begin pasireotide Refer for transsphenoidal surgery Correct Answer: ELearning objective:Treat Cushing disease during pregnancy.

Rationale: Treatment of Cushing disease during pregnancy results in better fetal outcomes. Thus, deferring therapy until after delivery (Answer A) is inappropriate. In expert neurosurgical hands, transsphenoidal surgery (Answer E) has a cure rate of 80% to 90% for microadenomas with very low complication and fetal loss rates when performed in the second trimester. Ketoconazole (Answer B) has a black box warning regarding liver function abnormalities; it has never been approved for use during pregnancy and normalizes urinary free cortisol levels in only about 50% of patients. Mifepristone (Answer C) (also known as RU486) was originally developed as a progesterone receptor blocker and is a potent abortifacient; therefore, its use in pregnancy is absolutely contraindicated. Pasireotide (Answer D) has been used successfully to treat Cushing disease, but there is no experience with its use during pregnancy. Data regarding the use of other somatostatin analogues during pregnancy are also limited, and somatostatin receptors are widely distributed in the fetal brain and other tissues. Currently, it is not recommended that somatostatin analogues be used during pregnancy. 

Reference(s): Marions L. Mifepristone dose in the regimen with misoprostol for medical abortion. Contraception . 2006;74(1):21-25. PMID: 16781255 Lindsay JR, Jonklaas J, Oldfield EH, Nieman LK. Cushing’s syndrome during pregnancy: personal experience and review of the literature. J Clin Endocrinol Metab. 2005;90(5):3077-3083. PMID: 15705919Cohen-Kerem R, Railton C, Oren D, Lishner M, Koren G. Pregnancy outcome following non-obstetric surgical intervention. Am J Surgery. 2005;190(3):467-473. PMID: 16105538Molitch ME. Current approaches to the pharmacological management of Cushing's disease. Mol Cell Endocrinol. 2015;408:185-189. PMID: 25450859

ITE 2019 Question 88 A 31-year-old woman was hospitalized after a motor vehicle hit her while she was crossing the street. She was previously healthy and was taking no medications before admission. She sustained pneumothorax, multiple rib fractures, and a femoral fracture. After initial stabilization, she underwent surgery to evacuate a right subdural hematoma. No sellar masses were noted on MRI. After initial resuscitation, her blood pressure was 112/75 mm Hg, pulse rate was 82 beats/min, and urine output was 90 to 120 cc/h. The patient is now somnolent but arousable and can follow simple commands. Her lungs are clear to auscultation. There is no peripheral edema. During the first postoperative week, she is noted to have persistent hyponatremia (serum sodium, 125-128 mEq /L [125-128 mmol/L]) without new findings on physical examination. Laboratory test results: Serum sodium = 127 mEq/L (136-142 mEq/L) (SI: 127 mmol/L [136-142 mmol/L])Urinary sodium = 54 mEq/L (SI: 54 mmol/L)Urine osmolality = 440 mOsm/kg (150-1150 mOsm/kg) (SI: 440 mmol/kg [150-1150 mmol/kg])TSH = 0.6 mIU/L (0.5-5.0 mIU/L)Free T4 = 0.9 ng/dL (0.8-1.8 ng/dL) (SI: 11.6 pmol/L [10.30-23.17 pmol/L])Total T3 = 60 ng/dL (70-200 ng/dL) (SI: 0.9 nmol/L [1.08-3.08 nmol/L]) Morning serum cortisol = 2.8 µg/dL (SI: 77.2 nmol/L) at baseline, rising to a peak cortisol level of 19 µg/dL (524.2 nmol/L) after cosyntropin administration (250 mcg)

Which of the following is the best next management step? Administration of hydrocortisone in stress doses Administration of levothyroxine Fluid restriction Administration of 3% saline Administration of conivaptan Correct Answer: A Learning objective:Diagnose and manage central hypoadrenalism occurring after traumatic brain injury.

Rationale: Hypopituitarism is common after traumatic brain injury. In the acute phase, central hypoadrenalism may present with hypotension or hyponatremia. The occurrence of hyponatremia in patients with hypoadrenalism is a consequence of a decrease in free water clearance, which is partly mediated by an increase in vasopressin secretion, as well as changes in renal hemodynamics. In the present case, the low morning serum cortisol level is diagnostic of hypoadrenalism, despite the normal cortisol response after cosyntropin administration (which is anticipated in a patient with a pituitary insult of recent onset). In this patient, hyponatremia resolved after administration of glucocorticoids in stress doses (Answer A). The presence of normal TSH and free T4 levels is consistent with normal thyroid function in this patient; the low T3 level is most consistent with euthyroid sick syndrome. Therefore, administration of levothyroxine (Answer B) would be inappropriate. In addition, administration of levothyroxine in the setting of adrenal insufficiency can precipitate adrenal crises, presumably by enhancing the metabolism of any residual cortisol in the circulation. It is also important to keep in mind that the diagnosis of the syndrome of inappropriate antidiuretic hormone secretion requires the exclusion of adrenal insufficiency and hypothyroidism as potential causes of hyponatremia. Thus, fluid restriction (Answer C), which is recommended in patients with syndrome of inappropriate antidiuretic hormone secretion, would not be advised as the immediate next step in management. Administration of 3% saline (Answer D) is not indicated in the absence of symptomatic, severe hyponatremia. Administration of conivaptan, a vasopressin antagonist (Answer E), is not indicated in this patient who does not meet diagnostic criteria for the syndrome of inappropriate antidiuretic hormone secretion.

Reference(s): Sundaram NK, Geer EB, Greenwald BD. The impact of traumatic brain injury on pituitary function. Endocrinol Metab Clin North Am. 2013;42(3):565-583. PMID: 24011887 Hannon MJ, Crowley RK, Behan LA, et al. Acute glucocorticoid deficiency and diabetes insipidus are common after acute traumatic brain injury and predict mortality. J Clin Endocrinol Metab. 2013;98(8):3229-3237. PMID: 23690314

THYROID

ITE 2019 Question 5 A 32-year-old woman who is 4 months postpartum following the birth of her second child is referred for evaluation. Graves disease was diagnosed 2 years earlier and this was initially treated with methimazole. Her antithyroid drug regimen was changed to propylthiouracil 3 months before she became pregnant, and antithyroid drug treatment was discontinued at 16 weeks’ gestation. She reports that she felt nauseated while taking propylthiouracil. The patient remained euthyroid during pregnancy but has now presented with a 4-week history of palpitations, weight loss of 11 lb (5 kg), heat intolerance, and tremulousness. She reports that she is breastfeeding her baby and she plans to continue. On physical examination, her height is 68 in (172.7 cm) and weight is 150 lb (68.2 kg) (BMI = 22.8 kg/m2). Her blood pressure is 124/62 mm Hg, and pulse rate is 84 beats/min. There is a tremor of the outstretched hands, and her hands are moist. She has mild periorbital and conjunctival swelling. A small, soft, diffusely enlarged thyroid gland is palpable in her neck. Laboratory test results:TSH = <0.01 mIU/L (0.5-5.0 mIU/L)Free T4 = 5.1 ng/dL (0.8-1.8 ng/dL) (SI: 65.6 pmol/L [10.30-23.17 pmol/L])Free T3 = 29.3 pg/mL (2.3-4.2 pg/mL) (SI: 45.01 pmol/L [3.53-6.45 pmol/L])TSH-receptor antibodies = 23.0 IU/L (≤1.75 IU/L)

Which of the following is the best next step in this patient’s management? Start methimazole and continue breastfeeding Start propylthiouracil and continue breastfeeding Start propranolol and continue breastfeeding Recommend total thyroidectomy and continue breastfeeding Stop breastfeeding due to risk of hyperthyroidism in infant Correct Answer: A Learning objective:Counsel a breastfeeding patient regarding the safety of antithyroid drugs.

Rationale: Thyroid dysfunction is relatively common in the postpartum period. In patients presenting with thyrotoxicosis in the first 12 months following delivery, a differential diagnosis between Graves disease and postpartum thyroiditis must be established. The patient in this vignette has a history of Graves disease that remitted during pregnancy. She now presents with clinical and biochemical findings consistent with relapsed Graves thyrotoxicosis, including signs of ophthalmopathy and elevated TSH-receptor antibody ( TRAb ) concentrations. A few studies have indicated that maternal hyperthyroidism impairs lactation, although the evidence is not consistent and guidelines do not currently recommend treatment of maternal hyperthyroidism be warranted on the grounds of improving lactation. The decision to treat hyperthyroidism in lactating women is governed by the same principles that apply to nonpregnant women. Because the findings in this vignette indicate relapsed Graves hyperthyroidism, antithyroid treatment is warranted and symptomatic control with β-adrenergic–blocking agents only (Answer C) is not sufficient.

Rationale Continued: Studies evaluating the amount of propylthiouracil secreted into breast milk indicated that only 0.007% to 0.077% of the ingested dose is detected. Studies of methimazole confirm a 4- to 7-fold higher proportion of the medication transferred into breast milk compared with propylthiouracil, and approximately 0.1% to 0.2% of an orally administered methimazole dose is excreted into breast milk. A large study of methimazole use during breastfeeding assessed neonatal thyroid function in the breastfeeding offspring, as well as intellectual development and physical growth in a subset of infants, and no difference was observed in IQ or physical development of the breastfeeding children compared with the control children. Overall, experts have confirmed the safety of low to moderate dosages of both propylthiouracil (up to 450 mg daily) and methimazole (up to 20 mg daily) in breastfeeding infants. Given the small but detectable amount of both propylthiouracil and methimazole transferred into breast milk, the lowest effective dosages of these medications should be administered. The patient in this vignette has previously experienced adverse effects when taking propylthiouracil (Answer B) and it would seem more appropriate to start methimazole (Answer A). Breastfed children of women who are treated with antithyroid drugs should be monitored for appropriate growth and development during routine pediatric health and wellness evaluations. Routine assessment of serum thyroid function in the child is not recommended as long as moderate dosages of antithyroid drugs are used. Fetal Graves disease is caused by transplacental transfer of TRAb . While it is possible that this patient’s baby was affected by the presence of these antibodies during pregnancy, there is no evidence that the high levels of TRAb in the mother are transferred in breast milk, thereby adversely affecting the baby. Similarly, the amount of T4 and T3 that is transferred in breast milk is minimal, and maternal Graves hyperthyroidism is not a contraindication for breastfeeding (thus, Answer E is incorrect).

Rationale Continued: Total thyroidectomy (Answer D) is a suitable therapeutic option for patients with relapsed Graves hyperthyroidism. However, this patient has severe thyrotoxicosis and is likely to require pretreatment with antithyroid drugs before surgery becomes a feasible option. The use of 131I is contraindicated during lactation because of its relatively long half-life (8 days). If required, 123I can be used if breast milk is pumped and discarded for 3 to 4 days before breastfeeding is resumed. Similarly, 99mTc pertechnetate administration requires breast milk to be pumped and discarded during the day of testing.

Reference(s): Azizi F, Khoshniat M, Bahrainian M, Hedayati M. Thyroid function and intellectual development of infants nursed by mothers taking methimazole. J Clin Endocrinol Metab. 2000;85(9):3233-3238. PMID: 10999814Alexander EK, Pearce EN, Brent GA, et al. 2017 guidelines of the American Thyroid Association for the diagnosis and management of thyroid disease during pregnancy and the postpartum. Thyroid. 2017;27(3):315-389. PMID: 28056690

ITE 2019 Question 25 A 71-year-old man with hypothyroidism is admitted to the hospital after a stroke. As an outpatient, he was taking levothyroxine, 88 mcg daily. His 2 most recent TSH measurements (while on this dosage of levothyroxine) were 3.2 and 4.1 mIU /L. He has not yet resumed oral medication or feeding because of his persistent swallowing difficulties. Currently, he is receiving continuous tube feedings and medications are being delivered via the nasogastric tube.

On physical examination, his thyroid gland is small, without palpable nodules. Laboratory data from his hospital stay are shown (see table). Measurement 1 Week 2 Weeks 3 Weeks TSH (0.5-5.0 mIU/L) 15.7 mIU/L23.0 mIU/L 35.0 mIU/L Free T 4 (0.8-1.8 ng/dL [SI: 10.30-23.17 pmol/L]) … 0.6 ng/dL (SI: 7.7 pmol/L) 0.5 ng/dL (SI: 6.4 pmol/L) Total T 3 (70-200 ng/dL [SI: 1.08-3.08 nmol/L]) … … 60 ng/dL (SI: 0.9 nmol/L) Levothyroxine dosage 88 mcg daily 100 mcg daily 112 mcg daily

Which of the following is the best management plan for this patient? Switch his thyroid hormone replacement to liothyronine, 25 mcg orally twice daily Add liothyronine, 5 mcg orally twice daily Increase his oral levothyroxine dosage to 250 mcg daily Switch his regimen to levothyroxine, 75 mcg daily given intravenously Administer intravenous potassium iodine Correct Answer: D Learning objective:Manage levothyroxine therapy in a hospitalized patient receiving enteral feeding.

Rationale: Foods, supplements, and other medications can all interfere with levothyroxine absorption. Chief among interfering components of foods and supplements are iron and calcium. Tube feeds given to hospitalized patients can also interfere with levothyroxine absorption. The easiest way to ensure adequate circulating levels of thyroid hormone when poor absorption has resulted in overt hypothyroidism is to deliver the levothyroxine intravenously. When given orally, about 75% to 80% of an administered dose is absorbed. Therefore, when converting an oral dosage to an intravenous dosage, about 75% to 80% of the oral dosage can be administered. Some experts recommend using a lower intravenous dose that is 50% of the oral dose. This patient was euthyroid while taking 88 mcg of levothyroxine orally; 80% of this dose is 70 mcg, making 75 mcg daily given intravenously a reasonable dosage (Answer D), and this is the best management plan. Increasing the patient’s levothyroxine dosage from 112 mcg to 250 mcg (Answer C) might compensate for the impaired absorption. However, it is difficult to know what dosage of levothyroxine would be appropriate, and temporary use of intravenous levothyroxine may be a more expeditious way of normalizing the patient’s serum TSH level. Liothyronine (Answer A) is better absorbed than levothyroxine, with about 90% absorption being documented. Although use of liothyronine may be associated with better absorption than levothyroxine in this particular case, use of liothyronine is generally not recommended in hospitalized patients, other than in the setting of myxedema coma. Use of combination therapy by addition of liothyronine to the patient’s levothyroxine (Answer B) has not been adequately studied in hospitalized patients, although it has been used to treat myxedema coma.

Rationale Continued: Consumptive hypothyroidism is an unusual cause of a high levothyroxine requirement. This condition has typically been described in children with hemangiomas overexpressing the type 3 deiodinase, but can be found in certain gastrointestinal stromal tumors, which also overexpress the type 3 deiodinase. Associated with the high levels of this deiodinase, which converts T4 to reverse T3 and T3 to diiodothyronine (T2), there is a requirement for massive levothyroxine dosages to adequately treat hypothyroidism. Patients with such tumors have been noted to require levothyroxine dosages as high as 300 to 400 mcg daily. Although gastrointestinal stromal tumors are the most common mesenchymal tumor of the gastrointestinal tract, they are uncommon (annual incidence is approximately 0.68 per 100,000 in the United States). Furthermore, the prevalence of consumptive hypothyroidism in patients with gastrointestinal stromal tumors is not well studied. The findings in the vignette do not suggest that this diagnosis should be considered in this patient. Iodine deficiency, although a common cause of hypothyroidism in certain regions of the world, is relatively rare in the United States. Without being given a history of thyroidectomy or other procedure resulting in destruction of the thyroid, it would be assumed that this patient has Hashimoto hypothyroidism. Administration of intravenous iodine (Answer E) is of no benefit to treat hypothyroidism in an iodine-replete patient, and it may even worsen hypothyroidism by inhibiting any endogenous thyroid function that remains. Iodine, however, is part of the regimen used to treat thyroid storm, although it is usually given orally.

Reference(s): Jonklaas J, Bianco AC, Bauer AJ, et al; American Thyroid Association Task Force on Thyroid Hormone Replacement. Guidelines for the treatment of hypothyroidism: prepared by the American Thyroid Association Task Force on Thyroid Hormone Replacement. Thyroid . 2014;28(12):1670-1751. PMID: 25266247 Maynard MA, Marino-Enriquez A, Fletcher JA, et al. Thyroid hormone inactivation in gastrointestinal stromal tumors. N Engl J Med. 2014;370(14):1327-1334. PMID: 24693892Stathatos N, Wartofsky L. Perioperative management of patients with hypothyroidism. Endocrinol Metab Clin North Am. 2003;32(2):503-518. PMID: 12800543

ITE 2019 Question 30 A 42-year-old man is noted to have a serum TSH value of 12.1 mIU /L (0.5-5.0 mIU /L). He is asymptomatic and physical examination findings are normal. Further laboratory test results: Free T4 = 1.3 ng/dL (0.8-1.8 ng/dL) (SI: 16.7 pmol/L [10.30-23.17 pmol/L])TPO antibodies, negativeThyroglobulin antibodies, negativeThyroid ultrasonography is normal.One of the patient’s 2 children has similar laboratory test results.

Which of the following is the most likely source of this patient’s thyroid abnormality? TSH resistance Hashimoto thyroiditis Adrenal insufficiency Resistance to thyroid hormone Excess selenium exposure Correct Answer: A Learning objective:Distinguish resistance to thyrotropin from subclinical hypothyroidism.

Rationale: This patient’s thyroid function test results might suggest subclinical hypothyroidism, but he has no goiter, thyroid ultrasonography is normal, and TPO antibodies are negative. This case is typical of inactivating pathogenic variants in the gene encoding the TSH receptor. Thus, TSH resistance (Answer A) is correct. This situation may be more common than once realized. One study found that 4 of 10 patients with findings similar to those of this patient had inactivating pathogenic variants in the TSH receptor gene. There is an autosomal dominant pattern of inheritance, but with variable clinical expression. In one study, 64% of offspring were affected. TSH resistance can results from other genetic abnormalities. For example, pathogenic variants in the PAX8 gene have also been linked to TSH resistance. Although the most common cause of subclinical hypothyroidism is Hashimoto thyroiditis (Answer B), this patient has negative TPO antibodies, no goiter, and homogeneous echotexture on ultrasonography, which, taken together, virtually exclude this disorder. Adrenal insufficiency (Answer C) can be associated with an isolated mild elevation in TSH due to an absence of the tonic suppressive effect of corticosteroids, but generally not to this degree. Furthermore, this patient has no clinical evidence to support a diagnosis of adrenal insufficiency. Patients with resistance to thyroid hormone (Answer D) have elevated, rather than normal, levels of thyroid hormones. Selenium excess (Answer E) does not cause thyroid dysfunction.

Reference(s): Alberti L, Proverbio MC, Costagliola S, et al. Germline mutations of TSH receptor gene as cause of nonautoimmune subclinical hypothyroidism. J Clin Endocrinol Metab. 2002;87(6):2549-2555. PMID: 12050212Grasberger H, Mimouni-Bloch A, Vantyghem MC, et al. Autosomal dominant resistance to thyrotropin as a distinct entity in five multigenerational kindreds: clinical characterization and exclusion of candidate loci. J Clin Endocrinol Metab. 2005;90(7):4025-4034. PMID: 15870119

ITE 2019 Question 38 A 72-year-old man presents with a 6-month history of an enlarging neck swelling. He describes a feeling of choking at night and increasing dysphagia when eating solid foods. He has mild shortness of breath on exertion and reports “3-pillow” orthopnea. He has no symptoms of thyroid dysfunction, and his weight has remained stable over the last 12 months. He underwent quadruple-vessel bypass surgery for ischemic heart disease 11 months ago. His current medications are aspirin, lisinopril, and atorvastatin. There is no notable family history. On physical examination, his trachea is deviated to the left, and his lung fields are clear. A right-sided goiter is visible in his neck, and palpation reveals a smooth, 4 x 5-cm, right-sided thyroid swelling that moves with swallowing. There is no palpable cervical lymphadenopathy. The Pemberton sign is negative, and there is no clinical evidence of retrosternal extension.

Laboratory test results: TSH = 3.7 mIU /L (0.5-5.0 mIU /L) Free T4 = 1.3 ng/dL (0.8-1.8 ng/dL) (SI: 16.7 pmol/L [10.30-23.17 pmol/L])Thyroid ultrasonography indicates the presence of a right-sided, large (32 x 38 x 51 mm), well-defined, thin-walled, anechoic thyroid lesion with a minimal amount of isoechoic solid tissue causing a small degree of tracheal deviation. The nodule has regular margins, no microcalcifications, and no other suspicious features.

Which of the following is the most likely diagnosis? Anaplastic thyroid cancer Differentiated thyroid malignancy with cystic degeneration Benign thyroid cyst Multinodular goiter Thyroglossal duct cyst Correct Answer: C Learning objective:Identify ultrasound features of a large, benign thyroid cyst.

Rationale: This patient presents with a slowly enlarging neck swelling that is causing compressive symptoms. Clinical findings are unilateral thyroid enlargement without retrosternal extension. He has no clinical features of thyroid dysfunction. Because thyroid enlargement caused by thyroid dysfunction requires different management than euthyroid nodules and goiters, the first evaluation in patients presenting with thyroid swelling should be measurement of serum TSH (which is within normal limits in this patient). If serum TSH is subnormal, a radioisotope scan should be performed to assess for the presence of toxic nodules. To assess the size and nature of the thyroid nodule, high-resolution thyroid ultrasonography with survey of the cervical lymph nodes should be performed. The ultrasound report should convey nodule size (in 3 dimensions) and location, as well as a description of the nodule’s sonographic features, including composition (solid, cystic proportion, or spongiform), echogenicity, margins, presence and type of calcifications, shape ( eg , taller than wide), and vascularity. The pattern of sonographic features associated with a nodule confers a risk of malignancy, and combined with nodule size, this guides decision-making with regard to FNA biopsy. In this patient, the ultrasonographic findings are consistent with a benign thyroid cyst (Answer C).

Rationale Continued: While multiple thyroid nodules may be present in patients presenting with a visible and palpable dominant nodule, there is no evidence of multiple nodules either on clinical or radiologic grounds and the presence of a multinodular goiter (Answer D) is less likely. Anaplastic thyroid cancer (Answer A) is the most aggressive and rarest form of thyroid cancer, often originating from a differentiated tumor following a process of dedifferentiation. Affected persons typically present with locally advanced disease, including tracheal and esophageal invasion, carotid artery involvement, and distant metastases. The clinical presentation and ultrasonographic findings of a large cystic mass with regular margins and not invading adjacent structures make a diagnosis of anaplastic thyroid cancer unlikely. Differentiated thyroid cancers can undergo cystic degeneration (Answer B), but in the absence of suspicious ultrasound features such as an irregular nodule margin, microcalcifications, and a large, less homogeneous solid component, this diagnosis is less likely. A thyroglossal cyst (Answer E) is a fibrous cyst that originates from a persistent thyroglossal duct during developmental stages, and it usually presents as an irregular neck mass or a lump. Thyroglossal cysts are the most common cause of midline neck masses and are generally located caudal to the hyoid bone. These neck masses can occur anywhere along the path of the thyroglossal duct, from the base of the tongue to the suprasternal notch, but they are usually located in the midline. Typically, the cyst moves upwards on protrusion of the tongue, given its attachment to the embryonic duct. While the image in this vignette may represent a thyroglossal duct cyst, the thyroid swelling is not located in the midline and there is no suggestion that this moves with tongue protrusion.

Rationale Continued: Surgery is the long-established therapeutic option for benign thyroid nodules, which steadily grow and become symptomatic. The cost of thyroid surgery, the risk of temporary or permanent complications, and the effect on quality of life, however, remain relevant concerns. Therefore, various minimally invasive treatments have gained momentum in recent years. Ultrasound-guided percutaneous ethanol injection is a very effective treatment for relapsing thyroid cysts, and radiofrequency ablation of solid, nonfunctioning nodules usually results in a 50% volume decrease. Hyperfunctioning nodules remain best treated with radioactive iodine, which results in better control of hyperthyroidism and reduction in thyroid size. Newer minimally invasive approaches that are being investigated include high-intensity focused ultrasound and microwave or laser ablation.

Reference(s): Haugen BR, Alexander EK, Bible KC, et al. 2015 American Thyroid Association management guidelines for adult patients with thyroid nodules and differentiated thyroid cancer: the American Thyroid Association Guidelines Task Force on Thyroid Nodules and Differentiated Thyroid Cancer. Thyroid . 2016;26(1):1-133. PMID: 26462967 Papini E, Gugliemi R, Pacella CM. Laser, radiofrequency, and ethanol ablation for the management of thyroid nodules. Curr Opin Endocrinol Diabetes Obes. 2016;23(5):400-406. PMID: 27504993

ITE 2019 Question 51 An 82-year-old woman has been referred to evaluate a neck swelling that has been present for at least 30 years. The size of the swelling has gradually increased. The patient reports the recent development of difficulty swallowing, mostly solid food, as well as a feeling of choking, especially when lying flat. She also reports increasing shortness of breath, most notably when climbing stairs. There is no notable family history. On physical examination, she has a multinodular goiter that is visible in an erect position with neck extension. A dominant left-sided thyroid nodule is palpable that measures 7 x 5 cm. The lower lobe of the thyroid gland extends below the sternal notch and cannot be palpated when the patient is lying in the supine position with mild neck extension or with swallowing. The Pemberton sign is negative. There is tracheal deviation to the right, but no audible stridor and lung fields are clear. Laboratory test results: Serum TSH = 4.7 mIU /L (0.5-5.0 mIU/L)Serum free T4 = 1.1 ng/dL (0.8-1.8 ng/dL) (SI: 14.2 pmol/L [10.30-23.17 pmol/L])

Which of the following is the most appropriate assessment to decide if surgery is warranted to alleviate compressive symptoms? Noncontrast CT of the neck and chest Barium swallow Flow-volume loop Thyroid ultrasonography and FNA biopsy of the left-sided nodule 18-Fluorodeoxyglucose PET-CT Correct Answer: ALearning objective:Diagnose large, nontoxic multinodular goiter.

Rationale: Symptoms and signs of goiter depend on the size and location although often the relationship between size, morphology, and function of the thyroid gland and the symptoms of the individual patient is not clear-cut. The symptoms and signs of goiter are usually related to compression of the trachea or esophagus and these occur more often when there is an intrathoracic extension of the goiter. Such goiter often develops insidiously in elderly patients with longstanding goiter (such as the patient in this vignette). When there is substantial growth into the thorax, usually into the anterior mediastinum, the thoracic inlet may become occluded, known as “the thyroid cork phenomenon.” Evidence of venous outflow obstruction ( ie , compression or thrombosis of the jugular or subclavian veins or the superior vena cava) can be revealed by extending the arms over the head (Pemberton maneuver), which raises the goiter into the thoracic inlet, resulting in shortness of breath, stridor, distension of neck veins, or facial plethora. The symptoms of tracheal compression are dyspnea, stridor, cough, and choking sensation. Tracheal compression is described as the greatest percentage reduction in tracheal diameter and is more common in substernal goiter than in cervical goiter. Earlier reports recommended thyroidectomy for any degree of luminal narrowing on CT imaging, but more recent studies have found that positional dyspnea improves or resolves after surgery for substernal goiter, varied stepwise with the degree of preoperative tracheal compression. Narrowing of less than 35% is associated with a 65% to 70% likelihood of resolution of positional dyspnea, whereas narrowing of 35% or greater is associated with a 95% to 98% chance of resolution after resection. A consensus statement concludes that surgery may be reasonable for patients with smaller degrees of tracheal compression if they are symptomatic or for younger patients to prevent progression. Asymptomatic, elderly, and/or medically infirm patients may be followed up conservatively for evidence of progression.

Rationale Continued: Clinical features due to esophageal compression, including dysphagia for solids, are less common and must be distinguished from those secondary to laryngopharyngeal reflux. Even more rare is vocal cord paralysis, which can be transient or permanent, caused by stretching or compression of one or both recurrent laryngeal nerves. Phrenic nerve paralysis and Horner syndrome, due to compression of the cervical sympathetic chain, have been described but are extremely rare. Although high-resolution thyroid ultrasonography is the standard practice to evaluate any kind of thyromegaly, this imaging modality is more limited in cases with substernal extension, and cross-sectional imaging with CT should preferentially be performed (Answer A). CT scanning is especially useful in assessing the mass effect on the trachea and in determining the diameter of the trachea. Often noncontrast CT scans are sufficient unless the goiter extends caudally beyond the aortic arch. Ultrasonography (Answer D) will better define the size of the nodule as well as the presence of other nodules, at least for the portion of the thyroid that is not substernal. Ultrasound-guided FNA biopsy could also provide important information regarding the potential presence of malignancy. However, in view of the significant compressive symptoms in this patient, Answer A is a better option. Many centers consider flow-volume loops (Answer C) to be an important part of the workup of patients with thyroid enlargement. A number of studies have indicated their usefulness in detecting upper airway obstruction secondary to tracheal compression from a goiter. While this investigation is likely to shed further light on the patient’s symptoms of dyspnea and choking, it will not evaluate the presence of esophageal compression. Similarly, a barium swallow (Answer B) will provide important information relating to the patient’s dysphagia but will not assess the patient’s symptoms of potential tracheal compression. 18-Fluorodeoxyglucose PET-CT (Answer E) scanning is useful in the follow-up of patients with thyroid cancer, especially in those with raised serum thyroglobulin concentrations and negative iodine whole-body scans, but it is unlikely to add significant further information regarding this patient’s symptoms.

Reference(s): Hegedus L, Bonnema SJ, Bennedbaek FN. Management of simple nodular goiter: current status and future perspectives. Endocr Rev. 2003;24(1):102-132. PMID: 12588812Chen AY, Bernet VJ, Carty SE, et al; Surgical Affairs Committee of the American Thyroid Association. American Thyroid Association statement on optimal surgical management of goiter. Thyroid. 2014;24(2):181-189. PMID: 24295043

ITE 2019 Question 53 A 68-year-old man with refractory atrial fibrillation is prescribed amiodarone. Baseline thyroid function is normal. One month later, the patient is asymptomatic, but is noted to have the following laboratory findings: Total T 4 = 13.4 μ g/dL (5.5-12.5 μg/dL) (SI: 172.5 nmol/L [94.02-213.68 nmol/L])Free T4 = 1.91 ng/dL (0.8-1.8 ng/dL) (SI: 24.6 pmol/L [10.30-23.17 pmol/L])Total T3 = 65 ng/dL (70-200 ng/dL) (SI: 1.0 nmol/L [1.08-3.08 nmol/L])TSH = 3.9 mIU/L (0.5-5.0 mIU/L)

Which of the following is the most likely explanation for these findings? Type 1 amiodarone-induced thyrotoxicosis Type 2 amiodarone-induced thyrotoxicosis Expected changes in euthyroid patients on amiodarone Assay interference by amiodarone metabolites Euthyroid sick syndrome Correct Answer: C Learning objective:Distinguish expected changes in thyroid function parameters in patients taking amiodarone from amiodarone-induced thyroid dysfunction.

Rationale: Apart from drug-induced hyperthyroidism (5% of treated patients) or hypothyroidism (7%), amiodarone has dramatic effects on thyroid function tests in clinically euthyroid patients. A large iodine load (74 mg total iodine, 7.4 mg of free iodine per 200 mg tablet) is delivered with each dose. Amiodarone inhibits both peripheral and central ( intrapituitary ) conversion of T 4 to T 3 through its action on type 1 and type 2 5'-monodeiodinase, respectively. Lastly, amiodarone has T3 antagonistic effects at the nuclear level. The common pattern shown in euthyroid patients is a high free and total T4, a low-normal T3, and a high-normal TSH (thus, Answer C is correct). These changes tend to persist over time, although serum TSH values may gradually normalize in some patients.Amiodarone and its metabolites are not known to cause artifactual interference with thyroid function assays (Answer D). This patient’s normal TSH and lack of symptoms exclude thyrotoxicosis (Answers A and B). This patient is not acutely ill (Answer E).

Reference(s): Danzi S, Klein I. Amiodarone-induced thyroid dysfunction. J Intensive Care Med. 2015;30(4):179-185. PMID: 24067547Basaria S, Cooper DS. Amiodarone and the thyroid. Am J Med. 2005;118(7):706-714. PMID: 15989900

ITE 2019 Question 56 A 23-year-old woman in her 12th week of pregnancy is found to have a 1.9-cm thyroid nodule. She has no associated adenopathy in the neck. Laboratory test results: Serum TSH = 0.1 mIU /L Free T 4 = 1.2 ng/dL (0.8-1.8 ng/dL) (SI: 15.4 pmol/L [10.30-23.17 pmol/L])Thyroid ultrasonography shows intranodular vascularity, but no other suspicious findings.

Which of the following should be the next step in this patient’s management? Fine-needle aspiration biopsy Technetium thyroid scan Thyroid lobectomy Initiation of methimazole Initiation of propylthiouracil Correct Answer: A Learning objective:Recommend fine-needle aspiration biopsy of a suspicious nodule despite mildly low TSH in the first trimester of pregnancy.

Rationale: This patient has a suppressed serum TSH level. If she were not in the first trimester of pregnancy, this would suggest the possibility of an autonomous nodule, which would not require FNA biopsy. However, in this case, the suppressed TSH is most likely due to the effects of hCG on the thyroid during the first trimester. Therefore, her nodule must be approached as it would be in a nonpregnant patient with a normal TSH level—with thyroid ultrasonography and FNA biopsy (Answer A). Nuclear thyroid scanning (Answer B), although appropriate in the workup of thyroid nodules in the setting of low serum TSH outside pregnancy, is not acceptable during pregnancy. Mildly suppressed TSH, which occurs in approximately 20% of pregnancies during the first trimester, does not require treatment with antithyroid drugs (Answers D and E). Thyroid lobectomy (Answer C) is not a reasonable response before performing FNA biopsy.

Reference(s): Stagnaro -Green A, Abalovich M, Alexander E, et al; American Thyroid Association Taskforce on Thyroid Disease During Pregnancy and Postpartum. Guidelines of the American Thyroid Association for the diagnosis and management of thyroid disease during pregnancy and postpartum. Thyroid . 2011;21(10):1081-1125. PMID: 21787128Cooper DS, Laurberg P. Hyperthyroidism in pregnancy. Lancet Diabetes Endocrinol. 2013;1(3):238-249. PMID: 24622372

ITE 2019 Question 62 A 72-year-old man is noted to have a repeatedly low serum TSH level of less than 0.01 mIU /L. Thyroid hormone levels are normal. The patient’s only other medical problem is hypertension that is well controlled on a single agent. He feels well and has no complaints. On physical examination, he has a left-sided nodule that is about 3 cm in dimension and feels firm to palpation. A radioactive iodine uptake and scan shows homogeneously increased iodine uptake throughout the thyroid gland, but with a cold defect corresponding to the patient’s palpable left thyroid nodule. FNA biopsy of the nodule is performed under ultrasound guidance and the cytology is interpreted to be atypia of undetermined significance. No right lobe nodules are identified. Left lobectomy is performed and the nodule is found to be a benign hyperplastic nodule. A 7-mm papillary thyroid cancer is also identified. The surgical margins are negative.

Which of the following is the best next step in this patient’s management? Completion thyroidectomy Levothyroxine therapy to achieve a TSH level between 0.1 and 0.3 mIU /L Radioactive iodine therapy Another thyroid ultrasound Monitoring of the patient’s TSH without further therapy nowCorrect Answer: ELearning objective: Determine the appropriate management of microscopic papillary thyroid cancers.

Rationale: This patient presented with subclinical hyperthyroidism. Given the homogenous uptake throughout his thyroid gland, radioactive iodine therapy could have been administered, were it not for the presence of the nonfunctioning nodule, which appropriately was targeted for FNA biopsy. Given both the hyperthyroidism and the indeterminate cytology, a total thyroidectomy would have been a reasonable treatment approach for the patient’s hyperthyroidism and establishing the histopathology of the left-sided nodule. Because the patient underwent a left lobectomy, the issues to consider are whether he has had sufficient treatment for his microscopic papillary thyroid cancer and whether he will now be euthyroid or will continue to have subclinical hyperthyroidism. With respect to his diagnosis of microscopic papillary thyroid cancer, the patient needs no further treatment. In fact, it is highly likely that the removal of his microscopic thyroid cancer has not altered his life expectancy. Neither a completion thyroidectomy (Answer A) nor radioactive iodine therapy (Answer C) will improve his prognosis. Similarly, TSH suppression (Answer B) is not indicated for a papillary microcarcinoma. Thyroid ultrasonography (Answer D) is not needed now as the patient recently had an ultrasound that did not show any right lobe nodules suggestive of bilateral or multifocal papillary thyroid cancer.

Rationale Continued: With his hyperfunctioning right thyroid lobe remaining in place, it is not clear what the patient’s thyroid status will be. He could now be (1) euthyroid, (2) hyperthyroid, or (3) currently euthyroid, but trend back into hyperthyroidism over time. Given this uncertainty, monitoring his serum TSH (Answer E) is the best approach. Radioactive iodine therapy could be used to treat hyperthyroidism in this patient. However, it has not yet been established that he is hyperthyroid after his lobectomy. Although no trials of treatment of subclinical hyperthyroidism directly address this, it would be anticipated that treatment to resolve his hyperthyroidism would be more likely to decrease adverse outcomes than treatment of his microscopic thyroid cancer.

Reference(s): Carle A, Andersen SL, Boelaert K, Laurberg P. Management of endocrine disease: subclinical thyrotoxicosis: prevalence, causes and choice of therapy. Eur J Endocrinol . 2017;176(6):R325-R337. PMID: 28274949Cecoli F, Ceresola EM, Altrinetti V, et al. Therapeutic strategies and clinical outcome in papillary thyroid microcarcinoma: a multicenter observational study. Eur Thyroid J. 2016;5(3):180-186. PMID: 27843808Haugen BR, Alexander EK, Bible KC, et al. 2015 American Thyroid Association management guidelines for adult patients with thyroid nodules and differentiated thyroid cancer: the American Thyroid Association Guidelines Task Force on Thyroid Nodules and Differentiated Thyroid Cancer. Thyroid. 2016;26(1):1-133. PMID: 26462967Ross DS, Burch HB, Cooper DS, et al. 2016 American Thyroid Association guidelines for diagnosis and management of hyperthyroidism and other causes of thyrotoxicosis. Thyroid. 2016;26(10):1343-1421. PMID: 27521067

ITE 2019 Question 66 A 32-year-old man with widely invasive follicular thyroid cancer undergoes total thyroidectomy followed by radioiodine remnant ablation with 150 mCi   131 I. He subsequently develops lung metastases and is treated with an additional 200 mCi , with an objective decrease in the size of the lung metastases and reduction in serum thyroglobulin levels.

Which of the following is the most likely adverse effect from this patient’s therapy with radioiodine? Permanent loss of taste Excessive dental caries Azoospermia Leukemia Hypoparathyroidism Correct Answer: B Learning objective:Anticipate the most likely adverse effects of high-dose radioactive iodine treatment.

Rationale: Xerostomia is a relatively common sequela of high-dose radioactive iodine therapy, and it is frequently reversible. Approximately 4% to 5% of patients have persistent xerostomia, which predisposes them to excessive caries (Answer B). One study showed a 40% reduction in parotid gland function following 131I doses of 270 mCi . An alteration of taste (Answer A), which commonly occurs after radioiodine therapy, is typically transient rather than permanent. Patients may describe a metallic or chemical taste that occasionally persists for months after therapy. Permanent hypoparathyroidism (Answer E) is a very rare consequence of radioiodine therapy, with only 2 cases described in the literature to date. Transient depression of sperm counts occurs following radioiodine, and a cumulative effect on the testes occurs, manifested as an elevation in serum FSH and slight decreases in sperm count, but not azoospermia (Answer C). It is recommended that men desiring future fertility and who will be receiving radioiodine doses greater than 400 mCi proceed with sperm cryopreservation before treatment. Leukemia (Answer D) is a potential consequence of receiving very high doses of radioiodine, with a frequency of 0.3%, typically occurring in older patients receiving a cumulative dose greater than 800 mCi. Patients who receive the highest doses over the shortest intervals are most likely to develop this rare complication, with a latency period of less than 10 years. Ideally, patients requiring repeated treatments with radioiodine should receive treatment at intervals of at least 1 year, but patients with aggressive radioiodine-avid disease should not be denied therapy at shorter intervals when appropriate.

Reference(s): Clement SC, Peeters RP, Ronckers CM, et al. Intermediate and long-term adverse effects of radioiodine therapy for differentiated thyroid carcinoma--a systematic review. Cancer Treat Rev . 2015;41(10):925-934. PMID: 26421813

ITE 2019 Question 79 A 59-year-old woman with postsurgical hypothyroidism presents for routine follow-up. She has been on a stable levothyroxine dosage for many years. In addition to hypothyroidism, her medical history is notable only for newly diagnosed osteopenia, for which she has started raloxifene and vitamin D, taken at 7AM along with her levothyroxine. She also started calcium carbonate, taken with lunch and evening meals. She describes feeling fatigued, which is new since her last visit. Her serum TSH level is 11.3 mIU /L (0.5-5.0 mIU /L).

Which of the following is the most likely reason for her serum TSH elevation? Medication nonadherence Raloxifene Calcium supplementation Celiac disease Vitamin D supplementation Correct Answer: B Learning objective:Identify raloxifene as an agent that may alter levothyroxine requirements.

Rationale: Concomitant use of raloxifene (Answer B) has been reported to decrease levothyroxine absorption, although the mechanism is unknown. Separating the levothyroxine and raloxifene doses by several hours will alleviate the problem. Medication nonadherence (Answer A) is a very common cause of TSH elevations in levothyroxine-treated patients. However, given the history of long-term levothyroxine dose-stability in this patient, together with the new raloxifene use, nonadherence is less likely in this setting than is interference from raloxifene. Taking supplemental calcium (Answer C) concomitantly with levothyroxine can decrease levothyroxine absorption by up to 60%, but taking it 4 hours apart from the levothyroxine would not be expected to affect levothyroxine requirements. Celiac disease (Answer D) can cause levothyroxine malabsorption, and, because this often co-occurs in patients with autoimmune thyroid disease, celiac disease is an important etiology to exclude in hypothyroid patients who require higher-than-expected levothyroxine dosages. However, new onset of celiac disease is less likely than raloxifene use to be the culprit in this patient, especially given that she is asymptomatic. Vitamin D supplementation (Answer E) is not known to interfere with levothyroxine absorption.

Reference(s): Garwood CL, Van Schepen KA, McDonough RP, Sullivan AL. Increased thyroid-stimulating hormone levels associated with concomitant administration of levothyroxine and raloxifene. Pharmacotherapy . 2006;26(6):881-885. PMID: 16716142 Liwanpo L, Hershman JM. Conditions and drugs interfering with thyroxine absorption. Best Pract Res Clin Endocrinol Metab. 2009;23(6):781-792. PMID: 19942153Skelin M, Lucijanic T, Amidzic Klaric D, et al. Factors affecting gastrointestinal absorption of levothyroxine: a review. Clin Ther. 2017;39(2):378-403. PMID: 28153426

ITE 2019 Question 81 A 66-year-old woman with follicular thyroid cancer and known distant metastases reports rib pain. She initially underwent thyroidectomy 8 years ago, and she has received multiple doses of radioiodine therapy, the last of which was 1 year ago. Her posttreatment scans have been negative. Laboratory test results: TSH = 0.03 mIU /L (0.5-5.0 mIU /L)Thyroglobulin = 3600 ng/mL (<1.0 ng/mL) (SI: 3600 µg/L [<1.0 µg/L])Thyroglobulin antibodies, negativePET-CT demonstrates diffuse bone metastases in the ribs and spine.

Which of the following is most likely to reduce the risk of skeletal complications of her thyroid cancer? Total-body irradiation Zoledronic acid Systemic chemotherapy with alkylating agents Teriparatide Dosimetry-based radioiodine Correct Answer: B Learning objective:Discuss the benefits of bisphosphonate therapy in patients with bone metastases from thyroid cancer.

Rationale: Bisphosphonate therapy (Answer B) has been shown to reduce pain intensity and analgesic requirements in patients with differentiated thyroid cancer with metastases to bone. Bisphosphonates also reduce the frequency of skeletal complications and delay the time to development of those complications in such patients. Although no prospective randomized controlled trial data are available, the data to date in patients with thyroid cancer who have bone involvement, as well as those pertaining to patients with nonthyroidal cancer and metastatic disease to bone, are sufficient to warrant this therapy. The dosing used by Orita et al was zoledronic acid, 4 mg intravenously monthly, for 1 to 33 months (on average 16 doses). Total-body irradiation (Answer A), used in some hematologic malignancies, would not be indicated for focal metastatic disease. Teriparatide (Answer D) is not recommended for use in patients with skeletal malignancies. Systemic chemotherapy (Answer C) is ineffective in the treatment of thyroid cancer. Dosimetry-based radioiodine (Answer E) is incorrect because the patient has already been shown to have non–radioiodine-avid disease.

Reference(s): Orita Y, Sugitani I, Toda K, Manabe J, Fujimoto Y. Zoledronic acid in the treatment of bone metastases from differentiated thyroid carcinoma. Thyroid . 2011;21(1):31-35. PMID: 21058881Wexler JA. Approach to the thyroid cancer patient with bone metastases. J Clin Endocrinol Metab. 2011;96(8):2296-2307. PMID: 21816796

ITE 2019 Question 84 A 43-year-old woman who is having difficulty losing weight is referred for abnormal thyroid function test results. She is otherwise asymptomatic and takes no medications. Her mother has hypothyroidism. On physical examination, her pulse rate is 84 beats/min, her thyroid is slightly enlarged without nodules or bruit, there is no tremor, and deep tendon reflexes are normal. Laboratory test results: TSH = 0.01 mIU /L (0.5-5.0 mIU /L)Free T4 = 1.7 ng/dL (0.8-1.8 ng/dL) (SI: 21.9 pmol/L [10.30-23.17 pmol/L])Total T3 = 150 ng/dL (70-200 ng/dL) (SI: 2.3 nmol/L [1.08-3.08 nmol/L])Radioactive iodine uptake = 22% at 24 hours (15%-30%)

Which of the following is the best next step in this patient's management? Start methimazole, 20 mg daily Start atenolol, 50 mg daily Treat with radioiodine therapy Repeat laboratory tests in 3 months Correct Answer: D Learning objective :Describe the natural history of nonnodular causes of subclinical hyperthyroidism.

Rationale: This patient has subclinical hyperthyroidism, most likely due to TSH receptor–stimulating antibodies. Her family history of probable autoimmune thyroid disease and her own diffuse goiter are also suggestive of this etiology. Patients with mild degrees of subclinical hyperthyroidism, particularly when it is due to thyroiditis or mild Graves disease, very often experience spontaneous resolution. Thus, the best course of action for this patient would be to repeat laboratory tests in 3 months (Answer D). Predictors of reversibility include having detectable (albeit subnormal) TSH values and having etiologies other than nodular hyperthyroidism (toxic adenoma or toxic multinodular goiter), such as thyroiditis or mild Graves disease, as is illustrated by the current case. The natural history of subclinical hyperthyroidism is illustrated (see image).

Rationale Continued: Redrawn from Mai VQ, Burch HB. A stepwise approach to the evaluation and treatment of subclinical hyperthyroidism. Endocr Pract . 2012;18(5):772-780 (adapted from Cooper DS, Biondi B. Subclinical thyroid disease. Lancet. 2012;379(9821):1142-1154.). It is premature to start methimazole (Answer A) in this young patient who has a high likelihood of remission, but this could be considered in an elderly patient with similar findings because of the increased risk of arrhythmia, although the dosage of 20 mg daily would be excessive for this situation. This patient does not require atenolol (Answer B) because she is asymptomatic and has a resting pulse rate less than 90 beats/min. Although treatment with radioiodine therapy (Answer C) could be considered, it would most likely result in permanent hypothyroidism, and she may not require therapy at all with further observation..

Reference(s): Mai VQ, Burch HB. A stepwise approach to the evaluation and treatment of subclinical hyperthyroidism. Endocr Pract . 2012;18(5):772-780. PMID: 22784850